You are on page 1of 131

ENGLISH PRACTICE 1

PART ONE: PHONETICS


I. Choose the word whose underlined part is pronounced differently fromthat of the others)
1. A. supposedly B. markedly C. allegedly D. determinedly
2. A. endure B. feature C. procedure D. measure
3. A. complete B. command C. common D. community
4. A. comb B. climb C. debt D. cable
5. A. neighbour B. height C. sleigh D. weight
II. Choose the word whose stress pattern is different fromthat of the others
1. A. possibility B. disappointed C. manufacture D. instrument
2. A. environment B. mystery C. contribute D. terrific
3. A. deficiency B. psychology C. ecological D. competitor
4. A. recommend B. difficulty C. admirable D. document
5. A. encouragement B. interviewer C. acknowledge D. miraculously
PART TWO: LEXICO-GRAMMAR
I. Choose the best option to complete each of the following sentences
1. Only after the atomic bomb ________ and development in the air travel _______, ______ science
fiction really become popular.
A. had created/ had taken off/ was B. had been created/ had been taken off/ has
C. had been created/ had taken off/ did D. had been created/ / had taken off/ had
2. We’ve bought some ________ chairs for the garden so that they are easy to store away.
A. adapting B. adjusting C. bending D. folding
3. I don’t think she can get her message _______ to the students. She seems too nervous.
A. across B. around C. out D. over
4. _______, it is obvious that the whole thing was a waste of time and effort.
A. None of us wanted to go in the first place
B. Staff meetings are often boring and have no apparent point to them
C. Since the results were far more satisfactory than anyone had expected
D. Seeing that we couldn’t solve anything in the end
5. There are ______ words in English having more than one meaning. Pay close attention to this fact.
A. a large many B. quite many C. a great many D. quite a lot
6. This car has many features including _________.
A. stereo, safety devices, air condition, and it saves gas
B. good music, safety devices, air conditioning, and gas
C. stereo, safety devices, air conditioned, and good gas
D. stereo, safety devices, air conditioning, and low gas mileage
7. Round and round ___________.
A. the wheels of the engine went B. did the wheels of the engine go
C. went the wheels of the engine D. going the wheels of the engine
8. The replacement of shops such as the groceries and chemists’ by the café _______ the housewives
with insufficient facilities for shopping.
A. leave B. have left C. has left D. to have left
9. Your argument _______ that Britain is still a great power, but this is no longer the case.
A. outlines B. presupposes C. concerns D.
presents
10. They are happily married although, of course, they argue _______.
A. most times B. from day to day C. every now and then D. on the occasion
11. He promised to mend the broken wheel soon without ___________ .
A. fail B. failure C. trouble D. mistake
12. One of the first exercises in math class is ______ measure the radius of a circle.
A. to learn and B. to learn how to C. learning to D. learn to
13. We were shocked to hear the news of your ________.
A. having fired B. being fired C. having been fired D. to have been fired
14. I don’t know French, but I’ll ________.
A. get Tom to translate it B. have it translate
C. have Tom to translate it D. make it translate
Page 1
15. _______ as taste is really a composite sense made up of both taste and smell.
A. That we refer to B. What we refer to
C. To which we refer D. What do we refer to
16. _______ have settled, one of their first concerns has been to locate an adequate water supply.
A. Wherever people B. There are people who
C. Where people D. People
17. Politicians should never lose ______ of the needs of the people they represent.
A. view B. sight C. regard D. prospect
18. _______ team sports require cooperation.
A. Of all B. They are all C. Why all are D. All
19. Studies indicate _________ collecting art today than ever before.
A. there are that more people B. more people that are
C. that there are more people D. people there are more
20. Doctors advise people who are deficient __________ vitamin C to eat more fruit and vegetables.
A. from B. of C. in D. for
21. Dick put ____ ball in ____ net in _____ second half but ____ goal was disallowed.
A. a - a - a - a B. the - the - the - the
C. the - the - a - a D. a - a - the - the
22. There is a real possibility that these animals could be frightened _______ a sudden loud noise.
A. being there B. should there be C. there was D. there have been
23. The computer has dramatically affected ______ photographic lenses are constructed.
A. is the way B. that the way C. which way do D. the way
24. Six novels a year, you say? He’s certainly a __________ writer.
A. fruitful B. fertile C. virile D.
prolific
25. The handwriting is completely ______ . This note must have been written a long time ago.
A. inedible B. indelible C. illegible D. unfeasible
II. Give the correct form or tense of the verbs in brackets.
a. The children were frightened because the lights suddenly (1.go) ________ out and they (2.sit)
________ in the dark.
b. What tune (3. play) ________ when we (4. come) ________ in?
c. She was badly hurt when her car hit another car. If she (5. wear) ________ her seat belt, she (6. not
hurt) ________ so badly.
d. It is vital that no one else (7. know) _______ about the secret government operation.
e. It seems strange to be standing here, (8. look) _______ out at SydneyHarbor.
f. Tom had a lucky escape. He (9. kill) ________ when a car crashed into the front of his house.
g. _____ (10. Rank) as a masterpiece, a work of art must transcend the ideals of the period in which it
was created.
III. There are ten mistakes in the following passage. Find and correct them. (1.5pt)
It seems that the mystery of why the Pyramids were built may have solved. Until quite recently
people got used to think that they were just tombs for pharaohs. Instead, the connection with astronomy
seemsmuch more important. Egyptologists have often asked them how long it spent to build them and
why people built them in first place. Experts came up with a suggestion that the Egyptians may have
believed in the River Nile was the earthly equivalent of the Milky Way. Many agree that the sizes of the
three Giza Pyramids are in promotion to the three stars of Orion. Nothing, then, was by the chance.
Rather, the souls of dead pharaohs were deliberately beingproject through shafts toreach at their goal of
the Orion constellation.
IV. Give the correct form of the word in each of the following brackets.(1.5 pt)
It was not so long ago that we dealt with colleagues through face-to-face (1. INTERACT)
______ and with counterparts and customers by phone or letter. But the world of communication has
undergone a dramatic transformation, not for all the good. Email, while (2. DOUBT) _____ a swift
means of communication providing your server is fully (3.FUNCTION) ______ and that the address
you have contains no (4. ACCURATE) _____ has had a (5. SIGNIFY) _____ effect on certain people’s
behaviour, both at home and business. For those people, the use of email has become irresistibly (6.
ADDICT) _____ to the extent that it is (7. THREAT) _____ their mental and physical health. Addicts
spend their day (8. COMPULSION) _____ checking for email and have a (9. TEND) ______ to panic
Page 2
if their server goes down. It is estimated that one in six people spend four hours a day sending and
receiving messages, the equivalent to more than two working days a week. The negative effect on (10.
PRODUCE) ________ is something employers are well aware of.

PART THREE: READING (6.0 POINTS)


I. Read the following passage and choose the option that indicates the correct answer to each of the
following questions.(2.0 pts)
In the United States in the early 1800's, individual state governments had more effect on the
economy than did the federal government. States chartered manufacturing, banking, mining, and
transportation firms and participated in the construction of various internal improvements such as canals,
turnpikes, and railroads. The states encouraged internal improvements in two distinct ways; first, by
actually establishing state companies to build such improvements; second, by providing part of the
capital for mixed public-private companies setting out to make a profit.
In the early nineteenth century, state governments also engaged in a surprisinglylarge amount of
direct regulatory activity, including extensive licensing and inspectionprograms.Licensing targets
reflected both similarities and differences between the economy of the nineteenth century and that of
today: in the nineteenth century, state regulation through licensing fell especially on peddlers,
innkeepers, and retail merchants of various kinds. The perishable commodities of trade generally came
understate inspection, and such important frontier staples as lumber and gunpowder were also subject to
state control. Finally, state governments experimented with direct labor and business regulation designed
to help the individual laborer or consumer, including setting maximum limits on hours of work and
restrictions on price-fixing by businesses.
Although the states dominated economic activity during this period, the federal government was
not inactive. Its goals were the facilitation of western settlement andthe development of native
industries. Toward these ends, the federal government pursued several courses of action. It established a
national bank to stabilize banking activities in the country and, in part, to provide a supply of relatively
easy money to the frontier, where it was greatly needed for settlement. It permitted access to public
western lands on increasingly easy terms, culminating in the Homestead Act of 1862, by which title to
land could be claimed on the basis of residence alone. Finally, it set up a system of tariffs that was
basically protectionist in effect, although maneuvering for position by various regional interests
produced frequent changes in tariff rates throughout the nineteenth century.
1. What does the passage mainly discuss?
A. States's rights versus federal rights.
B. The participation of state governments in railroad, canal, and turnpike construction.
C. The roles of state and federal governments in the economy of the nineteenthcentury.
D. Regulatory activity by state governments.
2. The word “effect” in bold in paragraph 1 is closest in meaning to __________.
A. value B. argument C. influence D. restraint
3. All of the following are mentioned in the passage as areas that involved statevernments in the
nineteenth century EXCEPT _________.
A. mining B. banking C. manufacturing D. higher education
4. The word “distinct” in bold in paragraph 1 is closest in meaning to_________.
A. separate B. innovative C. alarming D. provocative
5. It can be inferred from the first paragraph that in the nineteenth century canals and railroads were
_________.
A. built with money that came from the federal government
B. much more expensive to build than they had been previously
C. built predominantly in the western part of the country
D. sometimes built in part by state companies
6. The regulatory activities of state governments included all of the following EXCEPT_______.
A. licensing of retail merchants B.inspecting materials used in turnpike maintenance
C. imposing limits on price-fixing D. control of lumber
7. The word “setting” in bold in paragraph 2 is closest in meaning to _________.
A.discussing B. analyzing C.establishing D.avoiding
8. The word “ends” in bold in paragraph 3 is closest in meaning to ________.
A.benefits B.decisions C.services D.goals
Page 3
9. According to the passage, which of the following is true of the Homestead Act of 1862?
A. It made it increasingly possible for settlers to obtain land in the West.
B. It was a law first passed by state governments in the West.
C. It increased the money supply in the West.
D. It established tariffs in a number of regions.
10. Which of the following activities was the responsibility of the federal government in the nineteenth
century?
A.Control of the manufacture of gunpowder.
B. Determining the conditions under which individuals worked.
C. Regulation of the supply of money.
D. Inspection of new homes built on western lands.

II. Read the text below and fill in each blank with ONE suitable word. (2.0 pts)
The British are widely (1) _____ to be a very polite nation, and in (2)______ respects this is true.
An Italian journalist once commented of the British that they need (3) _____ fewer than four “thank
you” merely to buy a bus ticket. The first, from the bus conductor means, “I’m here.” The second
accompanies the handing over of the money. The third, again from the conductor, (4) ____ “Here is your
ticket.”, and then the passenger utters a final one as he accepts the tickets. Such transactions in most (5)
____ parts of the world are usually conducted in total silence. In sharp contrast to this excessive
politeness with strangers, the British are strangely lacking (6) _____ ritual phrases for social interaction.
The exhortation “Good appetite”, uttered in so (7) ______ other languages to fellow-diners before a
meal, does not exist in English. The nearest equivalent – Enjoy your dinner! – is said only by people
who will not be partaking of the meal in question. What’s more, the British (8) ____ happiness to their
friends or acquaintances only at the start of a new year and at (9)_____ such as birthdays, (10) _____ the
Greeks routinely wish all and sundry a “good week” or a “good month”.
Your answers:
1._____________ 2._____________ 3._____________ 4._____________ 5.____________
6._____________ 7._____________ 8._____________ 9._____________ 10.____________
III. Read the passage and choose the best option for each of the following blanks. (2.0 pts)
Media and advertising
After more than fifty years of television, it might seem only obvious to conclude that it is here to
(1) ______. There have been many objections to it during this time, of course, and (2) ______ a variety
of grounds. Did it cause eye-strain? Was the (3) ______ bombarding us with radioactivity? Did the
advertisements contain subliminal messages, persuading us to buy more? Did children turn to violence
through watching it, either because so (4) ______ programmes taught them how to shoot, rob, and kill,
or because they had to do something to counteract the hours they had spent glued to the tiny screen? Or
did it simply create a vast passive (5) ______ drugged by glamorous serials and inane situation (6)
______ ? On the other hand did it increase anxiety by sensationalizing the news [or the news which was
(7) ______ by suitable pictures] and filling our living rooms with war, famine and political unrest? (8)
______ in all, television proved to be the all-purpose scapegoat for the second half of the century,
blamed for everything, but above all, eagerly watched. For no (9) ______ how much we despised it,
feared it, were bored by it, or felt that it took us away from the old paradise of family conversation and
hobbies such as collecting stamps, we never turned it off. We kept staring at the screen, aware that our
own tiny (10) ______ was in if we looked carefully.
1. A. be B. stay C. exist D. prolong
2. A. with B. over C. by D. on
3. A. screen B. danger C. machine D. reason
4. A. that B. far C. many D. what
5. A. programme B. personality C. audience D. tense
6. A. comedies B. programmes C. perhaps D. consequently
7. A. taken B. presented C. capable D. accompanied
8. A. Taken B. All C. Somewhat D. Thus
9. A. one B. matter C. difference D. reason
10. A. fault B. reflection C. situation D. consciousness
PART FOUR: WRITING (6.0 POINTS)

Page 4
I. Finish the second sentence in such a way that it means exactly the same as the sentence printed
before it. (2.0 pts)
1. If the work is finished by lunchtime, you can go home.
Get____________________________________________________________________________
2. You haven’t done your work, have you?
It’s about_______________________________________________________________________
3. The fourth time he asked her to marry him, she accepted.
Only on his _____________________________________________________________________
4. He said that he had won as a result of good luck.
He attributed____________________________________________________________________
5. That reminds me of the time I climbed to the top of Mount Fuji.
That takes me____________________________________________________________________
6. People rumour that he is rich but stingy.
What___________________________________________________________________________
7. Such a ridiculous proposal isn’t worth serious consideration.
There is_________________________________________________________________________
8. Just thinking about his face at that moment makes me laugh.
The very_________________________________________________________________________
9.We cannot see animals in a vast area after the forest fire.
There is an_______________________________________________________________________
10.The staff hated Frank’s new policies so intensely that they went on strike.
So intense _______________________________________________________________________

II. Write a new sentence similar in meaning to the given one, using the word given in the brackets. Do
not alter the word in any way. (2.0 pts)
1. I suddenly realized the meaning of a “freebie”.(dawned)
_______________________________________________________________________________
2. After the scandal, he was asked to resign. (HAND)
_______________________________________________________________________________
3. Bruce said that the situation at work was like a family argument. (likened)
_______________________________________________________________________________
4. My father is not feeling well these days. (weather)
_______________________________________________________________________________
5. I don’t think this record will ever be popular. (catch)
_______________________________________________________________________________
6. His arrival was completely unexpected. (took)
_______________________________________________________________________________
7. The success of our local theater has made our city famous. (map)
_______________________________________________________________________________
8. He is certainly not a reliable witness. (means)
_______________________________________________________________________________
9. Our company is the only company allowed to import these chemicals. (monopoly)
_______________________________________________________________________________
10. It’s uncertain whether the band’s tour will take place. (BALANCE)
_______________________________________________________________________________
III. Essay writing (2.0 pts)
Some high schools require all students to wear school uniforms. Other high schools permit students
to decide what to wear to school. Which of these two school policies do you think is better?
In about 250 words, write an essay to expess your opinion,usingspecific reasons and examples to
support your opinion.
ENGLISH PRACTICE 2
A. PHONETICS(10points)
Question 1: Pick out the word whose underlined part is pronounced differently from those of the other
words. Write your answers in the numbered blanks provided below (5p)
01. A.random B.canal C.many D.explanatory
02. A.good B.moon C.food D.balloon
Page 5
03. A.provide B.product C.production D.procedure
04. A.kite B.bite C.Christian D.Christ
05. A.voltage B.voyage C.massage D.dosage

Question 2: Choose the word whose stress pattern is differently from those of the other words. Write
your answers in the numbered blanks provided below(5p)
06. A.argumentative B.psychological C. contributory D.hypersensitive
07. A.atomic B.brilliant C. determined D.ambitious
08. A.trigonometry B.explanatory C. immediately D.democracy
09. A.photograph B.payroll C. accent D.regretful
10. A.majority B.ceremony C. astronomy D.investiture

B. GRAMMAR & VOCABULARY(60points)


Question 3: Give the correct tense/ form of the verbs in the brackets. Write your answers in the
numbered blanks provided below (30p)
a. It’s time we (11. go)................... If we..................... (12. not leave) now, we................... (13. miss) the
train.
b. I know I .......................(14. not write) to you before, but I ..................(15. be) so busy recently that
I...................... (16. not have) time for writing letters. I .....................(17. telephone) you instead, but
I ................................(18. forget) yournumber.
c. Why didn’t you tell me you could lend me the money? I .......................(19. not borrow) it from the
bank.
d. He is walking....................... (20. hurry) to the place where his ex-wife, a famous naturalist
now .........................(21. lie) ............................(22. bury) among the animals she ..............................(23.
love) somuch.
e. We will see Alice at the corner, she............................ (24. wait) for us when we....................(25.
arrive).
f. He resents.......................... (26. be)....................... (27. tell) what...............................(28. do).
g. Her mother.............................. (29. go) abroad last month, so it ..........................(30. not be) her you
see at the theater last Sunday.

Question 4: The passage below contains 10 errors. IDENTIFY and CORRECT them. (0) has been
done as an example. Write your answers in the numbered blanks below(20p)

Question 5: Fill in each blank with a suitable PREPOSITION or PARTICLE. Write your answers in
the numbered blanks provided below the passage.(10p)
41. We’re all very obliged_________ you
42. When the wound has healed_________ you can remove theplaster.
43. Without a fridge, fresh food will go_________ veryquickly.
44. I’m faithful_________ my principle.
45. I haven’t heard_________ Mandy since she wrote inJuly.
46. This service is free_________ charge.
47. They went ahead contrary_________ myadvice.
Page 6
48. The weather was fine, and everyone was making_________ the coast.
49.We’re going_________ Tom’s cartomorrow.
50. Have a card_________ your sleeve.

C. READING (40points)
Question 6: Read the following passage and choose the best answer. Write your answers in the
numbered blanks provided below the passage. (20p)
In addition to the great ridges and volcanic chains, the oceans conceal another form of undersea
mountains: the strange guyot, or flat-topped seamount. No marine geologist even suspected the
existence of these isolated mountains until they were discovered by geologist Harry H. Hess in 1946.
He was serving at the time as naval officer on a ship equipped with a fathometer. Hess named these
truncated peaks for the nineteenth-century Swiss-born geologist Arnold Guyot, who had served on the
faculty of Princeton University for thirty years. Since then, hundreds of guyots have been discovered in
every ocean but the Arctic. Like offshore canyons, guyots present a challenge to oceanographic theory.
They are believed to be extinct volcanoes.Their flat tops indicate that they once stood above or just
below the surface, where the action of waves leveled off their peaks. Yet today, by definition, their
summits are at least 600 feet below the surface, and some are as deep as 8,200 feet. Most lie between
3,200 feet and 6,500 feet. Their tops are not really flat but slope upward to a low pinnacle at the center.
Dredging from the tops of guyots has recovered basalt and coral rubble, and that would be expected
from the eroded tops of what were once islands. Some of this material is over 80 million years old.
Geologists think the drowning of the guyots involved two processes: The great weight of the volcanic
mountains depressed the sea floor beneath them, and the level of the sea rose a number of times,
especially when the last Ice Age ended, some 8,000 to 11.000 yearsago.
51. What is the author's main purpose in writing thispassage?
A. To trace the career of Arnold Guyot.
B. To describe feature of the underseaworld.
C. To present the results of recent geologicresearch.
D. To discuss underwater ridges and volcanochains
52. The word “conceal” is closest in meaning to which of thefollowing?
A. Contain B.Erode C.Hide D.Create
53. The passage implies that guyots were first detected by means of________.
A. afathometer B. computer analysic
C. a deep-seadivingexpedition D. researchsubmarines
54. The author indicates that ArnoldGuyot________.
A. was HarryHess'sinstructor B. invented thefathometer
C. named the guyedafterhimself D. taught at PrincetonUniversity
55. What does the passage say about the ArcticOcean?
A. The first guyot was discoveredthere.
B. No guyots have ever been foundthere.
C. There are more guyots there than in any otherocean.
D. It is impossible that guyots were ever formedthere.
56. The author states that offshore canyons and guyots have which of the following characteristics in
common?
A. Both are found on the ocean floor near continentalshelves.
B. Both present oceanographers with amystery.
C. Both were formed by volcanicactivity.
D. Both were, at one time, above the surface of thesea.
57. According to the passage, most guyots are found at a depth of________.
A. less than600feet. B. between 600 and 3,200feet.
C. between 3,200 and6,500feet D. more than 8,200feet
58. Which of the following is closest in meaning to the word“rubble”?
A. Fragments B.Mixture C.Columns D.Core
59. According to the passage, which of the following two processes were involved in the submersion of
guyots?
A. Erosion and volcanicactivity.
B. The sinking of the sea floor and the rising of sealevel.
Page 7
C. Mountain building and the action of oceancurrents
D. High tides and earthquakes
60. According to the passage, when did sea level significantlyrise?
A. In1946 B. In the nineteenthcentury
C. From 8,000 to11,000years D. 80 million yearsago

Question 7: Read the following passage and choose the best answer. Write your answers in the
numbered blanks provided below the passage.(20p)
As Christmas evolved in the United States, new customs were adopted and many old ones were
reworked. The legend of Santa Claus, for example, had origins in Europe and was brought by Dutch
settlers to New York in the early 18th century. Traditionally, Santa Claus - from the Dutch Sinter Klaas -
was depicted as a tall, dignified, religious figure riding a white horse through the air. Known as Saint
Nicholas in Germany, he was usually accompanied by Black Peter, an elf who punished disobedient
children. In North America he eventually developed into a fat, jolly old gentleman who had neither the
religious attributes of Saint Nicholas nor the strict disciplinarian character of BlackPeter.
Santa’s transformation began in 1823, when a New York newspaper published the poem A Visit
from Saint Nicholas, which Clement Clark Moore had written to amuse his daughter. The poem
introduced many Americans to the story of a kindly saint who flew over housetops in a reindeer-drawn
sleigh. Portraits and drawings of Santa Claus by American illustrator Thomas Nast further strengthened
the legend during the second half of the 19 th century. Living at the North Pole and assisted by elves, the
modern Santa produced and delivered toys to all good children. By the late 19 th century he had become
such a prominent figure of American folklore that in 1897, when Virginia O’Hanlon wrote to the New
YorkSunnewspaperaskingifSantawasreal,shereceivedadirectanswer:“Yes,Virginia,thereis aSanta Claus”.
61. Who brought the legend of Santa Claus to the USA according to thepassage?
A. SinterKlaas B. Saint Nicholas C. AGerman D. Dutchsettlers
62. Santa Claus was traditionally described as a________.
A. tall man who could walk through theair. B. fat, jolly, oldman.
C. religious figure. D. fat man riding a whitehorse.
63. Santa Claus in North America was depicted as________.
A. a man with the strict disciplinarian character of BlackPeter.
B. a good old man with less religiouscharacter.
C. one with religious attributes of SaintNicholas.
D. a jolly man onhorseback.
64. Who was BlackPeter?
A. an elf accompanying SaintNicholas. B. an elf who rode a whitehorse.
C. one of the disobedient children. D. a popular traditional figure.
65. What word is closest in meaning to“attributes”?
A. symbols of a person B. naturalqualities C. effects D. outerappearance
66. Where did the legend of Santa Claus comefrom?
A. theNorthPole B. Europe C. NorthAmerica D. the City of NewYork
67. 1823 was mentioned as a year when________.
A. Clement Clark Moore wrote his firstpoem
B. Clement Clark Moore’s poem made himpopular
C. Saint Nicholas visited NewYork
D. the image of Santa Claus wastransformed
68. According to Clement Clark Moore’spoem
A. Santa Claus had nothing different in appearance from the traditionalone.
B. Santa Claus had wings and couldfly.
C. Santa Claus liked poetry.
D. Santa Claus was a kindly saint who flew over housetops in asleigh.
69. The answer “Yes, Virginia, there is a Santa Claus” is an illustration for the factthat________.
A. the New York Sun was popular withchildren.
B. Santa Claus was a prominent figure at thattime.
C. newspapers are unreliable.
D. Virginia O’Hanlon was a reader of the New YorkSun
70. Which of the following statements isTRUE?
Page 8
A.Santa Claus was an imaginary old man created by artists based on traditional figures.
B. Living in the North Pole, Santa Claus visited children atChristmas.
C. Santa Claus was a real figure living in northernAmerica.
D. Santa Claus was a story based on Saint Nicholas and BlackPeter.

D. USE OF ENGLISH(30points)
Question 8: Choose the word that best fits each of the blanks in the following passage. Write
youranswers in the numbered blanks provided below the passage. (0) has been done as an example.
(10p)
Vietnamese generally shake hands when greeting and parting. Using both hands shows respect as
does a (71)________ bow of the head. In rural areas, elderly people do not extend their hand are greeted
with a bow. Women are more (72)________ to bow the head than to shake hands. Vietnamese names
begin with the family name and are (73)________ by a given name. For example, in the name Nguyen
Van Due, Nguyen is the family name. People address (74)________ by their given names, but add a
title that indicates their perceived(75)________ to the other person. These titles are family related
rather than professional. Among colleagues, for example, the (76)________ of the two might combine
the given name with the title of Anh ("Older Brother"). A(n) (77)________ greeting combined with the
given name and title is Xin chao ("Hello."). Classifiers for gender and familiarity are also combined with
the greeting. In formal meetings, business cards are sometimes (78)________ ongreeting.
Vietnamese people have a strong (79)________ of hospitality and feel embarrassed if they
cannot show their guests full respect by preparing for their arrival. Therefore, it (80)________ to visit
someone without having been invited. Gifts are not required, but are appreciated. Flowers, incense, or
tea may be proper gifts for the hosts. Hosts also appreciate a small gift for their children or
elderlyparents
71. A. light B. slight C. lightly D. lighted
72. A. possible B. capable C. probable D. likely
73. A. continued B. chased C. followed D. taken
74. A. each one B. one other C. one the other D. one another
75. A. relation B. relationship C. relation D. relatives
76. A. younger B. most young C. youngest D. young
77. A. easy B. basic C. fundamentally D. elementary
78. A. changed B. transferred C. reserved D. exchanged
79. A. sense B. sensation C. sensitivity D. sensible
80. A. inactive B. inaccurate C. inappropriate D. inexact

Question 9: Fill each blank with ONE suitable word. Write your answers in the numbered blanks
provided below the passage. (0) has been done as an example. (10p)
Cell Phones
Cell phones have been popular in Japan (0. since) the early 1990s, but it was (81)________ until
1999 that their use really took off. The age of cell phones has emerged, but with it comeproblems.
Cell phones are used on buses and trains, in restaurants, and in all areas of (82)________. They
cause problems when they (82)________ during meetings, concerts, weddings, or even funerals.
What's more, peoplespeakloudlyinpublic,andstudentsreadandtextmessagesduringlessons.(83)________
seriously, when a cell phone is used near a person (84)________ a pacemaker to fegulate his heartbeat,
its radio waves may interfere with the functioning of thepacemaker.
Now, something is being done to solve these(86)________. In many places, new technology is
being used to block cell phone calls. Airline(87)________ are requested to stop using cell phones while
on board.Concerthallsasktheiraudiencetoswitchtheirphonestothe(88)________ mode. However, phone
users fear that if they do not(89)________ their phones, they will lose valuable business opportunities.
That's why many do not (90)________ off their phones even when they are askedto.

Question 10: Write the correct FORM of each bracketed word. Write your answers in the numbered
spaces provided below. (10p)
91. Those (theatre) group has never lost itsappeal.
92. Dolphins, (mammal)species, sometimes jump above the surface of thewater.
93. The situation is so (chaos)in some countries now that it is difficult to see anysolution
Page 9
94. He’s the most (please), ill-mannered person I’ve evermet.
95. Good friends can enrich the quality of our lives(measure).
96. This famous singer had two (enter) in the Guinness Book of WorldRecords.
97. Beauty is in the eye of the(hold).
98. Computers are now considered (dispense) in the businessworld.
99. Due to (electric)the difference between urban life and rural life is more and morereduced.
100. A doctor may prescribe (biotic) if the patient has aninfection.

E. WRITING (30p)
Question 11:Finish each of the following sentences in such a way that it means the same as the
sentence printed before it(10p).
101. Someone stole the old lady’shandbag.
→ The old lady was .........................................................................................
102. A true story forms the basis of Mary’s newnovel.
→ Mary’s new novel .........................................................................................
103. If you have completed your test, you can gohome.
→ Get ......................................................................................................................
104. Absolute secrecy was crucial to the success of themission.
→ Without ............................................................................................................
105. Something must be done to solve thisproblem.
→ Urgent .......................................................................................... ....................

Question 12:Finish each of the following sentences in such a way that it is as similar its possible in
meaning to the original sentence. Use the word given and other words as necessary. Do not change
the form of the given word(10p).
106. The job received over a hundred applications. (APPLIED)
→ .......................................................................................................................
107. Alan prides himself on his punctuality. (TAKES)
→ .......................................................................................................................
108. Many people attended this year’s festival. (TURNOUT)
→ .......................................................................................................................
109. He is becoming quite famous as an interviewer. (NAME)
→ .......................................................................................................................
110. The whole team was in a happy mood. (SPIRITS)
→ .......................................................................................................................

Question 13: Writing anessay (40p)


It is widely believed nowadays that social knowledge and the ability to adapt in a changing society make
a school-leaver succeed in getting a job.
What do you think about the saying?
Write an essay of around 250 words to prove your points.

ENGLISH PRACTICE 3

I. LEXICO-GRAMMAR (7/20 points)


Part 1: Choose the word or phrase that best completes each sentence. Write your answer (A, B, C, or
D) in the numbered box.
1. Sheila will inherit everything ________ her uncle’s death.
A. on account of B. in spite of C. in the event of D. in place of
2. His poor handling of the business ________ on negligence.
A. neared B. edged C. approached D. bordered
3. Down ________ for three days.
Page 10
A. the rain poured B. poured the rain
C. did the rain pour D. do the rain poor
4. Can I ________ your brains for a moment? I can’t do this crossword by myself.
A. have B. pick C. mind D. use
5. The job wasn’t giving the ________ of the experience he wanted.
A. width B. depth C. length D. breadth
6. I suppose I could ________ advertising.
A. catch on B. get out of C. go in for D. work out
7. The storm ripped our tent to ________.
A. slices B. shreds C. strips D. specks
8. She travelled the world in ________ of her dreams.
A. pursuit B. finding C. chase D. trail
9. The agency is ________ and not run for profit.
A. charitable B. donated C. voluntary D. free
10. Mike, _______, will you switch off that television!
A. once and for all B. now and then
C. over and above D. from time to time
Your answers
1. 2. 3. 4. 5.
6. 7. 8. 9. 10.
Part 2: Write the correct FORM of each bracketed word in the numbered space provided in the
column on the right.
11. The (PLACE)_______ of natural resources is becoming inadequate 11. ________
for the support of increasing population.
12. The grammar section provides coverage of all the most
(PROBLEM)_______ areas. 12. ________
13. He has recorded the (NARRATE)______ for the production. 13. ________
14. In this profession, women (NUMBER)______ men by two to one. 14. ________
15. An (LAY)_______ showing population can be placed on top of the 15. ________
map.
16. The shells are so hard and they are virtually (DESTROY)_______. 16. ________
17. We should arrive two days early in order to (CLIMATE)________. 17. ________
18. These changes are likely to (POOR)________ single-parent families 18. ________
even further.
19. I’m afraid the hospital is desperately (STAFF)______ at the moment. 19. ________
20. This book will challenge your (CONCEPT)_______ about rural life. 20. ________
Part 3: The passage below contains 10 mistakes. Underline the mistakes and correct them in the space
provided in the column on the right. (0) has been done as an example.
Human and primates, the family of apes, gorillas, and chimpanzees, among others, 0. Humans
divide many common traits. 21. ____________
While primates are deemed the most intelligent of animals, most researchers believed
they lack the capacity to produce language. However, a research project in the 1970s 22. ____________
at University of Georgia showed promise that chimpanzees have the ability to learn a 23. ____________
Page 11
certain language, just as human children do.
The project used several chimpanzees as test subjects in which Lana, a female chimp 24. ____________
was the study focus.
Though the primates lack the vocal constructions to make human speech patterns, the 25. ____________
researchers created a language called Yerkish, using lexigram made up of symbols 26. ____________
that represent sounds and words. 125 symbols were placed on a keyboard, which 27. ____________
Lana was taught how to use the board to communicate with the researchers. She 28. ____________
successfully expressed her thoughts by pressing different keys in succession. In some 29. ____________
cases, she used up to seven at times. 30. ____________
Part 4: Supply the correct form of the VERB in brackets to complete the passage. Write your answer
in the numbered box.
I don't normally go to the cinema. Not because I don't like it but because it's just a habit I
(31.never GET)__________ into. However, on this occasion I (32. DECIDE)__________ to go because
my friends (33. constantly GO)__________ on about this film all week and eventually wore me down. It
was starring some ephemeral Hollywood actor whom I had vaguely heard of but couldn't put a face to.
We got to the cinema early to find people (34. already WAIT)__________ outside, which suggested
that my friends weren't the only ones who thought it was worth seeing although I could still think of
several other things I would rather having been doing at that moment. In the end, the film (35.
TURN)_________ out to be not half as bad as expected, though I (36. PREFER)__________ something
with a bit more action. The plot centred on two men who were planning to carry out some immensely
complicated robbery, though what they (37. completely FAIL) ___________ to realize was that all the
time their plans (38. closely MONITOR) ____________ by the police. Somewhat unpredictably,
however, they got away with it because they (39. CHANGE) _____________ their plans at the last
minute. It was okay but I (40.not THINK)________ of going again.
Your answers
31. 36.
32. 37.
33. 38.
34. 39.
35. 40.

Part 5: Fill each gap in the following sentences with one of the prepositions or particles in the box.
Use each word only ONCE and write your answer in the numbered box. (Please note that the
given words outnumber the gaps.)
by through with For in out
up into off Across down beyond
41. He likes to show _________ how well he speaks French.
42. They were sad because the plan fell _________ at the last minutes.
43. Ben is a true adventurer. He has climbed this country's highest mountain, canoed _________ the
continent, and hiked through the Amazon jungle.
44. She wrote the book _________ collaboration with one of her students.
45. You'd better write _________ the appointment in your agenda, or you'll probably forget.
Page 12
46. Mary has always looked _________ to her uncle, who is a very successful actor.
47. I do feel ________ you, honestly!
48. As I had put on weight, my dress was too tight so I had to let it ________ especially around the
waist.
49. I'm sorry but Dr. Ho sees patients _________ appointments only.
50. He spoke _________ such assurance that we couldn't but believe him.
Your answers
41. 46.
42. 47.
43. 48.
44. 49.
45. 50.
II. READING (6/ 20 points)
Part 1: Read the following passage and decide which answer (A, B, C, or D) best fits each gap. Write
your answer in the numbered box. (0) has been done as an example.
When faced (0)________ some new and possible bewildering technology change, most people
(51)________ in one of two ways. They either recoil (52)________ anything new, claiming that it is
unnecessary, or too complicated or that it (53)________ makes life less than human. Or they learn to
adapt to the new invention and (54)________ wonder how they could possibly have existed
(55)________ it. Take computers as example. For many of us, they still (56)________ a threat to our
freedom and give us a frightening (57)________ of a future in which all decisions will be (58)________
by machines. This may be because they seem (59)________, and difficult to understand. Ask most
people what you can use a home computer for, and you usually get vague answers about how ‘they give
you information’. In fact, even those of us who are (60)________ with computer and use them in our
daily work, have little idea of how they work. But it does not take long to learn how to operate a
business programme, even if things occasionally go wrong for no apparent (61)________. Presumably,
much the same happened when telephone and television became widespread. What seems to alarm most
people is the (62)________ of technology change, (63)________ than change itself. And the objections
that are made to new technology may (64)________ have a point to them, since change is not always an
improvement. As we discover during power cuts, there is a lot to be said for the oil lamp, the coal fire,
and forms of entertainment, such as books or board (65)________, which don’t have to be plugged into
work.

0. A. with B. to C. for D. on
51. A. react B. treat C. solve D. perform
52. A. of B. out of C. away from D. from
53. A. somewhere B. someplace C. someway D. somewhat
54. A. eventually B. possibly C. initially D. naturally
55. A. with B. without C. on D. for
56. A. show B. meet C. face D. represent
57. A. possibility B. sense C. idea D. prospect
58. A. invented B. changed C. taken D. done
Page 13
59. A. unsteady B. unsure C. mysterious D. obvious
60. A. accustomed B. familiar C. used D. commonplace
61. A. reason B. cue C. excuse D. cause
62. A. rate B. swiftness C. speed D. tempo
63. A. more B. less C. rather D. other
64. A. badly B. better C. worse D. well
65. A. sports B. games C. plays D. shows
Your answers
0. A
51. 52. 53. 54. 55.
56. 57. 58. 59. 60.
61. 62. 63. 64. 65.

Part 2: Read the following passage and answer the questions from 66 to 75 that follow.

THE HISTORY OF THE GUITAR


The word 'guitar' was brought into English as an adaptation of the Spanish word 'guitarra’, which
was, in turn, derived from the Greek 'kithara'. Tracing the roots of the word further back into linguistic
history, it seems to have been a combination of the Indo-European stem 'guit-’, meaning music, and the
root '-tar’, meaning chord or string. The root '-tar' is actually common to a number of languages, and can
also be found in the word 'sitar’, also a stringed musical instrument. Although the spelling and
pronunciation differ between languages, these key elements have been present in most words for 'guitar'
throughout history.
While the guitar may have gained most of its popularity as a musical instrument during the
modern era, guitar-like instruments have been in existence in numerous cultures throughout the world
for more than 5.000 years. The earliest instruments that the modern eye and ear would recognise as a
'normal' acoustic guitar date from about 500 years ago. Prior to this time, stringed instruments were in
use throughout the world, but these early instruments are known primarily from visual depictions, not
from the continued existence of music written for them. The majority of these depictions show simple
stringed instruments, often lacking some of the parts that define a modern guitar. A number of these
instruments have more in common with the lute than the guitar.
There is some uncertainty about the exact date of the earliest six-string guitar. The oldest one still
in existence, which was made by Gaetano Vinaccia, is dated 1779. However, the authenticity of six-
string guitars alleged to have been made prior to 1790 is often suspect, as many fakes have been
discovered dating to this era. The early nineteenth century is generally accepted as the time period
during which six-string guitars began taking on their modern shape and dimensions. Thus for nearly two
hundred years, luthiers, or guitar makers, have been producing versions of the modern acoustic guitar.
The first electric guitar was not developed until the early twentieth century. George Beauchamp
received the first patent for an electric guitar in 1936, and Beauchamp went on to co-found
Rickenbacker, originally known as the Electro String Instrument Company. Although Rickenbacker
began producing electric guitars in the late 1930s, this brand received most of its fame in the 1960s,
when John Lennon used a Rickenbacker guitar for the Beatles' debut performance on the Ed Sullivan
show in 1964. George Harrison later bought a Rickenbacker guitar of his own, and the company later
Page 14
gave him one of their earliest 12-string electric guitars. Paul McCartney also used a Rickenbacker bass
guitar for recording. The Beatles continued to use Rickenbacker guitars throughout their career, and
made the instruments highly popular among other musicians of the era.
The Fender Musical Instruments Company and the Gibson Guitar Corporation were two other
early electric guitar pioneers, both developing models in the early 1950s. Fender began with the
Telecaster in 1950 and 1951, and the Fender Stratocaster debuted in 1954. Gibson began selling the
Gibson Les Paul, based partially on assistance from jazz musician and guitar innovator Les Paul, in
1952. The majority of present day solid-body electric guitars are still based largely on these three early
electric guitar designs.
Throughout the history of the guitar, an enormous number of individuals have made their mark
on the way in which the instrument was built, played and perceived. Though some of these individuals
are particularly well known, like the Beatles or Les Paul, the majority of these people are virtually
invisible to most modern guitar fans. By looking at the entire history of the guitar, rather than just recent
developments, largely confined to electric guitars, it is possible to see more of the contributions of
earlier generations.

Questions 66-70
Complete the sentences. Choose NO MORE THAN THREE WORDS from the passage for each
answer. Write your answer in the numbered box.
66. Despite differences in _________, 'guit-' and '-tar' appear in the word for 'guitar' in many languages.
67. Instruments that we would call acoustic guitars have been made and played for approximately
_________.
68. The ________ of acoustic guitars have not changed much in 200 years.
69. Les Paul, the well-known ________ guitarist, was involved in the development of the electric guitar.
70. Most ________ of the guitar know little about its rich history.
Your answers
66. 67.
68. 69.
70.

Questions 71-75
Complete the summary. Choose NO MORE THAN TWO WORDS from the passage for each
answer. Write your answer in the numbered box.
Instruments similar to the guitar have been played by musicians for over (71) ________ years. What we
know about many of these instruments comes from (72) ________ rather than actual physical examples
or music played on them. In some ways, these early stringed instruments were closer to (73) ________
than the guitar as we know it today. We do have examples of six-string guitars that are 200 years old.
However, the (74) ________ of six-string guitars made by guitar makers who are also known as luthiers
before the final decade of the eighteenth century is often open to question.
Although the electric guitar was invented in the 1930s, it took several decades for electric guitars to
develop, with the company Rickenbacker playing a major part in this development. Most (75) ________

Page 15
electric guitars in use today are similar in design to guitars produced by the Fender Musical Instruments
Company and the Gibson Guitar Corporation in the 1950s.

Your answers
71. 72.
73. 74.
75.
Part 3: Some parts of the following passages have been removed. Read the passage and then choose
from the list A-J given below the best phrase to fill each of the spaces. Some of the suggested answers
do not fit at all. Write your answer in the numbered box. (0) has been done as an example.
PEDAL POWER
If you are trying to get around central London when the traffic is bad and you can’t find a taxi,
(0)____________: pedal-powered rickshaws are becoming increasingly popular in the capital. In fact, it
is estimated that 350 of them now cruise the streets, though there is no official licensing procedure for
these diminutive vehicles, so precise numbers are not available. And this is exactly the problem; it is
claimed by residents’ associations and taxi drivers. Since rickshaws are not subject to the same
regulations as other vehicles, (76) ____________. Consequently, the areas around popular tourist sights
are sometimes packed with stationary pedicabs, as the rickshaws are also known. And (77)
____________ as no specific law is being broken.
On the other hand, (78) ____________, which naturally get stuck in traffic jams like other motor
vehicle. And unlike traditional London cabs, pedal-powered rickshaws cause no pollution whatsoever.
Many tourists like them because they are an amusing, unusual way to get around. But are they
potentially dangerous? Well, (79) ____________. Some taxi drivers claim the three-wheeled vehicles
are inherently unstable and may tip over if the driver takes a corner too quickly. Rickshaw drivers insist
that they drive almost exclusively in the narrow streets of the city centre, where (80) ____________.
There has as yet been no serious accident involving a pedicab, but the government is considering some
form of regulation just to be on the safe side.

A. it could hardly be any different in this context


B. it is difficult for traffic police to do anything about them
C. it never seems to cause too much inconvenience
D. it very much depends who you talk to
E. it annoys pedestrians as much as motorists
F. it is clear that there is a need for an alternative to taxis
G. it is impossible to gain enough speed for this to be a problem
H. it is not clear whether they are subject to parking restrictions
I. it is tempting to try an alternative

Your answers
0. I
76. 77. 78. 79. 80.

Page 16
Part 4: Fill each blank with ONE suitable word. Write your answer in the numbered box provided
below the passage. (0) has been done as an example.

Although the (0)________ in the global temperature by 4 per cent predicted by many scientists
may not sound like much, it is the difference between (81)________ and the last Ice Age, when huge
glaciers (82)________ Europe and most of Britain. Nobody knows exactly what would happen in a
warmer world, (83)________ we do know some things. Heat a kettle and the water inside it expands.
The temperature of the world has climbed more than half a degree this century, and the oceans have
risen by at (84)________ 10 cm.
But just as it takes several minutes for a kettle to begin (85)________, so it may have taken the
oceans thirty years to swell. This means that the global warming we are now (86)________ is a result
only of the carbon dioxide we have dumped into the atmosphere up to (87)________ 1960s. Since then,
the use of fossil (88)________ has increased rapidly. Scientists working for the United Nations and
European governments have (89)________ warning that what the Dutch and the people of the East
Anglia will need to do will be to build more extensive sea defences. Many of the world’s greater cities
are at (90)________, because they are located at sea level. Miami, (91)________ entirely built on a
sandbank, could be swept away. But the effects of (92)________ sea levels will be much worse for the
developing countries. With a metre rise in sea levels, 200 million people could become (93)________.
There are other fears too, (94)________ to a recent United Nations report. The plight of the
hungry in the northern Africa could (95)________, as rainfall in the Sahara and beyond is reduced by 20
per cent.
Your answers
0. rise
81. 82. 83. 84. 85.

86. 87. 88. 89. 90.

91. 92. 93. 94. 95.

III. WRITING (7/ 20 points)


Part 1:
Finish each of the following sentences in such a way that it means the same as the sentence printed
before it.
96. Half the afternoon was spent on clay-modelling.
Clay-modelling ………………………….........................................................................
97. I heard her use those words many times.
Many’s …………………………………………………………………………………………
98. It was more an argument than a discussion.
It was not so ..................................................................................................................
99. They only reimbursed us because we took legal advice.
We wouldn't .................................................................................................................
100. Your silly question distracted me.
Page 17
You drove .................................................................................................................

Part 2:
Use the word given in brackets and make any necessary additions to write a new sentence in such a
way that it is as similar as possible in meaning to the original sentence. Do NOT change the form of
the given word.
101. From the educational point of view his childhood years had been well spent (TERMS)
………………………………………………………………………….………...........................
102. He's very good at tennis and he's also a very good footballer. (ADDITION)
………………………………………………………………………….………...........................
103. Why does everything seem to be difficult to me? (ONLY)
………………………………………………………………………….………...........................
104. If you work without a break, you are more likely to make an error. (PRONE)
………………………………………………………………………….………...........................
105. They were very keen to hear the president’s speech. (EARS)
………………………………………………………………………….………...........................

Part 3:
The chart below shows the results of a survey on various home activities among young people aged
11 to 16 in four countries. Write a report describing the information in the chart.

Home activities among young people

........................................................................................................................................................................

........................................................................................................................................................................

........................................................................................................................................................................

........................................................................................................................................................................

........................................................................................................................................................................

........................................................................................................................................................................

Page 18
........................................................................................................................................................................

........................................................................................................................................................................

........................................................................................................................................................................

........................................................................................................................................................................

..............................

Part 4:
A large number of people think that nowadays women should not work hard to earn their living;
they should spend more time in caring for their families.
To what extend do you agree or disagree with this opinion?
Write about 300 words to support your position (and do not include your personal information).
........................................................................................................................................................................

........................................................................................................................................................................

........................................................................................................................................................................

........................................................................................................................................................................

........................................................................................................................................................................

........................................................................................................................................................................

........................................................................................................................................................................

........................................................................................................................................................................

........................................................................................................................................................................

........................................................................................................................................................................

........................................................................................................................................................................

........................................................................................................................................................................

........................................................................................................................................................................

........................................................................................................................................................................

........................................................................................................................................................................

........................................................................................................................................................................

..................................................................................................................................................................

THE END

ENGLISH PRACTICE 5
Page 19
SECTION A: PRONUNCIATION
Choose the word whose stress is pronounced differently from the other three in the following
questions
1. A. disappearance B. Sympathetic C. capacity D. conversation
2. A. argument B. achievement C. Involvement D. confinement
3. A. completion B. behaviour C. material D. understand
4. A. opinion B. comfortable C. powerful D. accurate
5. A. penalty B. habitat C. decision D. Countryman
6. A. Atmosphere B. emission C. disposal D. volcanic
7. A. economic B. considerate C. territorial D. continental
8. A. commit B. recipe C. index D. preview
9. A. economic B. considerate C. territorial D. continental
10. A. commit B. recipe C. index D. preview
SECTION B: READING
Part 1: Read the following passage and choose the correct answer to each of the questions.
Scientists have established that influenza viruses taken from man can cause diseases in animals.
In addition, man can catch the diseases from animals. In fact, a greater number of wild birds seem to
carry the viruses without showing any evidence of illness. Some scientists conclude that a large family
of influenza viruses may have evolved in the bird kingdom, a group that has been on earth 100 million
years and is able to carry the viruses without contracting the disease. There is even convincing evidence
to show that virus strain are transmitted from place to place and from continent to continent by
migrating birds.
It is known that two influenza viruses can recombine when both are present in an animal at the
same time. The result of such recombination is a great variety of strains containing different H and N
spikes. This raises the possibility that a human influenza viruses can recombine with an influenza virus
from a lower animal to produce an entirely new spike. Research is underway to determine if that is the
way major new strains come into being. Another possibility is that two animal influenza strains may
recombine in a pig, for example, to produce a new strain which is transmitted to man.
11. According to the passage, scientists have discovered that influenza viruses _______
A. cause ill health in wild animals
B. do not always cause symptoms in birds
C. are rarely present in wild birds
D. change when transmitted from animals to man
12. What is known about the influenza viruses?
A. It was first found in a group of very old birds.
B. All the different strains can be found in wild birds.
C. It existed over 100 million years ago.
D. It can survive in many different places.
13. According to the passage, a great variety of influenza strains can appear when______
A. H and N spikes are produced
B. animal and bird viruses are combined
C. dissimilar types of viruses recombine
D. two viruses of the same type are contracted
14. New strains of viruses are transmitted to man by_______
A. a type of wild pig B. diseased lower animals
C. a group of migrating birds D. a variety of means
15. It can be inferred from the passage that all of the following are ways of producing new
strains of influenza EXCEPT_______
A. two influenza viruses in the same animal recombining
B. animal viruses recombining with human viruses
C. two animal viruses recombining
D. two animal viruses recombining in a human
Your answers:
11. 12. 13. 14. 15.

Page 20
Part 2: Read the text and then select the correct answer, A, B, C or D. Write your answer in the right
box below.
The Netherlands
Welcome to the Netherlands, a tiny country that only extends, at its broadest, 312 km north to
south, and 264 km east to west - (16) ______ the land area increases slightly each year as a (17)  ______
of continuous land reclamation and drainage. With a lot of heart and much to offer,
'Holland,' (18)  ______ it is commonly known to most of us abroad - a name stemming (19)  ______ its
once most prominent provinces - has more going on per kilometre (20)  ______ most countries, and
more English-speaking natives. You'll be impressed by its (21)  ______ cities and charmed by its
countryside and villages, full of contrasts. From the exciting variety (22)  ______ offer, you could
choose a romantic canal boat tour in Amsterdam, a Royal Tour by coach in The Hague, or a hydrofoil
tour around the biggest harbour in the world - Rotterdam. In season you could visit the dazzling bulb
fields, enjoy a full day on a boat, or take a bike tour through the pancake-flat countryside spiced with
windmills. The possibilities are (23)  ______ and the nationwide tourist office, which is on hand to give
you information and (24)  ______ reservations. You'll have (25)  ______ language problems here,
as the Dutch are true linguists and English is spoken here almost universally.
16. A. so B. despite C. in spite of D. although
17. A. whole B. consequently C. rule D. result
18. A. as B. like C. so D. that
19. A. in B. from C. on D. of
20. A. then B. than C. like D. to
21. A. historic B. historical C. historically D. historian
22. A. at B. in C. on D. for
23. A. countdown B. counted C. countless D. countable
24. A. sit B. catch C. do D. make
25. A. few B. a few C. little D. a little

Your answers:
16. 17. 18. 19. 20.
21. 22. 23. 24. 25.

SECTION B: VOCABULARY AND GRAMMAR


Part 1: Choose the best answer to complete each of the following sentences. Write A, B, C or D in
your answers
26. Please and see us some time - you’re always welcome.
A. come to B. come about C. come round D. come away
27. This is the most difficult job I’ve had to do.
A. by far B. by chance C. by all means D. by myself
28. You’d better stop spending money, you’ll end up in debts.
A. or so B. if not C. otherwise D. so that
29. One person……..seven in the world speaks perfect English.
A. from B. within C. for among D. out of.
30. The agency is ________ and not run for profit.
A. charitable B. donated C. voluntary D. free
31. He retired early ________ his ill health.
A. on behalf of B. in front of C. on account of D. in spite of
32. Job specialisation takes place…………of production is seperated into occupations.
A. whenever the work is B. when the work
C. is when the work D. whenever working
33. The judge decided to……….the thief to three months’imprisonment.
A. sentence. B. prison. C. put. D. imprison.
34. I met Jim at college. It was the start of a………….friendship.
A. longlife B. prolong C. lengthened D. lifelong
35. Bats avoid………..objects by emitting high-frequency sounds and listening for echoes.
A. to run into B. running onto C. running into D. to run onto
Page 21
Your answers :
26. 27. 28. 29. 30.
31. 32. 33. 34. 35.

Part 2: There are 8 mistakes in the following passage. Underline the mistakes and write the correct
answers in the space provided.
Let us suppose that you are in the position of a parent. Would you allow your children read any
book they wanted to without first checking its content? Would you take them to see any film without
first finding whether it is suitable for them? If your answer of these questions is yes, then you are either
extremely permissive, or just plain irresponsible. If your answer is not, then you are exercising your
right as a parent to protect your children from what you consider to be desirable influences. In other
words, by acting as a censor yourself, you are admitting that there is a strong case for censorship.
Children need protection and it is the parents' responsible to provide it. But what about adults?
Aren't they old enough to decide that is good for them? The answer is that many adults are, but don't
make the mistake of thinking that all adults are as yourself. Censorship is for the good of society as a
whole. Highly civilized people might find it possible to live together without laws of any kind: they
would just rely on good sense to solve their problems. But imagine what chaos there would be if we live
in a society without laws! Like the law, censorship contributes to the common good.
Your answers:
MISTAKES CORRECTION MISTAKES CORRECTION
36. 40.
37. 41.
38. 42.
39. 43.

Part 3: Fill in each blank with a suitable word to fill in the blanks.
Vitamins are substances required for the proper functioning of the body. In this century, thirteen
vitamins have been discovered .
A lack of any vitamins in a person’s body can cause illness. In some cases, an excess of vitamins can
also (44)……….to illness. For example, sailors in the past were prone to suffer from scurvy (45)………
is a disease resulting from the lack of vitamin C. It causes bleeding of the gum, loss of teeth and skin
rashes. Sailors suffer from scurvy because they did not eat fruits and vegetables. Fruits and vegetables
contain vitamin C ,which is necessary for good (46)…………..
Vitamin B complex is composed of eight different vitamins. A lack of any of these vitamins will
lead to different(47)……….. For instance, a person who has too little vitamin B1 will suffer from beri-
beri, a disease that causes heart problems and mental disorders. A lack of vitamin B2 results in eye and
skin problems while deficiency of vitamin B6 causes problems of the nervous system. Too little vitamin
B12 will cause anemia. The knowledge that vitamin deficiencies caused certain diseases led doctors to
(48)………people suffering from these illnesses by giving them doses of the necessary (49)
………….Today, vitamins are available (50)……….the form of pills and can easily be bought at any
pharmacy.
Your answers:
44. 45. 46. 47.
48. 49. 50.

Part 4: Complete the second sentence so that it has a similar meaning to first one. Use and don’t
change the word given. Write 2-5 words in total.
51. I tried to talk to Jack about the problem, but he was too busy. WORD
I tried to …………………………… about the problem ,but he was too busy
52. “ I don’t mind which film we see”, I said MATTER
I said that …………………………… me which film we saw.
53. I was too scared to tell him what I really thought. COURAGE
I ……………………………………… to tell him what I really thought.
54. People say that the pyramids are worth visiting. SUPPOSED
The pyramids ……………………………………………. worth visiting.
Page 22
55. I was so shocked that I couldn’t react. WASN’T
I …………………………………………………………… react.
SECTION C: WRITING
Part 1: Part 1: You have recently seen a film you enjoyed very much. Write a letter to an English-
speaking penfriend, recommending the film and explaining why you thought it was so good (80-90
words). Use your name and address as Tom Smith – 123, Red Street, Fantasy City.

Part 2: Write a composition about 150-200 words on the following topic:


Some students prefer to study alone. Others prefer to study with a group of students. Which do you
prefer? Use specific reasons and examples to support your answer.

THE END

ENGLISH PRACTICE 6
SECTION A: PHONETICS

I. Pick out the word whose underlined part is pronounced differently from that of the other words.
Write your answer in the space provided.
1. A. associate B. sociable C. ancient D. ancestor
2. A. walked B. threatened C. passed D. forced
3. A. too B. food C. soon D. good
4. A. legal B. legend C. generous D. manager
5. A. adventure B. future C. mature D. figure
1. 3. 5.
2. 4.

II. Pick out the word that differs from the other words in the position of the main stress. Write your
answer in the space provided.
6. A. comedy B. collection C. comical D. calculate
7. A. ambitious B. memorial C. memory D. mechanic
8. A. remove B. cancel C. copy D. answer
9. A. Japanese B. engineer C. practical D. questionnaire
10. A. document B. develop C. opponent D. astonish
6. 8. 10.
7. 9.

SECTION B: VOCABULARY AND GRAMMAR


I. Choose the word or phrase which best completes each sentence. Write your answer in the space
provided.
1. He has declared......building a new bridge.
A. against B. to C. in D. about
2. A washing machine of this type will certainly......normal domestic use.
A. stand up for B. get on to C. take down with D. come up with
3. She set......some money each month for her holiday.
A. aside B. back C. about D. up
4. .......of half-starving wolves were roaming the snow-covered countryside.
A. Packs B. Herds C. Flocks D. Swarms
5. We had to get a bank loan when the money finally.......
A. gave off B. gave in C. gave over D. gave out
6. Mary always takes great care.......her children.
A. with B. for C. of D. to
7. ........he did not attend the English class, he knew the lesson quite well.

Page 23
A. In spite of B. Despite C. Although D. Because
8. Please don't enter.......knocking.
A. without B. with C. except for D. while
9. While studying he was financially dependent.......his parents.
A. of B. from C. on D. to
10. I took your pen........mistake.
A. of B. with C. by D. from
11. ........of all the staff, I would like to wish you a happy retirement.
A. In place B. On account C. Instead D. On behalf
12. He kept his job.......the manager had threatened to sack him.
A. despite B. even C. although D. unless
13. Mary doesn’t mind living.......her own.
A. with B. by C. at D. on
14. Please cut my hair.......the style in this magazine.
A. the same long as B. the same length as C. the same long like D. the same
length like
15. Ancient Egyptians mummified the dead bodies through the use of chemicals, ......ancient
Peruvians did through natural processes.
A. whether or not B. even though C. whereas D. because
11. 14. 17. 20. 23.
12. 15. 18. 21. 24.
13. 16. 19. 22. 25.

II. There is one mistake in each of the following sentences. Find and correct it.
26. Because the torrential rains that had devastated the area, the governor sent the National Guard to
assist in the clean-up operation.
27. One of the most important things in life is a good health.
28. The city has spent a big amount of money on crime prevention.
29. Comparing with other countries, Libya spends a high percentage of income on education.
30. People are now enjoying a higher level of living.
31. In the United Kingdom women see their doctor on the average five times a year.
32. Although Mark has been cooking for many years, he still doesn’t know to prepare French foods in
the traditional manner.
33. When we arrived at the store to purchase the dishwasher advertise in the newspaper, we learned that
all the dishwashers had been sold.
34. After rising the flag to commemorate the holiday, the mayor gave a long speech.
35. This time tomorrow I will lie on the beach, enjoying the sunshine.

MISTAKE CORRECTION MISTAKE CORRECTION


26. 31.
27. 32.
28. 33.
29. 34.
30. 35.

III. Give the correct form of the words in the brackets in each of the following sentences. Write your
answer in the space provided.
36. I was annoyed at his (REFUSE)______ to co-operate.
37. The book doesn’t say much about prices, but it is very (INFORM)______ about everything else.
38. The noise (LESS)______ as the plane got farther away.
39. He lost in the election because he was a weak and (DECIDE)______ leader.
40. I couldn’t help it. The accident was (AVOID)______.
41. She was (EXTREME)______ knowledgeable about the history of China.
Page 24
42. He was very (SET)_______ when his cat was run over.
43. Jackson had another violent (AGREE)_______ with the referee.
44. Many people were buried (LIVE)______ after the earthquake.
45. She studied (ECONOMY)______ at university.
36. 39. 42. 45.
……………………… ……………………… ……………………… ………………………
37. 40. 43.
……………………… ……………………… ………………………
38. 41. 44.
……………………… ……………………… ………………………

SECTION C: READING COMPREHENSION


I. Fill in each numbered blank with one suitable word. Write your answer in the space provided.
Australia is a big country, but nearly all Australians live near the sea. On hot summer days, you can
see thousands of people at the beach. Many beaches have waves (46)______ are very high.
These large waves are known as surf and the people who ride them are called surfers. Surfing is a
skill, and it needs learning. Don’t (47)______ to be able to surf properly the (48)______ time you try.
However, by practising a few times you will learn (49)______ to do it.
Surfing is not a new sport. Perhaps its origins need explaining. It started hundreds (50)______ years
ago in Hawaii. Men swam (51)______ to sea to catch fish and found they could come back to land very
quickly by riding the waves. These first surfers did not (52)______ a board. They were “body surfers”.
Many people (53)______ do this type of surfing today.
After a while people started to use boards and rode the waves by lying, kneeling or standing
(54)______ them. These first surfboards were made of wood and the water made them rot after a while.
Today, surfboards are made of plastic or fibreglass (55)______.
46. 49. 52. 55.
……………………… ……………………… ……………………… ………………………
47. 50. 53.
……………………… ……………………… ………………………
48. 51. 54.
……………………… ……………………… ………………………

II. Read the following passage and decide which answer (A, B, C, or D) best fits each gap. Write your
answer in the space provided.
What do you do well? What do you enjoy doing? Your answers to these two questions will help you
identify your (56)______. An employer will consider you seriously for a (57)______ when you can
show them that you know who you are, what you can offer and which you have studied. Sometimes it is
difficult to know what your weaknesses are. Clearly not everyone is equally good (58)______
everything. You may need to improve yourself and so (59)______ courses in that field could turn a
weakness into strength.
You will need to (60)______ some time on your self-assessment. Your honesty and the desire for
self-improvement will lead to (61)______ in getting the right job. Explore the following seven areas to
start to get to know yourself: your aptitude, your skills, your personality, the level of responsibility you
feel comfortable with, your interests and your needs.
Ask (62)______ if you have any special talents and if you need to consider your physical health
when choosing a job. Be as honest and realistic as you can, and ask for other people's (63)______ if
necessary. Make a list of these things. It is usually a good idea to talk about your aptitudes with teachers,
family and friends.
If you are considering a career that (64)______ a special talent, such as art, acrobatics, mathematics
or music, discuss your aptitudes with (65)______ expert in that area and discover how they fit the needs
of the occupation.
56. A. strong B. strength C. strengthen D. strengthened
57. A. position B. location C. spot D. room
58. A. upon B. in C. at D. for
59. A. meeting B. taking C. making D. interviewing
60. A. use B. make C. lose D. spend
Page 25
61. A. success B. successful C. successfully D. succeed
62. A. you B. your C. yours D. yourself
63. A. interests B. fields C. opinions D. attendances
64. A. requires B. asks C. tells D. urges
65. A. a B. an C. the D. this

56. 59. 62. 65.


57. 60. 63.
58. 61. 64.

III. Read the following text and choose the best answer for the questions below. Write your answer in
the space provided.
May 7th 1840 was the birthday of one of the most famous Russian composers of the nineteenth
century: Peter Ilyich Tchaikovsky, the son of a mining inspector. Tchaikovsky studied music as a child
and later studied composition at the St. Petersburg Conservatory. His greatest period of productivity
occurred between 1876 and 1890, during which time he enjoyed patronage of Madame von Meck, a
woman he never met, who gave him a yearly living stipend. Madame von Meck later terminated her
friendship with Tchaikovsky, as well as his living allowance, when she, herself, was facing financial
difficulties. It was during the time of Madame von Meck’s patronage, however, that Tchaikovsky
created the music for which he is most famous, including the music for the ballets of “Swan Lake” and
“The Sleeping Beauty”. Tchaikovsky’s music, well-known for its rich melodic and sometimes
melancholy passages, was one of the first that brought serious dramatic music to dance. Before this, little
attention had been given to the music behind the dance. Tchaikovsky died ostensibly of cholera on
November 6th 1893, though there are now some scholars who argue that he committed suicide.
66. The best title for this passage could be ______.
A. “The Life and Music of Tchaikovsky”
B. “Development of Tchaikovsky’s Music for Ballets”
C. “Tchaikovsky’s Relationship with Madame von Meck”
D. “The Cause of Tchaikovsky’s Death”
67. According to the passage, all of the following describe Madame von Meck EXCEPT ______.
A. she had economic troubles
B. she was generous
C. she was never introduced to Tchaikovsky
D. she enjoyed Tchaikovsky’s music
68. According to the passage, Tchaikovsky’s music is most well-known for ______.
A. its repetitive and monotonous tones
B. the ballet-like quality of music
C. the richness and melodic drama of the music
D. its lively, capricious melodies
69. Which of the following is NOT mentioned in the passage?
A. Tchaikovsky’s influence on ballet music.
B. Tchaikovsky’s unhappiness leading to suicide.
C. The patronage of Madame von Meck.
D. Tchaikovsky’s productivity in composing.
70. It can be inferred from the passage that ______.
A. it was not the music behind the dance that made Tchaikovsky famous
B. there is suspicion on the cause of Tchaikovsky’s death
C. Madame von Meck was one of the most famous Russian composers
D. Madame von Meck was one of Tchaikovsky’s girlfriends
66. 68. 70.
67. 69.

SECTION D: WRITING
I. Finish the second sentence in such a way that it means the same as the sentence printed before it.
71. It is extremely difficult for us to make ends meet these days.
Page 26
We find
......................................................................................................................................................
72. Alice and Charles did not decide to move to a bigger house until after the birth of their second
child.
Only when
...................................................................................................................................................
73. While mending the road, they accidentally blocked our water pipes.
They accidentally cut off
............................................................................................................................
74. He brought the umbrella along but it didn't rain.
He needn't
.................................................................................................................................................
75. While I strongly disapprove of your behaviour, I will help you this time.
Despite
.........................................................................................................................................................
II. Use the word given in brackets and make any necessary additions to write a new sentence in such
a way that it is as similar as possible in meaning to the original sentence. Do NOT change the form of
the given word(s).
76. When I was driving, I realized that the car wasn’t working properly.
(WRONG)
…………………………………………………………………………………………………
77. Sandra said that she was willing to work late.
(MIND)
…………………………………………………………………………………………………
78. They pretended to be enjoying themselves, but they weren’t really. (AS)
…………………………………………………………………………………………………
79. I can't lift this table on my own. (UNLESS)
…………………………………………………………………………………………………
80. The coins are believed to have been buried for safe-keeping. (IT)
…………………………………………………………………………………………………

III. Write a composition about the following topic:


What do you think an ideal teacher should be?
Give reasons for your answer and include any relevant examples from your own knowledge or
experience.
Write about 200 words.

ENGLISH PRACTICE 7
SECTIONI–PHONETICS
I.Choose the word whose underlined part is pronounced differently from the rest in the same line.(5p)
1.A.several B. suppose C. decisive D. sister
2.A. muddle B. struggle C. bush D. interrupt
3.A. ready B. measure C. steak D. stealth
4.A. wind B. behind C. mind D. find
5.A.chalice B. challenge C. choice D. chemist
II. Pick out the word that has the stress syllable differently from that of the other words.(5p)
1.A. application B. certificate C. biology D. security
2. A. university B. punctuality C. agricultural D.mathematician
3. A. identify B. secondary C. luxuriously D. majority
4. A. academic B. engineering C. availabe D.sympathetic
5.A. obligatory B. geographical C. international D. undergraduate
SECTION II- VOCABULARY AND GRAMMAR
I. Choose the best option ( A, B, C or D) to complete sentence below and write your answers in the
answer sheet.( 10p)
1. She refused to eat meat under any ………………… .

Page 27
A. circumstances B. occasion C. opportunity D. reason
2. It was difficult to guess what he ……………. to the news would be.
A. feelings B. reaction C. capital D. opinion
3. Though badly damaged by fire, the palace was eventually …… to its original splendour.
A. repaired B. renewed C. restored D. renovated
4. He didn’t know anyone at the wedding ………….. than the bride and the groom.
A. except B. other C. apart D. rather
5. When you are an old age pensioner, you have to learn to ………… a very small income.
A. live on B. live up to C. live out D. live down
6. I felt so much better, my doctor ………….. me to take a holiday by the sea.
A. suggested B. considered C. accepted D. advised
7. He said that the plane had already left and that I …………….. arrived an hour earlier.
A. must have B. had to C. should have D. was supposed to
8. Alice didn’t expect ……………. to Bill’s party.
A. to ask B. being askedC. to be asked D. asking
9.The restaurants on the island are expensive, so it’s worth ……………… a packed lunch.
A. take B. to take C. taking D. taken
10. When the alarm went off, Mick just turned ………..…. and went back to sleep.
A. down B. over C. up D. out
II.Put the verb in brackets into the correct tense.( 10p)
1.When he ( not arrive) ……….. by 6.00, I ( know) ………. He ( miss) ………… the bus.
2. Something ( tell) ………….. me that you ( not listen) …………… to a single word I (say) ………..
in the past tense minutes.
3. Same ( not receive) ……………..…… the parcel the last time I ( speak) …………….. to him.
4. When you ( feel) ……….… hungry, room service ( bring) ………….. you whatever you want.
III. Give the correct forms of the words in brackets to complete the blank space. ( 10p)
1.The judges describe Jones as a ….. criminal who was a danger to members of the public. HARD
2. Rescue team held out little hope of finding other …………. SURVIVE
3. He works for UNESCO in a purely ………….. role. ADVICE
4. …………… of the new system will take several days. INSTALL
5. This type of behaviour is no longer …………. acceptable. SOCIETY
6. Watching television can be very …………….. EDUCATION
7. Teachers must keep a record of students’ ……………….. ATTEND
8. Our school set up a project to ……………. the library system. COMPUTER
9.The breakdown of the negotiations was not ……………….. EXPECT
10. Those countries are ……. on other countries for most of their food. DEPEND
IV. Fill in each gap with a suitable preposition. (10 points)
1.……………… behalf of the department I would like to thank you all.
2. Many people expressed a strong preference …………. the original plan.
3. They were refused entrance ………… the exhibition .
4. He graduated …………….. York with a degree in Psychology.
5. We had an argument ………….. the waiter about the bill.
6. She complimented him ………….. his exellent German.
7. She sacrificed everything ……………. her children.
8. Sit down and make yourself ……………. home.
9. He works away …………… home during the week.
10. The searchers spread ………….. to over the area faster.
SECTION III – READING
I.Read the following passage carefully and then answer the questions below.(10p)
Traditionally the United States has been described as a melting pot, a place where the previous
identities of each immigrant group are melted down to create an intergrated, uniform society. Since the
1960s, many Americans have rejected the melting pot metaphor in favour of the image of the mosaic, a
picture created by assembling many small stones or tiles. In a mosaic, each peace retains its own
distinctive identity, while contributing to a larger design. Advocates of the mosaic metaphor assert that it
better represents the diverse multicultural society of the United States. Today, may Americans value
their immigrant heritage as an important part of their identity. More recent immigrant groups from Asia,
Page 28
such as Vietnamese Americans and Korean Americans, have established communities alongside those
populated by the descendants of European immigrants, such as French Americans, German Americans,
Irish Americans and Italian Americans.
1.Why has the United States often been described as a melting pot?
2. Why have people been in favour of the image of the “ mosaic”?
3. What does the word “mosaic” mean?
4. What have immigrant groups from Asia done to maintain their identity?
5. Name some recent immigrant groups from Asia in the United States?
II.Read the following passage and then choose the most suitable word for each space. (10 p)
CAR CATCHES FIRE
Three people jumped ……………( 1) a car on a busy Oxford road after a fire started under the bonnet
this morning. They were just able to rescue their possessions before the car burst into …………..(2). Mr.
Peter Collins, 25, of Wey Road, Berinsfield …………….(3) his Avenger astate car home ………….(4)
work with two friends when he noticed smoke coming into the car. He stopped, …………(5) was unable
to open the bonnet. He poured a bottle of water over the radiator where the smoke was coming from, but
could not put out the flames. He then ………… (6) to get fire extinguishers from a nearby…………(7) ,
but ………….(8) also failed to have any effect. ……….(9) he telephoned for a ……..(10) , but by the
time it arrived, the car was totally burned out.
1. A. into B. on C. out of D. over
2. A. flames B. petrol C. smoke D. sparks
3. A.drove B. has driven C. was driving D. was driven
4. A. to B. from C. at D. in
5.A. but B. because C. although D. so
6. A. wandered B. jumped C. walked D. ran
7.A. hospital B. garage C.library D. swimming pool
8. A. this B. those C. that D. these
9. A. At length B.Finally C. At the end D. Terminally
10. A. police car B. ambulance C. taxi D. fire engine
III.Read the passage below carefully. Then choose the correct answer: A, B, C or D that best answers
the question about the passage. (5p)
When we were in England last year, I went fishing with my friend, Peter. Early in the morning
we were sitting quietly by the side of the lake when we had an unpleasant surprise. We saw a duck
come along with three ducklings padding cheerfully behind her. As we watched them, there was a
sudden swirl in the water. We caught a glimpse of the vicious jaws of a pike- a fish which is rather like a
freshwater shark- and one of the ducklings was dragged below the surface.
This incident made Peter furious. He vowed to catch the pike. On three successive mornings we
returned to the vicinity and used several different kinds of bait. On the third day Peter was lucky. Using
an artificial frog as bait, he managed to hook the monster. There was a desperate fight but Peter was
determined to capture the pike and succeeded. When he had got it ashore and killed it, he wieghed the
fish and found that it scaled nearly thirty pounds- a record for that district.
1.Why do you think Peter was sitting quietly by the lake ?
A. He was watching the ducks. C. He was waiting for the pike to appear.
B. He wasn’t very talkative. D. He was fishing
2. To what does surprise in line 3 probably refer?
A. to the duck. B. to the ducklings. C. to the action of the pike. D. to the time of the day.
3. What were Peter’s feelings about the incident two days later?
A. He caught and killed the pike. C. He vowed that he would catch the remaining ducklings
B. He remained determined to catch the pike. D. He caught a frog and used it as bait for the pike.
4. How much was the pike worth?
A. about thirty pounds. B. about two hundred and forty dollars.
C. the passage contains no information on this point.
D. the passage says that the fish scaled nearly thirty pound.
5. Which of the following titles best sums up the whole passage?
A. Mysterious disappearance of ducklings. C. Record pike caught by an angry fisherman.
B. Revenge on a duck. D. Huge pike caught by fisherman after desperate struggle at sea.
Section IV – writing
Page 29
I. Finish each of the following sentences in such a way that it means exactly the same as the sentence
printed before it (5 points).
1. But for your unsuspected talents as a mechanic, he would have been stranded there for hours.
- If …………………………………………………………………………………………
2. Let’s invite the Browns to the party on Sunday.
- He suggested ……………………………………………………………………………..
3. The only thing that prevented the passing of the bill was the death of the Prime Minister.
- Had it not ..………………………….……………………………………………………….
4. I’ve forgotten that commentator’s name but he’s very well-known.
- That commentator, …………………………………………………………………………..
5. I only recognised him when he came into the light.
- Not until ……………………………….……………………………………………………..
II. Using the word given and other words, complete the sencond sentence so that it has a similar
meaning to the first sentence. (5points)
1.There’s no point in asking George to help. (WORTH)
- It ………………………………………………………….Gerge to help.
2. The last time I saw him was when I lived in London. ( SINCE)
- I …………………………………….. I lived in London.
3. I don’t normally go into town by car. ( USED)
- I ……………………………………………into town by car.
4. Nobody plays this piece as beautifully as he does. (MORE)
- He plays this piece………………………………………………….else.
5. I couldn’t go to work because of the transport strike. ( PREVENTED)
- I ……………………………………………to work by the transport strike.
III. Using the cues below to complete each sentence of the following passage. (5p)
1.Teacher/ Day/ Vietnam/ fall/ twenty/ November/ each year.
.....................................................................................................................................................................
2. General/ it / be/ mean/ schoolchildren/ show/ appreciation/ teacher/ who/ guide/ in/ study.
…………………………………………………………………………………………………………
3. It / a/ time/ forget/ bad/ experience/ student/ have/ teacher/ scold/ punish.
…………………………………………………………………………………………………………
4. Students/ give/ teacher/ flower/ small/ gift.
…………………………………………………………………………………………………………
5. Small/ party/ hold/ and/ there/ be/ atmosphere/ fun/ amusement/ school.
…………………………………………………………………………………………………………
IV. Writing an essay (10p)
Living in a tranditional family of three or four generations has both advantages and disadvantages. What
do you think of that matter? Write an essay of about 180- 200 words to support your points.

ENGLISH PRACTICE 8
Part I: Grammar and Vocabulary – Choose the best answer
1. Our holiday was_________ by the weather.
A. spoilt B. damaged C. overcome D. wasted
2. The________ charged by the architect for the plans of the new building were unusually high.
A. hire B. price C. fees D. sum
3. He_________ his son of the dangers of driving too fast in his new car
A. warned B. remembered C. threatened D. concerned
4. The child was_________ by a lorry on the safety crossing in the main street.
A. knocked out B. run across C. run out D. knocked down
5. When Ali arrived in London he spent all his time_________ and visited all the important
museums and buildings.
A. sight-seeing B. traveling C. looking D. touring
6. If you want a cheap air ticket you must_________ well in advance.
A. book B. engage C. reserve D. buy
7. His sister was full of_________ for the way in which he had so quickly learned to drive a car.
A. pride B. admiration C. surprise D. jealousy
Page 30
8. He asked if we would_________ to share the room.
A. accept B. consider C. agree D. approve
9. I wondered whether you would like to_________ to the theater tomorrow.
A. visit B. go away C. go out D. walk out
10. _________ I would like to say how pleased I am to be here.
A. Primarily B. Foremost C. Earliest D. First
11. The independent arbitrator managed to_________ the confrontation between the union and the
employers.
A. refuse B. confuse C. refute D. defuse
12. When I heard the footsteps behind me I was_________ that I would be attacked.
A. horrified B. terror-struck C. terrorized D. terrified
13. His illness made him_________ of concentration.
A. incompetent B. unable C. incapable D. powerless
14. Has the committee_________ a decision yet?
A. done B. made C. arrived D. voted
15. I am a bit hungry. I think_____________ something to eat.
A. I’ll have B. I’ll be having C. I’m going to have D. I’m having
16. What do you plan to do when you_____________ your course at college?
A. finish B. will finish C. have finished D. is going to finish
17. Where_____________? Which hairdresser did you go to?
A. did you cut your hair B. have you cut your hair
C. did you have cut your hair D. did you have your hair cut
18. ‘Shall I stay here?’ ~ ‘I’d rather_____________ with us’.
A. you come B. you to come C. you would come D. you came
19. I_____________ saying what I think.
A. believe B. believe in C. believe for D. believe when
20. Somebody ran in front of the car as I was driving. Fortunately I_____________ just in time.
A. could stop B. could have stopped
C. managed to stop D. must be able to stop
Part II: The passage below contains 10 mistakes. Underline the mistakes and write their
correct forms in the space provided in the column on the right. (0) has been done as an
example.
Traditional, mental tests have been divided into two types. 0. traditional → traditionally
Achievement tests are designed to measure acquiring skills and 1. _____________________
knowledge, particularly those that have been explicitness taught. The 2. _____________________
proficiency exams required by few states for high school graduation 3. _____________________
are achievement tests. Aptitude tests are designed and measure a 4. _____________________
person’s ability to acquire new skills but knowledge. For example, 5. _____________________
vocation aptitude tests can help you decide whether you would do 6. _____________________
better like a mechanic or musician. However, all mental tests are in 7. _____________________
some sense achievement tests because they assumption some sort of 8. _____________________
past learning or experience with certainly objects, words, or 9. _____________________
situations. The difference between achievement and aptitude tests is 10. _____________________
the degree and intention use.

Part III: Give the correct form of the words in brackets to fill in the blanks.
Dark black clouds in a dull sky meant one thing and one thing only: there was going to be a
(1.thunder)….. Not one of us had brought an umbrella, or even a (2.rain)……. So when Jack suggested
we should go to a museum, we all agreed immediately. As we had been (3. shop)………all morning we
were now feeling very tired, it would be a (4. pleasant)………...to sit down. We took a bus and arrived
just as large shops of rain were beginning to fall.
The museum was quite (5.desert)……and very peaceful. We sat down in the main hall and listened to
the rain (6. beat)…….against the windows.

Page 31
Suddenly, there was a great (7. disturb)……..at the (8. enter)……… a large party of schoolboys were (9.
lead) ………in by a teacher. The poor man was trying to keep them quiet and threatening to punish
them, but they did not pay the (10.slight)……. attention.

Section B: Reading

Part I:
The shark is a meat- eating fish and one of the most feared animals of the sea. Scientists (1)…………
about 250 species of fish as sharks. These fish live in oceans (2)………..the world, but they are most
common in warm seas.
Sharks (3)………….greatly in size and habits. Whale sharks, the largest kind of shark, may grow 60
feet long. A whale shark weighs up to 15 tons, more than twice (4)…………….much as an African
elephant. The smallest shark may (5)…………..only 4 inches long and weigh less than 1 ounce. Some
kinds of sharks live in the depths of the ocean, but (6)……………are found near the surface. Some
species live in coastal waters, but others (7)……………far out at sea. A few species can even live in (8)
…………..water.
All sharks are carnivores (meat- eaters). Most of them eat (9)…………fish, including other sharks. A
shark’s only natural enemy is a large shark. Sharks eat their prey whole, or they tear off large chunks of
flesh. They also (10)………………..on dead or dying animals.
1. A. classify B. divide C. organize D. arrange
2. A. all B. through C. throughout D. over
3. A. grow B. rise C. evolve D. vary
4. A. as B. so C. very D. exactly
5. A. stretch B. measure C. develop D. expand
6. A. some others B. others C. different kinds D. some sharks
7. A. dwell B. exist C. emigrate D. migrate
8. A. fresh B. sweet C. light D. clear
9. A. uncooked B. live C. lively D. alive
10. A. eat B. swallow C. exist D. feed

Part II: Fill in each blank with a suitable word to fill in the blanks.
Vitamins are substances required for the proper functioning of the body. In this century, thirteen
vitamins have been (1)…………
A lack of any vitamins in a person’s body can cause illness. In some cases, an excess of vitamins can
also (2)…………to illness. For example, sailors in the past were prone to (3)…………from scurvy that
is a disease resulting from the lack of vitamin C. It causes bleeding of the gum, loss of teeth and skin
rashes. Sailors suffer from scurvy because they did not eat fruits and vegetables. Fruits and vegetables
(4)…………vitamin C which is necessary for good (5)…………
Vitamin B complex is composed of eight different vitamins. A lack of any of these vitamins will
lead to different (6)…………For instance, a person who has too little vitamin B1 will suffer from beri-
beri, a disease that causes heart problems and mental (7)…………A lack of vitamin B2 results in eye
and skin problems while deficiency of vitamin B6 causes problems of the nervous system. Too little
vitamin B12 will cause anemia. The (8)…………that vitamin deficiencies caused certain diseases led
doctors to cure people suffering from these illnesses by giving them doses of the (9)……...vitamins.
Today, vitamins are (10)…………in the form of pills and can easily be bought at any pharmacy.

Part III: Read the following passage and answer the questions by choosing the best answer among
A, B, C or D.
Over the past 600 years, English has grown from a language of few speakers to become the dominant
language of international communication. English as we know it today emerged around 1350, after
having incorporated many elements of French that were introduced following the Norman invasion of
1066. Until the 1600s, English was, for the most part, spoken only in England and had not extended even
as far as Wales, Scotland, or Ireland. However, during the course of the next two centuries, English
began to spread around the globe as a result of exploration, trade (including slave trade), colonization,
and missionary work. That small enslaves of English speakers became established and grew in various
parts of the world. As these communities proliferated, English gradually became the primary language of
Page 32
international business, banking, and diplomacy.
Currently, more than 80 percent of the information stored on computer systems worldwide is in English.
Two thirds of the world’s science writing is in English, and English is the main language of technology,
advertising, media, international airports, and air traffic controllers. Today there are 700 million English
users in the world, and over half of these are nonnative speakers, constituting the largest number of
nonnative users of any language in the world.
1. What is the main topic of the passage?
A. The number of nonnative users of English.
B. The French influence on the English language.
C. The expansion of English as an internatonal language.
D. The use of English for science and tecnology.
2. English began to be used beyond England approximately.............................
A. in 1066 B. around 1350 C. before 1600 D. after 1600
3. According to the passage, all of the following contributed to the spread of English around the world
EXCEPT A. the slave trade B. the Norman invasion
C. missionaries. D. colonization
4. Which of the following statements is NOT true?
A. Most of the information stored on computer systems is in English.
B. Only one thirds of the world’s science writing is in languages other than English.
C. English is the only language used in technology, and advertising.
D. International airports and air controllers use mostly English.
5. According to the passage, approximately how many nonnative users of English are there in the world
today?
A. A quarter million B. Half a million C. 350 million D. 700 million.

Part IV:
BITTER WATER HITS THE BIG TIME
Chocolate, which has its origins in South America, is now part of a multi-million pound
worldwide business.
At Easter, British people spend over $230 million on chocolate. A massive eight per cent of all
chocolate is bought at this time.
(1)____. Although the large scale industrial production of chocolate began in the last century,
the cacao plant was first cultivated by the Aztec, Toltec and Mayan civilizations of Central America
over three thousand years ago.
The cacao tree is an evergreen, tropical plant which is found in Africa, South and Central
America, the West Indies and South East Asia. The fruit of this tree is melon-sized and contains 20-40
seeds. (2)____. In English – speaking countries, they are called cocoa beans. This is a misspelling from
the 17th century when they were also called cacoa and cocao beans.
The Aztecs used cocoa beans as money. (3)____. This is from the world in the Aztec language,
Nahuatl, meaning “bitter water”. (4)____. The Spanish found the drink more palatable mixed with
cinnamon and sugar, but the recipe did not spread to the rest of Europe for another century. In the late
17th century, chocolate houses were set up in Europe’s capital cities, where people gathered to drink
chocolate.
(5)____. But in 1826, CJ van Houten of the Netherlands invented chocolate powder. (6)____.
The age of the chocolate bar as we know it began in 1847 when a Bristol company, Fry and
Sons, combined cocoa butter with pure chocolate liquor and sugar to produce a solid block that you
could eat. (7)____.
At the turn of the century, the British chocolate market was dominated by French companies. In
1879 the English company Cadbury even named their Birmingham factory Bournville (ville is the
French word for town) in the hope that a little glamour would rub off. But then came Cadbury’s famous
Dairy Milk bar which began life as a Dairymaid in 1905. (8)____.It seems that, for the time being at
least, chocolate intake in Britain has established at about four bars each week. (9)____. The latest market
trick is the so-called “extended line”. This is when the humble chocolate bar becomes an ice cream, a
soft drink or a dessert, to tempt chocoholics who have grown tired of conventional snacks.

Page 33
At the other end of the production process, cacao farmers are still feeling the effects of a crash
in cocoa bean prices at the end of 1980s. (10)____. Perhaps you could spare a thought for them as you
munch your next chocolate bars.

A. This was made by extracting most of the cocoa butter from the crushed beans.
B. A Swiss company then introduced milk solids to the process which gave us milk chocolate.
C. They also used them to make a drink called xocoatl.
D. Until the last century, the chocolate drink was made from solid blocks of chocolate which had to be
melted down in hot water.
E. When dried they become cacao beans, which can be used to make chocolate.
F. Clever advertising which associated it with the healthy qualities of milk from the English
countryside quickly established the bar as a rival to the more decadent French brands.
G. British manufacturers include up to 5 per cent vegetable fat in their chocolate, something forbidden
elsewhere.
H. As most cacao farmers operate on a very small scale, many were forced out of business.
I. This has forced manufacturers to look for new ways to attract customers.
J. In Aztec times the chocolate drink was flavored with spices and used on ceremonial occasions and
for welcoming visitors.
K. Only at Christmas do people eat more of the cocoa-based foodstuffs.

Section C: Writing
Part I: Rewrite the following sentences so that they have a similar meaning with the first one.
1. “Don’t forget to phone the police”, she said
She reminded him .................................................. .................................................. .........
2. It is believed that the man escaped in a stolen car.
The man............................................... .................................................. ...................
3. A small church lies at the foot of the hill.
At the foot .................................................. .................................................. .....................
4. If you changed your mind, you’ll be welcome to join our club .
Were you .................................................. .................................................. ......................
5. We don’t have to do so many things to please him.
It is .................................................. .................................................. .................................
6. I’m sure he didn’t do it by himself.
He................................................ .................................................. ........................................
7. He can’t afford to go to America this summer.
He doesn’t .................................................. .................................................. ......................
8. Timmy has become confident as a result of his success .
Timmy’s success has turned............................................ ................................................
9. I haven't seen my uncle for a long time.
It's a long time .................................................. .................................................. ....................
10. She dances beautifully and she sings sweetly, too.
Not only .................................................. .................................................. .............................

PART II: Rewrite the sentence, using the given word.

Page 34
1. You should think about the price before you decide whether to buy it or not. (consideration)
→………………………………………………………………………………………….
2. People don’t want to buy cars with large engines any more. (call)
→………………………………………………………………………………………….
3. Twenty years ago this region produced twice as much coal as it does now.(halved)
→………………………………………………………………………………………….
4. The prime minister in unlikely to call an early general election. (likelihood)
→………………………………………………………………………………………….
5. The policeman acted quickly and averted an accident (prompt)
→………………………………………………………………………………………….

Part III: Write a composition (300 words) about the following topic:
How do movies and television influence people’s behavior? Use reasons and specific examples to
support your answer.

ENGLISH PRACTICE 9
PART A. PRONUNCIATION
I. Choose the word whose stress pattern is different from the rest
1. A. literature B. concentrate C. comfortable D. rhinoceros
2. A. produce B. protect C. project D. purchase
3. A. policeman B. candidate C. fantastic D. position
4. A. counterpart B. outstanding C. applicant D. wilderness
5. A. mosquito B. document C. literature D. business
6. A. magazine B. preference C. cigarette D. engineer
7. A. regardless B. coincide C. specific D. inventive
8. A. promise B. tonight C. furnish D. reason
9. A. electronic B. proficiency C. petroleum D. equivalent
10. A. substantial B. dramatic C. terrorist D. demolish
PART B. LEXICO - GRAMMAR
I. Complete each of the following sentences with the correct answer (A, B, C or D). Identify your answer
by writing the corresponding letter A, B, C or D on your answer sheet.
1. You need a passport to cross the _________between Mexico and the United States.
A. edge B. line C. border D. rim
2. When you _______ your destination, your tour guide will meet you at the airport.
A. arrive B. get C. reach D. achieve
3. Rachel is mad ______ the Eurovision.
A. about B. for C. from D. against
4. We are pleased to announce that Keith Danish ______ replace Susan Williams as Operations Manager
from 24th September.
A. does B. is about to C. is to D. is due
5. Make sure you mix the ingredients well, ________ you might get lumps in your cake.
A. unless B. otherwise C. provided D. supposing.
6. If you hadn’t taken the money, you _____ in prison now.
A. wouldn’t be B. wouldn’t have been C. haven’t been D. hadn’t been.
7. The bank is reported in the local newspaper _______ in broad daylight yesterday.
A. to be robbed B. robbed C. to have been robbed D. having been robbed
8. Fiona can’t _______ about the meeting. I reminded her this morning!
A. forget B. have forgotten C. be forgetting D. have been forgetting
9. The doctor _______the cut on my knee and said it had completely healed up.
A. examined B. investigated C. researched D. looked into
10. Dr parker gave my mum a lovely ______for spaghetti carbonara.
Page 35
A. receipt B. recipe C. prescription D. paper
11. I was shocked when I crashed the car but at least I wasn’t __________.
A. injured B. broken C. damaged D. spoilt
12. I glanced at the newspaper and saw that the _______ said “President Resigns”
A. headline B. heading C. subtitle D. chapter
13. I think my favorite _____ is probably table tennis.
A. athletics B. exercise C. sport D. gym
14. I think the discussion has gone on _______ and we should make a decision.
A. long enough B. such long C. so long D. enough long
15. A: “Could you do something for me?” B: _________________
A. Certain! B. Certainly C. Right! D. Never mind!
16. Lucy was late for school this morning because the alarm didn’t _______ as usual.
A. ring off B. get off C. go off D. take off
17. She applied for paid leave but her boss rejected her application__________.
A. in hand B. on hand C. at hand D. out of hand
18. I’m older than you are, ____________?
A. am I B. isn’t it C. aren’t I D. aren’t you
19. ______ parrots are native to tropical regions is untrue.
A. That all B. All C. Why all D. Since all
20. Jane: “Would you rather go to the beach or to the mountains?” Kim: “_______________”
A. That’s very nice of you C. I’d love to go
B. Thanks for the mountains D. The beach definitely
II. There are 10 mistakes in the following passage. Find and then correct them. Write your
answers on your answer sheet.

It is very difficult to succeed in the music business; nine out of ten bands
that release a first record fail to produce the second. Surviving in the music
industry requires luck and patience, but most of all it requires and intricate
knowledge of how a record company is functioned. The process begins when a
presenter of a company’s Artist and Repertoire (A&R) department visits bars
and night clubs, scouting for young, talented bands, After the representative
identifies a promised band, he or she will work to negotiate a contract with that
band. The signature of this recording contract is a slow process. A company will
spend a long time to investigate the band itself as well as current trends for
popular music. During this period, it is important that a band reciprocates with
an investigation of its own, learning as much as possible about the record
company and making personnel connections within the different departments
III. Supply thehandle
that will correct form
their of the word in capital letter. Write your answers on your answer
recordings.
sheet.

Page 36
Culture clashes
One (1)______(CHARACTER) of the modern world is that people increasingly find themselves
living side by side with people from other cultures. While in the past people with different cultures
were able to live quite (2)______(SEPARATE), high mobility and freedom of movement mean
that we are more likely today to be confronted with people whose way the life is
(3)______(FAMILIAR) to us. In such circumstances, (4)______(RACE) is a read danger.
People feel a tremendous (5)_____(LOYAL) to their own culture, and are often unwilling to
develop an (6)______(APPRECIATE) of the positive aspects of other cultures. They may feel that
another culture presents a threat to their own (7)______(INHERIT), one that could even lead to
the (8)_____(APPEAR) of certain aspects of their way of life. Often, however, this threat is more a
matter of (9)_____(PERCEIVE) than reality and different groups live in (10) ______(RELATE)
harmony in many parts of the world

PART C. READING
I. Read the following passage and choose the best answer for each question. Identify your answer by
writing the corresponding letter A, B, C or D on your answer sheet.
This symptoms of hay fever include watery and itchy eyes and a runny, congested nose. People
suffering from hay fever may experience occasional wheezing and repeated bouts of sneezing and may
even lose their sense of smell. Some victims of hay fever may also have stopped-up ears. About 30
percent of those who suffer from hay fever may develop the symptoms associated with periodic asthma
or a sinus infection. The allergen-antibody theory does not fully explain allergic reactions because the
membranes and glands in eyes and ears are controlled by the independent nervous system, which keeps
these organs in balance. But the independent nervous system itself is part of the emotional- response
center and may cause the feelings of anger, fear, resentment, and lack of self-confidence in reaction to
allergy-causing substances.
The most common cause of hay fever is the pollen of ragweed, which blossoms during the
summer and autumn. When airborne pollen particles, as well mold, come into contact with the victim’s
membranes, they can cause allergic reactions that release histamine and result in virtual blockage of air
passages. To prevent hay fever or to decrease the severity of its symptoms, contact with the ragweed
pollen should be reduced. Although some communities have attempted to eliminate the plants that cause
the reactions, elimination programs have not been successful because airborne pollen can travel
considerable distances. Antihistamine can help with short but severe attacks. Over extended periods of
time, however, patients are prescribed a series of injections of the substance to which they are sensitive
in order to increase immunity and thus be relieved of the seasonal allergy.
(Resource: Toefl test strategies by Eli Hinkel, Ph.D)
1.It can be inferred from the passage that the phrase “hay fever” refers to
A. fodder for cattle B. a seasonal discomfort
C. viral bacteria D. a lung disease
2. According to the passage, the symptoms of the allergy are predominantly
A. abdominal B. intestinal C. respiratory D. chronic
3. What can be inferred from the first paragraph?
A. Hay fever may cause severe allergic reactions and even death.
B. The cause of allergic reactions has not been determined.
C. The nervous system balances allergic reactions.
D. People should not have an emotional response to allergic reactions.
Page 37
4. According to the passage, patients suffering from hay fever may also experience
A. hunger pains B. mood swings
C. nervous blockages D. sensory perceptions
5. The word “resentment” is closest in meaning to
A. reprieve B. reprisal C. irritation D. grief
6. According to the passage, the irritants are transported by
A. wind B. food C. travelers D. air passages
7. The word “blockage” in the passage is closest in meaning to
A. obstruction B. bleeding C. enlargement D. dryness
8. According to the passage, to avoid incidents of hay fever, patients need to
A. avoid interactions with other patients
B. avoid exposure to pollen
C. increase their self-confidence
D. take doses of prescribed medicine.
9. Which of the following is NOT mentioned in the passage as a cause of allergies?
A. pollen B. mold C. flowers D. injections
10. A paragraph following this passage would most probably discuss
A. how the nervous system alerts patients.
B. how the immune system reacts to allergens.
C. what other diseases can be relieved by vaccines.
D. What flowers are harmless to hay fever patients.
II. Complete the following passage by choosing the correct option (A, B, C or D) to fill in blanks.
Write your answers on your answer sheet.
Down from the mountains
On the fourth day, We started our descent from the mountain range. We walked down the narrow track
in single (1)______, taking care not to slip on the wet surface. We followed the river down a steep-sided
(2)_____, stopping just once along the way to take a shower under a beautiful______(3). Soon the
(4)_______ began to change, with more and more trees and (5) ______vegetation. Several (6)_______
of birds flew overhead, and we saw an amazing multi-coloured snake lying under a small (7)______ as
we walked past. By the afternoon, the countryside was changing again as we approached the coast. Here,
there were (8)_______ where farmers grew a variety of crops such as wheat, and we saw a
(9)_________ of cows near a farmhouse. Then we climbed up and down some tall, grass-covered
(10)_______ , and at last we had arrived at the sea.
1. A. line B. file C. track D. queue
2. A. channel B. lagoon C. canal D. valley
3. A. pond B. waterfall C. current D. bath
4. A. scenery B. sights C. settings D.scenes
5. A. dense B. tight C. intense D. close
6. A. crews B. bunches C. flocks D. crowds
7. A. wood B. hedge C. bush D. forest
8. A. deserts B. fields C. shores D. jungles
9. A. team B. gang C. pack D. herd
10. A. dunes B. beaches C. plains D. fences
II. Read the text below and think of the word which best fits each space. Use only ONE word in
each space. Write your answers on your answer sheet.
Travel Insurance
When going on holiday, it is always a good idea to take out travel insurance. This is just in case
something goes (1) _____ along the way. You could lose your luggage, you could be robbed, or even

Page 38
become ill and need expensive medical treatment. For millions of holiday makers, travel insurance is
just a precaution (2)_______ will help them have an enjoyable and worry-free holiday. But for (3)_____,
travel insurance is a way of earning money (4) ______ making false claims against insurance
companies. For (5) ______ some people pretend that they have had expensive equipment stolen which in
(6) ______ never even existed, and then claim large sums in compensation. Such claims cost insurance
company a total (7)_____ £ 50 million per year. But the cheats’ luck is about to run (8)____ . (9) ______
to a new computer system, companies will be able to tell at a glance (10) _____ someone has made a
claim within the last three years. Honest travelers will no longer have to pay through the nose for other
people’s dishonesty.
PART D. WRITING
I. Complete the second sentence so that it has the similar meaning to the first sentence. Write your
answers on your answer sheet.
1. Is there any chance of his coming to the party? (likely)
Is he ________________________________________________________?
2. I’m sure it wasn’t Jim who did it. (have)
It ________________________________________________________
3. He promised me he would be back before midnight. (word)
He _________________________________________________________
4. He prefers cycling to walking. (prefer)
He would _____________________________________________________.
5. That dress is a third of the cost of the blue one. (much)
The blue dress is _______________________________________________
II. Essay writing
” Nowadays, the increasing number of schools in Vietnam are facing with the alarming issue, school
violence. What do you think are the causes of this? What solutions can you suggest?”
You should write about 250 words.

ENGLISH PRACTICE 10
PART A. PRONUNCIATION
Choose the word whose stress pattern is different
1. A. different B. tradition C. opera D. capital
2. A. individual B. unemployment C. difficult D. Population
3. A. apply B. appear C. visit D. attend
4. A. kindergarten B. conventional C. contaminate D. curriculum
5. A. corporate B. different C. engineer D. difficult
6. A. adorable B. ability C. entertainment D. impossible
7. A. company B. customer C. atmosphere D. employment
8. A. university B. institution C. preferential D. indicative
9. A. environment B. difficulty C. inhabitant D. community
10. A. impossible B. entertainment C. adorable D. ability

PART B. LEXICO-GRAMMAR
I. Complete each of the following sentences with the correct answer (A, B, C or D). Identify your answer by
writing the corresponding letter A, B, C or D on your answer sheet.
1. The audience couldn’t listen to the lengthy and pointless speech _________.
A. far any more B. any longer C. any more far D. any lengthier
2. He missed two most important lectures. He _________very ill.
A. had to be B. must be C. was to be D. must have been
3. Allan: “Do you mind if I use your dictionary?” Nick: “_______.”
A. I’m afraid not B. Without doubt C. No, feel free D. Straight ahead
Page 39
4. Oh, I’m always forgetting _______ these medicines. Is that before or after meal, Ron?
A. when to take B. what I will take with
C. on which I should take D. when I take
5. We rang the doorbell again _______ they hadn’t heard it the first time.
A. because B. although C. for fear of D. in case
6. How boring the lecture was! I _______ just in the first half of it.
A. dropped off B. fell out C. called off D. came out
7. Will it make any _______ to them if we deliver their equipment tomorrow?
A. displeasure B. alteration C. difference D. conflict
8. What time are you _______ duty? Let’s have a coffee after that.
A. over B. on C. off D. out of
9. Don’t give up your harboring dream. _______ working harder and harder.
A. Try out B. Keep on C. Speed up D. Go off
10. It’s been a good year. I’ve written two books and a couple of articles, and _______ are now in print.
A. both of which B. both of them C. all of which D. all of them

II. Supply the correct form of the word in capital letter. Write your answers on your answer sheet.
With the development of fast motorway systems in many countries, once remote parts of the countryside
are (1. INCREASE) __________ becoming (2. ACCESS) ____________to people who live in towns
and cities. In many parts of the world, rising (3. PERSON) ____________incomes have resulted in mass
(4. OWN) ____________ of motor cars at a time when people have more leisure time to fill.
This means that there is a lot of (5. PRESS) ___________ on the countryside to accept a growing
number of visitors. In Britain, for example, the (6. NATION)____________ parks are used by more
than one hundred million people annually.
In (7. ADD) __________, similar numbers visit areas that do not enjoy the same level of (8. PROTECT)
____________. Some people regard this invasion of the countryside as (9. DESIRE) ____________and
they have begun campaigning to try and save some aspects of (10. TRADITION)____________ rural
culture.

III. In most lines of this text there is one unnecessary word. It is either incorrect gramm atically, or
does not fit the sense of the text. Write the unnecessary word in the space beside the text.
Tick (√) each correct line. There are two examples at the beginning. Transfer your
answers to your answer sheet.
I am really keen on going to the cinema, so I’ve got lots of favourite films. But 0 …√………
the best one as I’ve seen lately is called “Pressure”. In some ways, I suppose 00 …as………
that you could regard it as a detective film but it’s different from most films of 1 …………
that kind because the characters are they such unusual people. The detective in 2 …….………
it, for example, is a computer expert who solves crimes on her computer using 3 ….………
information given to her by her assistants, who go out and interview to people. 4 ……………
The case in the film concerns about the wife of a millionaire, who has gone 5 ……..………
missing. Sometimes the plot gets a bit complicated but it isn’t too hard to keep 6 …….…….
up with it. There are a lot of strange characters in it, such as a man who always 7 …….….…
wears two hats on, and some of the scenes really made me laugh. Also, there is 8……….……
a big surprise at the end but I won’t say what that is in the case you go to see it. 9. ……….……..
It’s very well acted and I also like the music in it. But what do I really like most 10. …………….
about the film is that it’s so original- I’ve certainly seen another film quite like
that.
PART C. READING
I. Read the following passage and choose the best answer for each question. Identify your answer by
writing the corresponding letter A, B, C or D on your answer sheet.
Have you ever thought about the names of the months? Why are “January” and “February” not
called “Primo” or “Secondo”? Is it because the original names were created in ancient times? Or is it
because the originators preferred odd words?
Take February, for example. Say it aloud a few minutes and you start to wonder. Most people
don’t know who developed these names. However, a little research reveals that the names of the months
Page 40
came mostly from a combination of the names of Roman gods and goddesses, important festivals, and
the original numbers of the months.
Julius Caesar and Pope Gregory XIII change the calendar to make it more exact. Caesar
developed a new calendar of 364 and a quarter days, the time it takes the earth to orbit the sun from one
spring season to the next. The Pope’s astronomers refined the calendar regarding leap years; they
determined that there should be no leap year in years ending in 00- unless they were divisible by 400;
the years 1700, 1800, 1900 and 2100 would not be considered leap years, while the years 1600 and 2000
would be. This new Gregorian calendar was so accurate that today, scientists need only add leap seconds
every few years to the clock in order to keep the calendar matching the Earth’s cycles.
1. What is the topic of the passage?
A. how the modern calendar was named and developed
B. how the months were named
C. how the leap year system was developed
D. how accurate the modern day is calendar
2. It can be inferred from paragraph 1 that the author think the names of the months are __________.
A. odd B. difficult to pronounce C. inappropriate D. none of the answers
3. The word “they” in paragraph 3 refers to ______________.
A. calendars B. days C. astronomers D. years
4. The word “accurate” in paragraph 3 is closest in meaning to ____________.
A. interesting B. informative C. correct D. simple
5. Which of the following will be a leap year?
A. 2300 B. 2400 C. 2200 D. 2500
6. Which of the following is true of the Gregorian calendar?
A. It needs major improvements.
B. It was so well designed, it needs little adjusting today.
C. It copied the Roman calendar’s formula of leap years.
D. none of the answers
7. Why is Caesar important in calendar making?
A. He changed the length of the year
B. He extended summer.
C. He has a month named for him
D. He altered the number of days in the year.
8. In what order is the information in the passage presented?
A. Caesar’s calendar, the Gregorian calendar, the modern calendar
B. Roman Gods, important festivals, original numbers of months
C. names of months, Caesar’s calendar, the Gregorian Calendar
D. none of the answers
9. The word “refined” in paragraph 3 is closest in meaning to ____________.
A. studied B. invented C. observed D. improved
10. Why is the number of 364 and a quarter important?
A. It is the length of time from the beginning of spring to the end of winter.
B. It is the length of a planetary year.
C. It is the most accurate number for calendars.
D. It was a number randomly chosen by Caesar for his calendar.

II. Read the text below and think of the word which best fits each space. Use only ONE word in each
space. Write your answers on your answer sheet.
The legend of the root
Ginseng is one of the great mysteries of the east. Often referred to as the “elixir of life”, its
widespread use in oriental medicine has led to many myths and legends building up around this
remarkable plant. Ginseng has featured (1) ____ an active ingredient in oriental medical literature for
over 5,000 years. Its beneficial effects were, at one time, (2) ____ widely recognized and praised that the
root was said to be worth its weight in gold.
(3) ____ the long history of ginseng, no one fully knows how it works. The active part of the
____ (4) is the root. Its full name is Panax Ginseng – the word Panax, (5) ____ the word panacea,
coming from the Greek for “all healing”. There is growing interest by western scientists in the study of
Page 41
ginseng. It is today believed that this remarkable plant may (6) ____ beneficial effects in the treatment of
many diseases which are difficult to treat with synthetic drug.
Today, ginseng is (7) ____ longer a myth or a legend. Throughout the world it is becoming
widely recognized that this ancient herb holds the answer to relieving the stresses and ailments of
modern living. It is widely used for the treatment of various ailments (8) ____ as arthritis, diabetes,
insomnia, hepatitis and anemia. However, the truth behind (9) ____ ginseng works still remains a
mystery. Yet its widespread effectiveness shows that the remarkable properties are (10) ____ than just a
legend.

III. Read the following passage and choose the best answer for each question. Identify your answer by
writing the corresponding letter A, B, C or D on your answer sheet.
Environmental Concerns
Earth is the only place we know of on the universe that can support human life. (1) ________
human activities are making the planet less fit to live on. As the western world (2) _______ on
consuming two- thirds of the world’s resources while half of the world’s population do so (3) ________
to stay alive we are rapidly destroying the (4) _______resource we have by which all people can survive
and prosper. Everywhere fertile soil is (5) ___________built on or washed into the sea. Renewable
resources are exploited so much that they will never be able to recover (6) ______. We discharge
pollutants into the atmosphere without any thought of the consequences. As a result the planet’s ability
to support people is being (7) _______ at the very time when rising human numbers and consumption
are (8) _______ increasingly heavy demands on it. The Earth’s natural resources are there for us to use.
We need food, water, air, energy, medicines, warmth, shelter and minerals to (9) _______ us fed,
comfortable, healthy and active. If we are sensible in how we use the resources they will (10) _______
indefinitely. But if we use them wastefully and excessively they will soon run out and everyone will
suffer.
1. A. Although B. Still C. Yet D. Despite
2. A. continues B. repeats C. carries D. follows
3. A. already B. just C. for D. entirely
4. A. alone B. individual C. lone D. only
5. A. sooner B. neither C. either D. rather
6. A. quite B. greatly C. utterly D. completely
7. A. stopped B. narrowed C. reduced D. cut
8. A. doing B. having C. taking D. making
9. A. hold B. maintain C. stay D. keep
10. A. last B. stand C. go D. remain

PART D: WRITING
I. Complete the second sentence so that it has the similar meaning to the first sentence. Write your
answers on your answer sheet.
1. Its lack of irregular verbs makes Esperanto a unique language.
Unlike_________________________________________________.
2. Mr. Smith knew little about the Internet, so he didn’t invest into any computer companies.
Had_________________________________________________.
3. The Board of Directors discussed the business in length, but came to no decision.
The Board of Directors had ________________________________.
4. When she got to the party, everyone was dancing and singing.
On_________________________________________________.
5. Barbara runs a successful company and she also manages to look after her five children.
Not only_________________________________________________.

II. Write a passage of about 200 words to show your opinion on the following statement:
“Some people believe that children’s leisure activities must be educational, otherwise they are a
complete waste of time”

Page 42
ENGLISH PRACTICE 11
I. Choose the word that has a different stress pattern from the others.
1. A.shower B.allow C.shout D.arrow
2. A.increase B.creation C.mean D.cheat
3. A.danger B.exchange C.blame D.anger
4. A.these B.theory C.worth D.threaten
5. A. chamber B. ancient C. danger D. ancestor
6. A. smoothly B. southern C. breath D. airworthy
7. A dozen B glove C love D stove
8. A kites B catches C oranges D buzzes
9. A wood B good C food D childhood
10. A whistled B laughed C lodged D received
I. Choose the word that has a different stress pattern from the others.
16. A. community B. developing C. conditioner D. interested
17. A. continue B. importance C. different D. directed
18. A. medicines B. opposite C. pollution D. capable
19. A. preservation B. inspiration C. disposable D. popularity
20. A. exhausted B. atmosphere C. suspect D. computer
II. Choose the word, phrase or expression which best completes each sentence.
21. Hoa: “Are you going to buy a new computer or just continue using the old one?”
Mary: “_________”
A. Yes, I am. B. Yes, I’d like one. Thank you.
C. That’s impossible. I can’t afford a new one. D. Neither. I’m going to lease one.
22. Mr. Black: “What a lovely house you have!”
Mr John: “_________”
A. No problem B. Thank you. Hope you will drop in.
C. Of course not, it’s not costly D. I think so.
23. You have never been to Italy, ________?
A. have you B. haven’t you C. did you D. had you
24. You can’t tell what someone is like just from their ________.
A. character B. looking C. appearance D. personality
25. “How is it going?” - “________”
A. By bike B. Not much C. It sounds better D. Mustn’t grumble
26. ________ a dentist, Mike is very concerned about having healthy teeth.
A. Because B. He is C. As D. That he is
27. ________, you need to achieve a score of 60% or more.
A. To pass this test B. For being passed this test
C. In order pass this test D. So that to pass this test
28. As a famous person ________ many children admire, it is important for her to act responsibly.
A. whose B. whom C. which D. when
29. The brochure says that the hotel has a great ________ of the sea.
A. appearance B. look C. sight D. view
30. Our new coach is popular ________ the whole team.
A. with B. to C. by D. for
31. As soon as you ________ that, I’d like you to go to bed.
A. have done B. did C. will do D. will have done
32. Margaret was slow at school, but she went on ________ Prime Minister.
A. being B. to be C. having been D. to have been
33. In 1870, ________, John D. Rockefeller and others created the Standard Oil Company.
A. in spite of oil prices fluctuated B. despite fluctuating oil prices
C. but the oil prices fluctuated D. oil prices were fluctuating
34. They’re staying in rented accommodation for the time ________.
A. going B. making C. doing D. being
35. “I have an idea. Let’s go for a swim on Sunday afternoon”. - “________”
A. OK, what time? B. You’re kidding C. I know D. I’m sure

Page 43
III. Give the correct form of the words in brackets.
36. We found it ____________ (thrill) to your wonderful news. THRILLING
37. He left the room without any ____________ (explain). EXPLANATION
38. He didn’t feel happy because he worked ____________ (success). UNSUCCESSFULLY
39. Many people expressed ____________ (disagree) with the whole idea. DISAGREEMENT
40. There was a ____________ (wide) dissatisfaction with the government’s policies. WIDESPREAD
41. Her health has ____________ (bad) considerably since we last saw her. WORSENED
42. A lot of plants and animals could be used as medicines against cancer, AIDS, heart diseases and
other ____________ (sick). SICKNESSES
43. He was ____________ (information) of the consequences in advance. INFORMED
44. I was kept ____________ (wake) last night by the noise from a party in the flat above. AWAKE
45. This road is so bad that it needs ____________ (surface). RESURFACING
IV. Choose the underlined word or phrase in each sentence that needs correcting.
46. Last week unless my mother had had enough money, she would have bought that toy for me.
47. It often takes me about fifteen minutes to go to work from here by foot.
48. Those people say that it is such polluted air that they can’t breath, don’t they?
49. It is noisy enough in this room, so I would rather you stop shouting like that.
50. Dr. Roberts, the first woman to be elected president of the university, is intelligent, capable and
awareness of the problem to be solved.
V. Fill in the gaps
A. terrible B. size C. hope D. wrong E. loose F. problem
G. want
Choosing clothes can be difficult. Some people (51)________ to be fashionable, but they don’t
want to look exactly like everybody else. Not all clothes are suitable for work or school, perhaps because
they are not formal enough, or simply not comfortable. It is easy to buy the (52)_________ size, and
find that your trousers are too tight, especially if you are a little bit overweight. Very (53)________
clothes make you feel slim, but when they have shrunk in the washing machine, then you have the same
(54)________! If you buy light cotton clothes, then they might not be warm enough for winter. If your
shoes are not tight, and if you aren’t dressed for the cold, you might look good, but feel (55)________!
II. Read the following passage and decide which option A, B, C or D best fits each space.
(10 points)
Schools in the United States have not always had a large number of libraries. As (56)________as
1958 about half of the public schools in the United States had no libraries at all. The number of public
school libraries increased dramatically (57)_______ the federal government passed the Elementary and
Secondary Education Act of 1965, ( 58)_______ provided funds for school districts to improve their
education programs and facilities, including their libraries. (59) ________, many educators claim that
since the legislation was passed federal spending has not increased sufficiently to meet the rising
(60)______ of new library technologies such as computer databases and Internet access.
Because the federal government provides only limited funds to schools, individual school districts
(61) _______ on funds from local property taxes to meet the vast majority of public school expenses.
Therefore, the libraries of the public schools tend to reflect the (62) ______ capabilities of the
communities in which they are located. Districts in wealthy suburbs often have fully staffed libraries
(63) _______ abundant resources, spacious facilities, and curricular and instructional support. In (64)
______, school districts in many poor areas house their libraries in ordinary classrooms or in small
rooms. The libraries in such areas are generally staffed by volunteers, who organize and (65) ______
books that are often out-of-date, irrelevant, or damaged.
56. A. freshly B. recently C. frequently D. newly
57. A. though B. with C. during D. when
58. A. that B. who C. which D. this
59. A. Nevertheless B. Therefore C. Consequently D. Otherwise
60. A. fine B. fee C. cost D. sum
61. A. go B. come C. rely D. stay
62. A. educational B. economical C. political D. financial
63. A. for B. with C. on D. by
64. A. country B. converse C. contrast D. conflict
Page 44
65. A. attain B. obtain C. contain D. maintain

III. Read the passage and choose the correct answer A, B, C or D(10 points).
Before the mid-nineteenth century, people in the United States ate most foods only in season.
Drying, smoking and salting could preserve meat for a short time, but the availability of fresh meat, like
that of fresh milk, was very limited; there was no way to prevent spoilage. But in 1810, a French
inventor named Nicolas Appert developed the cooking-and-sealing process of canning. And in the
1850’s an American named Gail Borden developed a means of condensing and preserving milk. Canned
goods and condensed milk became more common during the 1860’s, but supplies remained low because
cans had to be made by hand. By 1880, however, inventors had fashioned stamping and soldering
machines that mass-produced cans from tinplate. Suddenly all kinds of food could be preserved and
bought at all times of the year.
Other trends and inventions had also helped make it possible for Americans to vary their daily diets.
Growing urban population created demand that encouraged fruit and vegetable farmers to raise more
produce. Railroad refrigerator cars enabled growers and meat packers to ship perishables great distances
and to preserve them for longer periods. Thus, by the 1890’s, northern city dwellers could enjoy
southern and western strawberries, grapes, and tomatoes, previously available for a month at most, for
up to six months of the year. In addition, increased use of iceboxes enabled families to store perishables.
As easy means of producing ice commercially had been invented in the 1870’s, and by 1900 the nation
had more than two thousand commercial ice plants, most of which made home deliveries. The icebox
became a fixture in most homes and remained so until the mechanized refrigerator replaced it in the
1920’s and 1930’s.
Almost everyone now had a more diversified diet. Some people continued to eat mainly foods that
were heavily in starches or carbohydrates, and not everyone could afford meat. Nevertheless, many
families could take advantage of previously unavailable fruits, vegetables, and dairy products to achieve
more varied fare.
66. What does the passage mainly discuss?
A. Causes of food spoilage
B. Commercial production of ice
C. Population movements in the nineteenth century
D. Inventions that led to changes in the American diet
67. The phrase “in season” in line 1 refers to
A. a particular time of year B. a kind of weather
C. an official schedule D. a method of flavoring
68. During the 1860’s, canned food products were
A. unavailable in rural areas B. available in limited quantities
C. shipped in refrigerator cars D. a staple part of the American diet.
69. It can be inferred that railroad refrigerator cars came into use
A. before 1860 B. before 1890 C. after 1900 D. after 1920
70. The word” them” in line 13 refers to
A. refrigerator cars B.growers C. perishables D. distances.
71. The word” fixture” in line 18 is closest in meaning to
A. commonplace object B. substance C. luxury item D. mechanical device
72.The author implies that in the 1920’s and 1930’s home deliveries of ice
A. increased in cost B. occurred only in the summer
C. decreased in number D. were on an irregular schedule
73. The word “ Nevertheless” in line 21 is closest meaning to
A. occasionally B. however C. therefore D. because
74. Which of the following types of food preservation was NOT mentioned in the passage?
A. Drying B. Chemical additives C. Canning D. Cold storage
75. Which of the following statements is supported by the passage?
A. Most farmers in the United States raised only fruits and vegetables.
B. People who lived in cities demanded home delivery of foods.
C. Tin cans and iceboxes helped to make many foods more widely available.
D. Commercial ice factories were developed by railroad owners
E. Writing
Page 45
I. Finish each of the following sentences in such a way that it means exactly the same as the
sentences printed before. (5 points)
76. Having nothing else to do, we decided to go for a walk.
- Since.............................................................................................................................
77. She was not only bad-tempered but also very lazy.
- As well ........................................................................................................................
78. “ Nothing will persuade me to apply for that kind of job.” She said.
_ She flatly.......................................................................................................................
79. The school I studied at last year was better than this one.
- This school isn’t.............................................................................................................
80. You must leave now, or you’ll miss the bus.
- You’ll miss......................................................................................................................
II. Use the suggested words and phrases to write complete sentences of a letter. (10 points)
Dear Sir or Madam.
81. I/ write/ complain/ hair drier/ buy/ your shop/ last Saturday/ and/ treatment/ I receive/ when/ I try/
return/ a few days later.
............................................................................................................................................ ...........................
.................................
............................................................................................................................................ ...........................
.................................
82.I buy/ hair drier/ Wednesday, November 22nd.
............................................................................................................................................ ...........................
.................................
83. first time/ try/ use/ handle become/ extremely hot/ and within a few minutes/ part/ plastic casing/
begin/ melt.
............................................................................................................................................ ...........................
.................................
............................................................................................................................................ ...........................
.................................
84. I turn/ off/ immediately/ return/ with/ receipt/ your shop/ Saturday.
............................................................................................................................................ ...........................
.................................
85. I explain/ situation/ one/ assistants/ ask/ money back/ but / be told/ speak/ you.
............................................................................................................................................ ...........................
.................................
............................................................................................................................................ ...........................
.................................
86. Unfortunately you/ not available/ that day/ I/ write instead.
............................................................................................................................................ ...........................
.................................
87. I enclose/ hair drier/ copy of/ original receipt.
............................................................................................................................................ ...........................
.................................
88. Please send/ full fund/ soon/ possible.
............................................................................................................................................ ...........................
.................................
Yours faithfully,

ENGLISH PRACTICE 12
PART I: PHONETICS. (1, 0 POINT).
Choose the word whose stress pattern is different from that of the other words in the same group.
Write your answer in the numbered box.
1. A. exposure B. Terminal C. utterance D. discipline
2. A. Economy B. Elaborate C. Assassinate D. Fascinate
3. A. mountain B. sustain C. contain D. retain
4. A. fulltime B. farmhand C. bookshop D. tradesman
Page 46
5. A. Metropolitan B. Entrepreneurial C. Hippopotamus D. curiosity
6. A. company B. comfortable C. together D. business
7. A. Associate B. Formal C. Movement D. militant
8. A. Important B. Cigarette3 C. Protection2 D. informal
9. A. introduce B. conversation C. independent D. welcome
10. A. expect B. alone C. liquor D. invite
PART II: VOCABULARY AND GRAMMAR. (8, 0 POINTS).
I. Choose the best answer to fill in each gap. Write your answer in the numbered box. (1, 5 point).
1. In the ___of security, personnel must wear their identity badges at all times.
A. requirement B. interests C. demands D. assistance
2. ___ how angry he was he would never resort to violence.
A. No matter B. No problem C. Although D. Because
3. We played the game ___ the rules.
A. on account of B. ahead of C. according to D. apart from
4. ___ to get through to Jackie for days now. Either she’s away or her phone’s out of order.
A. I’ve been trying B. I had tried C. I’m trying D. I tried
5. This is ___ the most difficult job I have ever tackled.
A. by rights B. by all means C. by far D. by the way
6. She ___till the early hours listening to pop music.
A. took me up B. kept me up C. caught me up D. held me up
7. Please don’t ___yourself out. A sandwich will do.
A. let B. put C. leave D. take
8. The chairman requested that ___
A. the members studied more carefully the problem.
B. the problem was more carefully studied.
C. with more carefulness the problem could be studied.
D. the members study the problem more carefully.
9. Smoking is ___ in many companies in our country.
A. permitted B. taught C. banned D. stopped
10. After the battle, the ___ soldiers were helped by those who could walk.
A. injured B. wounded C. broken D. killed
11. I don’t think this strange new fashion will …………
A. turn up B. care for C. show off D. catch on
12. Their house is ___ near the Cathedral.
A. whereabouts B. anywhere C. somewhere D. any place
13. This young tree could not have been damaged by accident. I believe it was done ___
A. in fact B. on purpose C. by appointment D. by plan
14. There was hardly ___ money left in my bank account.
A. more B. no C. some D. any
15. It is wrongly believed that natural resources will never be used ___.
A. off B. out C. away D. up

II. Give the correct tenses of the verbs in brackets. Write your answer in the box. (1,5 point)
1. If he __ (1. not drink) too much last night, he __ (2. not - be) tired now.
2. I’m sure he must ___ (3. be) at home last night because the door was open when I _ (4. come).
3. At this time next week, they _(5. sit) in the train on their way to Paris.
4. It’s raining. I would rather you ___ (6. close) the window.
5. I don’t know what’s the matter with him. He ___ funny since you __ (8. be)away.
6. On the first of next month, he __ (9. be) in prison for five years.
7. His personal problems seem __ (10. distract) him from his work lately.
8. He suggested that a final decision ___ (11. make).
9. I don’t know why you __ (12. always - make) noise in class, Tom.
10. __ (13. you/visit) many museums when you were in Paris?
11. The car looks very clean. ___ (14. you/wash) it?
12. My best friend, James, was no longer there. He ___ (15. go) away.

Page 47
III. Put the correct form of the words in brackets. Write your answer in the box. (2,0 points)
AUTUMN COLORS
A new term is rapidly gaining recognition in the American language - a 'leaf peeper' is someone
who, in autumn, is on the ___ (1. LOOK) for areas where the leaves of deciduous trees have changed
colour. In New England, in the USA, 'leaf peeping' is big business, generating millions of dollars
annually. The first report that leaves are changing colour sets off an ___ (2. INVADE) of 'peepers' thus
causing serious obstruction on some roads. Thousands of  people log on to websites in their __ (3.
EAGER) to find the location of the trees that have foliage in the most ___ (4. GLORY)colours. The
popularity of 'leaf tourism' is well established in New England, and the changing colours of autumn
provide an annual __ (5. TALK) point. The colours vary from year to year since the __ (6. INTENSE)
of the colour is __(7. DEPEND) on the chemical composition of the dying leaves. After a long dry
summer, leaves often turn bright red, while cloudy autumn days will produce less spectacular yellows. If
climate change leads to __ (8. INCREASE) hot, dry summers in the northern hemisphere, then countries
in Europe can expect summers not __ (9. LIKE) those across the Atlantic. They will then enjoy a
kaleidoscope of ___ (10. SEASON)colour to rival the deep reds and blazing oranges seen in New
England.
IV. Find out and correct the mistake in each sentence. Write your answer in the box.(2,0 points)
Example: Thirty hours a week are a heavy work schedule.
1. A persimmon tastes best when it is such ripe that it looks wrinkled and almost spoiled.
2. American pioneers did water systems from logs with holes bored through theircenters.
3. The pituitary gland is a small endocrine gland at the base of the brain that releases many hormones
and regulates another endocrine glands.
4. In America, the Indians used crude oil for fuel and medicine hundreds of years before the first white
settlers arrive.
5. When radio programs became popular, approximately around 1925, many people stopped attending
movies.
6. Musical comedies, as an American form of entertainment, often take its subjects from America’s
present or past.
7. Of all seashore plants, seaweeds are best able to tolerate long periods out of water, followed bylong
periods coveringby water.
8. The fruit of the plantain looks much like a banana, and it is not so sweet or so pleasing in flavor.
9. The viceroy butterfly, an insect that birds like to eat, has a color pattern similar to that of the monarch
butterfly, whom birds do not like to eat.
10. Behavior therapy uses rewards and punishments to encourage patients to act in a way healthier.
V. Complete these sentences with proper prepositions or adverbs. Write your answers in the
numbered box.
1. Don't use pencils. Please write the letter ____________ ink.
2. My father made ______________ his mind to settle in the South.
3. Minh couldn’t start his motor engine. I think it ran __________fuel.
4. Time is off now. Please hand ______________ your papers.
5. We invited 40 people to the party but only 23 turned __________
6. That fish of yours has been in the fridge for weeks. It must have gone _by now.
7. He needed a model, someone to look _____________ _
8. Uncle Tom is an idiot, but I only have to put ____________ him once a year.
9. I don’t like to make friends with the person who always runs ____________ his old friends.
10. These two men’s farms are adjacent ___________ each other.
PART III: READING COMPREHENSION. (6,0 POINTS)
I. Read the passage below and decide which answer (A, B, C or D) best fits each gap.
It is now extremely popular to take a gap year between school and university or university and
work and to spend it traveling. There are plenty of reasons to recommend it - travel broadens the mind,
you’re (1) ___________young once, life isn’t a rehearsal and so on. And if you don’t do it, you may
always regret that you didn’t take the (2) __________In the end, there’s only one response: well, why
not?

Page 48
The idea may have its roots in the 18th century Grand tour once (3) ____________ by the young,
rich and noble, but it is the middle classes who have turned it (4) __________ something that 200,000
British youngsters do every year. (5) ____________ has never been so easy and cheap, with more places
open to tourists than ever. Also, the gap year is now (6) ____________ by many employers and
universities.
The States, the Far East and Australia were among the original (7) _________ _and although
these remain in the top five, young explorers are now going even further. The most far-flung corners of
the world are (8) ____________ in popularity year by year. About $700 will buy a student ticket (9)
___________ for six months that will take you from London to Calcutta, Singapore, Bangkok, Perth,
Sydney, Auckland, Fiji, Tahiti, Los Angeles and (10) ____________ again.
1. A. merely B. only C. slightly D. simply
2. A. occasion B. moment C. chance D. luck
3. A. undertaken B. gone C. done D. given
4. A. up B. out C. over D. into
5. A. Voyage B. Travel C. Excursion D. Tour
6. A. received B. stood C. accepted D. admitted
7. A. destinations B. endings C. landings D. terminals
8. A. spreading B. expanding C. enlarging D. growing
9. A. genuine B. valid C. effective D. legal
10. A. now B. here C. back D. then
II. Read the passage and fill in each blank with ONE suitable word. Write your answer in the
numbered box.(2,0 Ginseng is one of the great mysteries of the east. Often referred to as the “elixir of
life”, its widespread use in oriental medicine has led to many myths and legends building up around this
remarkable plant. Ginseng has featured (1) _ __ an active ingredient in oriental medical literature for
over 5,000 years. Its beneficial effects were, at one time, (2) __ widely recognized and praised that the
root was said to be worth its weight in gold.
(3) _the long history of ginseng, no one fully knows how it works. The active part of the _ (4) is
the root. Its full name is Panax Ginseng – the word Panax, (5) _the word panacea, coming from the
Greek for “all healing”. There is growing interest by western scientists in the study of ginseng. It is
today believed that this remarkable plant may (6) ___ beneficial effects in the treatment of many
diseases which are difficult to treat with synthetic drug.
Today, ginseng is (7) __ longer a myth or a legend. Throughout the world it is becoming widely
recognized that this ancient herb holds the answer to relieving the stresses and ailments of modern
living. It is widely used for the treatment of various ailments (8) __ as arthritis, diabetes, insomnia,
hepatitis and anemia. However, the truth behind (9) __ ginseng works still remains a mystery. Yet its
widespread effectiveness shows that the remarkable properties are (10) ___ than just a legend.
III. Read the passage, then choose the correct options. Write your answer in the box. (2,0 points)
One of the 7 wonders of the ancient world, the great pyramid of Giza was a monument of
wisdom and prophecy built as a tomb for Pharaoh Cheop in 1720 BC. Despite its antiquity, certain
aspects of this construction makes it truly one of the truly great wonders of the world. The thirteen – acre
structure near the Nile river is a solid mass of stone blocks covered with limestone. Inside are a number
of hidden passageways and the burial chamber for the pharaoh. It is the largest single structure in the
world. The 4 sides of the pyramid are aligned almost exactly on true north, south, east and west – an
incredible engineering feat. The ancient Egyptians were sun worshipers and great astronomers, so
computations for the great pyramid were based on astronomical observations.
Explorations and detailed examinations of the base of the structure reveal many intersecting
lines. Further scientific study indicates that these represent type of timeline of events – past, present and
future. Many of the events have been interpreted and found to coincide with known facts of the past.
Others are prophesied for future generations and currently under investigation. Many believe that
pyramids have supernatural powers, and this one is no exception. Some researchers even associate with
its extraterrestrial being of the ancient past.
Was it superstructure made by ordinary beings, or one built by a race far superior to any known today.
1. What has research of the base reveal?
A. There are racks in the foundation.
B. Tomb robbers have stolen pharaoh’s body.
C. The lines represent important events.
Page 49
D. A superior race of people built it.
2. Extraterrestrial beings are ____
A. very strong workers. B. astronomers in the ancient times.
C. researchers in Egyptology. D. living beings from other planets.
3. What was the most probable reason for providing so many hidden passages?
A. To allow the weight of the pyramid to settle evenly.
B. To permit the high priests to pray at night.
C. To enable the pharaoh’s family to bring food for his journey to the afterlife.
D. To keep grave robbers from finding the tomb and the treasure buried with the pharaoh.
4. The word “intersecting” in line 9 is nearest in meaning to ____
A. crossing B. coming C. observing D. cutting
5. What do the intersecting lines in the base symbolize?
A. Architect’s plans for the hidden passage. B. Pathways of the great solar body.
C. Astrological computation. D. Dates of important events taking place throughout time.
6. The word “prophesied” in line 12 is closest in meaning to ____
A. said B. armed C. terminated D. foretold
7. What is the best title for the passage?
A. Symbolism of the Great pyramid.
B. Problems with the construction of great pyramid.
C. Wonders of the Great pyramid of Giza.
D. Exploration of the burial chamber of Cheop.
8. On what did the ancient Egyptians based their calculation?
A. Observation of the celestial bodies. B. Advanced technology.
C. Advanced tools of measurements. D. Knowledge of the earth’s surface.
9. Why was the Great pyramid constructed?
A. As a solar observatory. B. As a religious temple.
C. As a tomb for the pharaoh. D. As an engineering feat.
10. Why is the Great pyramid of Giza considered one of the seven wonders of the world?
A. It is perfectly aligned with the 4 cardinal points of the compass and contains many prophecies.
B. It was selected as the tomb of the pharaoh Cheop.
C. It was built a super race.
D. It is very old.
PART IV: WRITING. (5,0 POINTS)
I. Rewrite each of the sentences without changing its meaning, using the cue given. (1,0 point)
1. We couldn’t have managed without my father’s money.
 If it hadn’t ..............................................................................................................................
2. House prices have risen dramatically this year.
 There has .... ........................................................................................................................
3. The only thing they didn’t steal was the television.
 They stole ........ ...................................................................................................................
4. He didn’t succeeded in searching for the stolen car.
 He tried in vain ...... ............................................................................................................
5. She started to clean up the house just after the guests had left.
 No sooner ..... ........................................................................................................................
6. The boy was about to cry when he was reprimanded by his mother.
→ The boy was on ..... .................................................................................................................
7. People believe that the Chinese invented paper in 105 A.D.
→ Paper....... ..................................................................................................................................
8. As people use a lot of wood-pulp, many trees are cut down.
→ The more ........ .........................................................................................................................
9. It takes six hours to drive from here to London.
→ It is ........ .....................................................................................................................................
10. He got so angry that no one dared to say anything.
→ So ... .................................................................................................................................................
II. Rewrite each of the sentences without changing its meanings, using the word given. (1,0 point)

Page 50
1. If you want to see me, come here by six a.m. (PROVIDED)
........................................................................................................................................................................
..................................
2. He won’t let anyone touch his records. (OBJECTS)
........................................................................................................................................................................
..................................
3. He didn’t think much of the musical show yesterday. (OPINION)
........................................................................................................................................................................
..................................
4. The concert was not as good as he had hoped. (EXPECTATIONS)
........................................................................................................................................................................
..................................
5. We missed the beginning of the concert because we had overslept. (CONSEQUENCE)
........................................................................................................................................................................
..................................
III.Write an essay (3, 0 points).
Face book dominates the free-time for too many people especially the students. It can have
negative effect on their study and the physical development.
Do you agree or disagree? Write an essay (250 - 300 words) to express your opinion.
Write an essay (250 - 300 words) to express your opinion.(3, 0 points).
ENGLISH PRACTICE 13
A. PHONETICS
Choose the word whose main stress pattern is different from the rest in each of the following
questions
1. A. career B. schoolgirl C. machine D. request
2. A. deposit B. complexion C. difference D. decision
3. A. tuition B. symbolize C. etiquette D. interest
4. A. semester B. acceptance C. procedure D. institute
5. A. industry B. comprehend C . confident D. socialize
B. VOCABULARY AND GRAMMAR (20 pts)
I. Choose the best option to complete each of the following sentences.
1. To meet this __________, we have brought 3 ships into regular operation.
A. demand B. success C. appeal D. situation
2. I can’t remember his name, but it’s on the tip of my _______________.
A. tongue B. mouth C. figure D. head
3. The discovery was a major ______________for research workers.
A. breakthrough B. breakdown C. breakout D. breakup
4. This is not the right ___________to ask for my help; I’m far too busy even to listen.
A. moment B. situation C. opportunity D. circumstance
5. He drives so quickly that I am afraid that one day he wll___________someone crossing the road.
A. crash down B. knock down C. turn over D. run across
6. When the electricity failed, he ____________a match to find the candles.
A. rubbed B. struck C. scratched D. started
7. The safety committee’s report recommend that all the medicine should be kept out of the ________of
children.
A. reach B. grasp C. hand D. hold
8. I __________part in the last competition if I__________younger.
A. would have taken/would be B. would take/were
C. would have taken/would have been D. would have taken/ were
9. It was desirable that we ____________at dawn.
A. started B. had started C. start D. would start
10. Thousand dollars ____________wasted by him in the casino.
A. was B. were C. are D. has
11. I’m going to stay here for___________few days.
A. another B. other C. others D. the other
12. “Can’t you read?” Mary said, _____________
Page 51
A. and angrily pointing to the notice. B. pointing angrily to the notice.
C. angrily pointed to the notice. D. and pointed angrily the notice
13. There has to be this starting and stopping movement __________the eye can only see only when it is
not moving.
A. because B. although C. when D. while
14. “________to Japan?” – “Yes, I _________there the year when there was an earthquake.”
A. Were you ever/was B. Have you ever been/ have been
C. Were you ever/have been D. Have you ever been /was
15. When the tenants failed to pay their bill, the authorities decided to cut ________the gas supply to the
flat.
A. down B. out C. across D. off
II. Some of the lines in the passage are correct, and some have a word that needs correction. If a line
is correct, put a tick (). If a line has a word that needs correction, write the word and its correction.
There are two examples at the beginning ( 0 and 00).
Women are experts at gossiping, and they always talk about trivial 0. _____
things, or at least that’s which men have been always thought. 00. ____
However, some new research suggests that when women talk to 1.______
women, whose conversations are far from frivolous, and cover many 2.______
more topics (up to 40 subjects) than when men talk to other men. 3.______
Women’s conversations ranged from health to their houses, from 4.______
politics to fashion, from films to family, from education to 5.______
relationship problems. Football is notable absent. Men tend to have a 6.______
more limited range of subjects, the most popular being work, sport, 7.______
jokes, cars, and women.

According to Professor Petra Boynton, a psychology at University 8.______


College London, which interviewed over 1000 women, women also 9.______
tend to move quickly from one subject to another in conversation, 10._____
whereas men usually stick about one subject for longer periods of 11._____
time.

At work, this difference can be an advantage for men, where they can 12._____
put another matters aside and concentrate fully on the topic being 13._____
discussed. In the other hand, it also means that they sometimes find it 14._____
hard to concentrate when several things have to be discussed at the 15._____
same time in a meeting.
0. 
00. which=> what
III. Supply the correct forms of the words in the CAPITAL letters
“Culture shock” describes the impact of ______(1) from a familiar MOVE
culture to one which is ________( 2). It is an experience described by FAMILIAR
people who have traveled abroad to work, live or study; it can be felt to a
certain extent even when abroad on holiday. It can ______(3) anyone, EFFECT
including _______(4) students. It includes the shock of a new NATION
environment, meeting lots of new people and learning the ways of a
_______(5) country. It also includes the shock of being separated from the DIFFER
______(6) people in your life, maybe family, friends, colleagues, IMPORT
teachers: people you would normally talk to at times of _____(7), people CERTAIN
who give you support and guidance. When familiar ______(8) , sounds, SEE
smells or tastes are no longer there you can miss them very much. If you
are tired and ______(9) when you arrive small things can be upsetting and JET-LAG
out of all proportion to their real __________(10). SIGNIFY
C. READING
I. Read the text and think of the word that best fits each space. (ONE word for each space)

Page 52
About 50 years ______(1) now, it is possible to use virtual reality to bring the_____(2) back to life,
people will drag out _____(3) old and boring movies and choose a relative _____(4) bring back to life.
The visual will be combined ______(5) records about that person to create a virtual relative.
Then the family can _____(6) a chat with that relative, _____(7) it running, and live with it. The
experience will be_____(8) living with a ghost. People who died long ago will be resurrected,____(9)
they can take on new _____(10).
II. Read the passage and choose the best heading in the box for each paragraph. There are three
extra ones that should not be used.

A. How to maximize the SPF of sunscreens?


B. Summer sport and skin protection
C. How the various types of radiation differ?
D. Sunscreens: the higher the SPF the better?
E. What is ultraviolet radiation?
F. Other ways to protect yourself
G. What are the first signs of skin cancer?
H. A short break in the sun won’t hurt, will it?
1.___________
Sunshine contains three different bans of ultraviolet radiation: UVA, UVB and UVC. Although UVC is
the most dangerous, because it is a shorter-wavelength radiation than UVA and UVB, it is screened out
by the Earth’s ozone layer. UVA used to be thought less dangerous than UVB, but it is now known that
both bands can cause skin cancer. It is UVB which causes sunburn. However, both UVA and UVB can
age the skin prematurely.
2.___________
Levels of UV rays can vary. A two-week holiday in the Mediterranean will expose you to the same
amount of sun as you would get in a year in Britain. Short periods of intense exposure to the sun are
thought to be more risky than regular daily exposure, particularly if you have fairer skin. However, even
if you have darker skin tones you will burn eventually. You can find out the daily UV rate by watching
the solar UV index which has recently been introduced on national weather forecasts across Europe.
3.___________
The ideal sunscreen to use is an SPF15. This means a sunscreen which gives you fifteen times more
protection that you would have normally. An SPF15 sunscreen will absorb proportionally equal amounts
of UVA and UVB, and will give you good protection if you are sensible about your exposure.
Sunscreens higher than SPF15 tend to lose their balanced effect: the chemicals in an SPF30, for
example, will not block UVA rays as effectively as UVB. The other danger with high SPFs is that
people will stay in the sun longer because they think are better protected. Higher SPFs do not give
proportionately greater protection. An SPF15 gives 93 percent protection, for example, while an SPF34
gives 97 percent protection.
4.____________
Only UVA rays can pass through glass, so you won’t get sunburnt sitting by a window, while you can
still enjoy the warmth of the sun. Be careful when sitting in the shade, however. You can still get burnt
because you will be exposed to rays bouncing off reflective surfaces nearby. This is particularly the case
near water. Not all clothing offers effective protection, either. If you can see the light through a piece of
clothing when you hold it up, it will not offer much of a barrier to UV rays.
5.____________
You should keep an eye out for any moles or dark spots on the skin that change in size, shape, or color,
become bigger, itchy or inflamed, or bleed. All these may be symptoms of skin cancer and should be
checked by a doctor. Once a mole has been identified as a potential melanoma, it is removed under local
anaesthetic and sent for examination. Most turn out to be harmless. Of the three forms of skin cancer the
two most common varieties – basal cell and squamous cell carcinomas are easily treatable and rarely
fatal, and even melanomas can be treated effectively if caught in time.
III. Read the following passages and choose the best answer to each question.
Scientists do not yet thoroughly understand just how the body of an individual becomes sensitive to a
substance that is harmless or even wholesome for the average person. Milk, wheat, and egg, for
Page 53
example, rank among the most healthful and widely used foods. Yet these foods can cause persons
sensitive to them to suffer greatly. At first, the body of the individual is not harmed by coming into
contact with the substance. After a varying interval of time, usually longer than a few weeks, the body
becomes sensitive to it, and an allergy has begun to develop. Sometimes it's hard to figure out if you
have a food allergy, since it can show up so many different ways. Your symptoms could be caused by
many other problems. You may have rashes, hives, joint pains mimicking arthritis, headaches,
irritability, or depression. The most common food allergies are to milk, eggs, seafood, wheat, nuts,
seeds, chocolate, oranges, and tomatoes. Many of these allergies will not develop if these foods are not
fed to an infant until her or his intestines mature at around seven months. Breast milk also tends to be
protective. Migraines can be set off by foods containing tyramine, phenathylamine, monosodium
glutamate, or sodium nitrate. Common foods which contain these are chocolate, aged cheeses, sour
cream, red wine, pickled herring, chicken livers, avocados, ripe bananas, cured meats, many Oriental
and prepared foods (read the labels!). Some people have been successful in treating their migraines with
supplements of B-vitamins, particularly B6 and niacin. Children who are hyperactive may benefit from
eliminating food additives, especially colorings, and foods high in salicylates from their diets. A few of
these are almonds, green peppers, peaches, tea, grapes. This is the diet made popular by Benjamin
Feingold, who has written the book Why your Child is Hyperactive. Other researchers have had mixed
results when testing whether the diet is effective.
1. The topic of this passage is 
A. reactions to foods B. food and nutrition
C. infants and allergies D. a good diet
2. According to the passage, the difficulty in diagnosing allergies to foods is due to
A. the vast number of different foods we eat
B. lack of a proper treatment plan
C. the similarity of symptoms of the allergy to other problems
D. the use of prepared formula to feed babies
3. The word "symptoms" is closest in meaning to
A. indications B. diet C. diagnosis D. prescriptions
4. The phrase "set off" is closest in meaning to
A. relieved B. identified C. avoided D. triggered
5. What can be inferred about babies from this passage?
A. They can eat almost anything.
B. They should have a carefully restricted diet as infants.
C. They gain little benefit from being breast fed.
D. They may become hyperactive if fed solid food too early.
6. The word "hyperactive" is closest in meaning to 
A. overly active B. unusually low activity
C. excited D. inquisitive
7. The author states that the reason that infants need to avoid certain foods related to allergies has to do
with the infant's
A. lack of teeth B. poor metabolism
C. underdeveloped intestinal tract D. inability to swallow solid foods
8. The word "these" refers to
A. food additives B. food colorings
C. unnutritious foods D. foods high in salicylates
9. Which of the following was a suggested treatment for migraines in the passage?
A. Eating more ripe bananas
B. Avoiding all Oriental foods
C. Getting plenty of sodium nitrate
D. Using Vitamin B in addition to a good diet
10. According to the article the Feingold diet is NOT
A. verified by researchers as being consistently effective
B. available in book form
C. beneficial for hyperactive children
D. designed to eliminate foods containing certain food additives
D. WRITING
Page 54
I. Rewrite the following sentences, using key words in the parentheses or the suggested phrases given
at the beginning of the sentences
1. Jack has become confident as a result of his success. (turned)
Jack’s success______________________________person.
2. To find new fuels it is necessary to spend a lot of money.( means)
Finding_____________________________________money.
3. Dan promised sincerely that he wouldn’t watch so much TV. (sincere)
Dan________________________________________not to watch so much TV.
4. Someone almost certainly broke the window on purpose. (must)
The window ________________________on purpose.
5. We were all surprised when she suddenly left the job. (found)
All__________________________________left the job.
6. The cost of living has gone up considerably in the last few years.
There________________________________________________
7. “No one can persuade me to sleep in that haunted house,” she said.
She flatly_____________________________________________.
8. If we can solve the problem soon, it will be better for all concerned.
The sooner____________________________________________.
9. We must continue our efforts, whether there are problems or not.
Regardless ___________________________________________.
10. “ Less noise!” the teacher told the boys.
The teacher shouted_______________.
II. The social networks such as Facebook or Twitter are very popular among teenagers. What are
their advantages? Give examples and reasons to support your opinion.
Write a paragraph of about 200 words.

Page 55
ENGLISH PRACTICE 14
PART I – PRONUNCIATION
Choose the word whose stress pattern is different from the rest
1. A. difficult B. corporate C. different D. engineer
2. A. demolish B. substantial C. dramatic D. terrorist
3. A. proficiency B. electronic C. petroleum D. equivalent
4. A. vineyard B. ignite C. business D. Finland
5. A. change B. image C. danger D. oasis
6. A. reference B. interview C. government D. understand
7. A. scholarship B. develop C. equipment D. discourage
8. A. expectation B. temperament C. opportunity D. decoration
9. A. religious B. miserable C. performance D. including
10. A. decision B. personal C. interest D. wonderful

PART II - LEXICO-GRAMMAR
I. Complete each of the following sentences with the correct answer (A, B, C or D). Identify your answer
by writing the corresponding letter A, B, C or D on your answer sheet.
1. Nathalie seems very tough at work. She’s a different person at home, _________.
A. though B. although C. as though D. even though
2. I kept out of the conversation because it _________ me.
A. wasn’t concerned B. wasn’t concerning C. didn’t concern D. didn’t concern to
3. The entire city was _________ electricity last night- it was chaotic.
A. no B. almost no C. hardly any D. without
4. Henry was overweight, so he went on s strict diet and _________ twenty kilos.
A. missed B. lost C. failed D. fell
5. He was arrested because he answered to the description of the _________ man.
A. searched B. pursued C. wanted D. hunted
6. Humanity has done great damage to the environment in its search for _________ materials.
A. live B. raw C. crude D. rude
7. _________, the balcony chairs will be ruined in this weather.
A. Leaving uncovered B. Having left uncovered
C. Left uncovered D. Been left uncovered
8. One way to let off _________ after a stressful day is to take some vigorous exercise.
A. cloud B. tension C. steam D. sweat
9. Their research into the causes of cancer promises to break the new _________ in the field and
possibly lead to a cure.
A. earth B. ground C. soil D. land
10. After three days in the desert, his mind began to play _________ on him.
A. games B. jokes C. tricks D. fun
11. The match will be screened on ITV with _________ commentary by Any Gray.
A. lively B. live C. alive D. living
12. I know you didn’t want to upset me but I’ sooner you _________ me the whole truth yesterday.
A. could have told B. told C. have told D. had told
13. As the drug took _________, the patient became quieter.
A. effect B. force C. influence D. action
14. The dawn redwood appears ____ some 100 million years ago in northern forests around the
world.
A. was flourished B. having to flourish
C. to have flourished D. have flourished
15. His comments _________ little or no relation to the facts and the figures of the case.
A. reflect B. bear C. give D. possess
16. All _____ is a continuous supply of the basic necessities of life.
Page 56
A. what is needed B. for our needs C. the thing needed D. that is needed
17. It is urgent that this letter _____ immediately.
A. was posted B. posted C. be posted D. be post
18. John: This grammar test is the hardest one we’ve ever had this semester!
Mary: _____ but I think it’s quite easy.
A. I couldn’t agree more B. I understand what you’re saying
C. You’re wrong D. I don’t see in that way
19. It is only recently that ballets have been based on the themes _____ American life.
A. reflecting B. reflects C. is reflecting D. reflected
20. I wish you’d do the accounts. I don’t have ________ for numbers.
A. a head B. a mind C. the heart D. the nerve
II. Supply the correct form of the word in capital letter. Write your answers on your answer sheet.
SKIING HOLIDAYS IN COLORADO
To ski or snowboard in Colorado is to experience the pinnacle of winter sports. The state of
Colorado is known for its spectacular scenery and (1. BREATH) ____ views, which inspire today's
travelers as much as they spurred on the (2. SETTLE) ____ who first arrived in this part of the US over
a century ago. And whether you're seeking the outdoor adventure of a (3. LIFE) ____ exciting nightlife
or a great family getaway, Colorado has everything you need.
November through April, snow conditions are (4. CONSIST) ____ and reliable, featuring
Colorado's (5. LEGEND) ____ “champagne powder” snow. Extensive snow making and grooming
operations always keep trails in top shape.
The mountain destinations in the Colorado Rockies can turn your wildest ski dreams into
thrilling (6. REAL) ______.There, you'll find the best skiing and snowboarding on (7. PICTURE)
____ slopes, as well as the finest ski schools in the US. Together, they present an (8. PARALLEL) ____
winter paradise. And the best part is that you'll enjoy friendly, (9. CARE) ____ service in resorts that are
(10. COMMIT) ____ to delivering the highest quality amenities.
III. Complete the following sentences with one appropriate preposition for each blank.
1. The government’s plans to reduce crime came ________for a lot of criticism from freedom
groups.
2. I was ________the impression that you like Indian food.
3. This is one of the exceptions ________the rule.
4. The factory paid ________nearly a million pounds to their employees who were injured in the
explosion.
5. Before they open the new factory, a lot of the young people round here were ________the dole.
6. Mr. Horrid was a terrible teacher and obviously not cut ________for teaching.
7. I can’t cancel my arrangements ________such short notice.
8. All the police’s efforts to find him were ________vain.
9. The farmhouse we stayed in was completely ________the beaten track.
10. She’s gone and this time it’s ________good.

PART III: READING COMPREHENSION


I. Read the passage then choose the best answer to each question that follows. Identify your answer by
writing the corresponding letter A, B, C or D on your answer sheet.
THE ATMOSPHERE OF VENUS
Venus, also called the Morning Star and Evening Star, is the second-closest planet to the sun and
the brightest object in the night sky. The planet orbits the sun every two hundred and twenty four Earth-
days and is sometimes referred to as Earth’s sister planet because the two share both a similar size and
bulk. What is not similar, however, is Venus’s atmosphere in comparison to Earth’s atmosphere.
The atmosphere on Venus is much heavier and has a higher density than that of Earth. Venus’s
atmosphere also expands significantly higher than Earth’s atmosphere although a thick cloud cover
makes the surface of Venus nearly impossible to see unless observed through radar mapping.
While the pressure and temperature of Venus’s upper atmosphere are comparable to those of
Earth, the heat and pressure of the lower atmosphere are not unlike a furnace. Venus’s atmosphere is
Page 57
very thick due to a composition consisting mainly of carbon dioxide, and a small amount of nitrogen. If
man could survive the extreme heat of Venus’s surface (400 degrees Celsius), then he would have
to contend with a surface pressure that is more than 90 times that of Earth. Venus’s extremely high
temperature is thanks to the greenhouse effect caused by such a large amount of carbon dioxide. The
greenhouse effect is a process by which the sun’s infrared radiation is more readily absorbed by the
atmosphere. Just like in a real greenhouse used to grow plants years round, the proliferation of carbon
dioxide traps radiation and warms Venus’s atmosphere. Due to this phenomenon, Venus boasts a higher
atmospheric temperature than Mercury, even though Venus is twice the distance from the sun.
However, scientists postulate that Venus’s atmosphere was not always so hot. [A] Studies show
that large bodies of water were once on Venus’s surface but that eventually evaporation of all the water
caused the runaway greenhouse effect which regulates the planet today. [B] Thus Venus has become a
critical study for today’s scientists, as human being are only beginning to struggle with the early stages
of the greenhouse effect. [C] Our problems do not stem from evaporated water supplies but from a
propagation of carbon dioxide and other greenhouse gases due to industrial and automobile emissions.
[D]
Another interesting characteristic to note regarding Venus’s atmosphere is that its daytime
temperatures and nighttime temperatures are not that far removed from each other. This is due to the
thermal inertia, the ability of a substance to store heat despite changing temperatures and the transfer of
heat by Venus’s strong winds. Although winds on the surface of Venus move slowly in comparison with
Earth’s winds, Venus’s air is so dense that a slow-moving there can move large obstructions and even
skip stones along the planet’s surface.
In 1966, humankind made its first attempt at sending a recording instrument into Venus’s
atmosphere. The Venera 3 probe did collide with Venus surface; however, the abrupt impact caused its
communication system to fail, and it was unable to send and feedback. In 1967, Venera 4 successfully
enter Venus’s atmosphere and was able to take many readings, one of which recorded that Venus’s
atmosphere was between ninety and ninety-five percent carbon dioxide. Subsequent Venera probes were
sent into Venus’s atmosphere, but most of them succumbed to the crushing air pressure.

Questions 1-7: Choose the answer (A, B, C or D) which you think fits best according to the passage.
1. According to paragraph 1, Venus is named the Morning Star and Evening Star
because_________.
A. it is very bright B. it is close to the sun
C. it can be seen from evening till morning D. it is used to find the direction by sailors
2. The word that in paragraph 2 refers to _________.
A. size B. bulk C. atmosphere D. density
3. Which of the following best expresses the essential information in the bold sentence in paragraph
3? Incorrect answer choices change the meaning in important ways or leave out essential
information.
A. Earth experiences greater surface pressure than Venus.
B. If a man could survive its surface pressure.
C. The surface pressure and heat of Venus are much greater than those on Earth.
D. Venus’s surface temperature and pressure make it uninhabitable by humans.
4. According to paragraph 3, the greenhouse effect on Venus is owed to _________.
A. the small amounts of nitrogen
B. the rapid increasing amounts of carbon dioxide
C. growing plants
D. the high atmospheric temperatures
5. In paragraph 4, the author of the passage implies that Earth _________.
A. might suffer the same greenhouse effect as Venus
B. once had an atmosphere similar to Venus’s
C. has bodies of water similar to those on Venus today
D. is experiencing a reduction of carbon dioxide emissions
6. Look at the four blanks […] in paragraph 4 that indicate where the following sentence could be
added to the passage.
Page 58
Although the causes are different, the ramifications are the same.
Where would the sentence best fit?
7. The word propagation in paragraph 4 is closest in meaning to _________.
A. generation B. elimination C. evaporationD. desecration
Questions 8 to 10: Complete the brief summary of the passage by selecting the THREE answer
choices that express important ideas in the passage. The introductory sentence for the summary is
provided bellowed.

Scientists look at Venus to predict Earth’s future.


8. …………………………………………..…………………………………………..……
9. ………………………………………………….…………………………………………
10. ………………………………………………………………..…………………………

Answer Choices

A. Venus once had large bodies of water that elaborated and cause a rapid increase in carbon
dioxide.
B. Earth’s wind has a greater velocity than Venus’s because the air movement on Venus is denser
and can even more large obstructions.
C. Spaceships landing on Venus, though often crushed by Venus’s atmosphere, have revealed much
about its carbon dioxide filled atmosphere.
D. If man could survive the hot temperature of Venus, then he would have to contend with the great
surface pressure.
E. The first space probe of Venus was made in 1966.
F. Scientists are concerned that conditions on Earth that propagate significant quantities of carbon
dioxide will produce greenhouse effects similar to Venus’s.

II. Read the text below and think of the word which best fits each space. Use only ONE word in
each space. Write your answers on your answer sheet.
SPLENDID SPAS OF ASIA
Lying on a bed almost on the (1) ________ of a cliff, with a stupendous ocean view and the (2)
________ of waves, aches and pains are soothed away (3) ________ expert hands. Only two steps are
needed to reach the private pool, which seems to merge (4) ________ the ocean.
Such a scenario is no longer a fantasy (5) ________ an increasingly popular reality in Asia for
many stressed out businessmen and visitors from all over the world in search of that peaceful time and
space for their body and mind.
In the last four years, at (6) ________ 17 hotel spas have opened in South East Asia to (7)
________ this need. The tropical climate of the region and its reliable sunshine make for an ideal spa
setting. Picturesque environments (8) ________ with a series of rejuvenating treatments bring the
desired result. The Asian spa resorts have acquired a formidable reputation for their professional
services as (9) ________ as for the decor of their large treatment rooms. Visitors relax with Thai music
and soak in the warm tones of the room. As all these take (10) ________ in individual rooms, precious
privacy is guaranteed, a rare privilege often absent from other spas where guests share rooms or
changing areas.

III. Read the following passage and choose the best answer for each question. Identify your answer by
writing the corresponding letter A, B, C or D on your answer sheet.
WIND - The untamable weather machine
On 15 October 1987, the southern counties of Britain were struck by the strongest winds they
had experienced in 200 years. Gusts of over 130 kilometers per hour slammed across the region and
£1.5- billion-worth of damage was (1)__________ in just a few hours.
Extreme weather events like this are dramatic (2)__________ of the power of the wind. It’s one
part of the weather we generally don't give a second thought to in Britain but it plays a vital role in
Page 59
people’s lives across the world. Without the formation and circulation of winds there would quite
(3)__________ be no climate.
Some parts of the world seem to suffer more than others from the effects of ‘ill winds’, and links
between particular winds and psychological problems (4)__________ back centuries. The Föhn, the hot
dry wind that slides off the slopes of the Alps, is capable of boosting temperatures quite suddenly by
10°C or more. It affects as many as one in three people in its (5)__________, making them feel anxious,
irritable and generally ill. In California, many people (6)__________ the arrival of the Santa Ana, which
rushes down from the high Mojave desert. Lyall Watson, in his book Heaven’s Breath, claims that when
the Santa Ana blows, murder rates soar.
The wind may get into the headlines when it comes in the form of tornadoes and hurricanes, but
for the (7)__________ part it goes about its job of shifting huge masses of air around the planet. Plants
take (8)__________ of this free ride to send their pollen grains far and wide. Trees (99)__________ on it
to remove old leaves and make way for new growth. Spiders have been caught (10)__________ a lift at
altitudes of almost 4.5 kilometers.
1. A. made B. caused C. destroyed D. completed
2. A. reminders B. recollections C. mementos D. memorial
3. A. easily B. rightly C. surely D. simply
4. A. last B. originate C. pass D. date
5. A. line B. road C. path D. bypass
6. A. despair B. respect C. dread D. warn
7. A. most B. maximum C. majority D. general
8. A. benefit B. chance C. occasion D. advantage
9. A. need B. trust C. hope D. realize
10. A. traveling B. hitching C. borrowing D. making

PART IV: WRITING


I. Rewrite the following sentences without changing their meaning, using the words given. These
words must not be changed in any way.
1. I would like to be able to speak French. (HAD)
I wish ________________________________________________________ speak French.
2. It was raining cats and dogs. (TORRENTS)
The rain was ______________________________________________________________
3. It was wrong of you to borrow my book without asking. (HAVE)
You ____________________________________________before you borrowed my book.
4. When I was younger, this record was one of my favourites. (FAVOURITE)
This record used ______________________________________ mine when I was younger.
5. My sister finds commuting every day annoying. (PUT)
It’s difficult for my sister ___________________________________________ every day.
6. The Mediterranean is warm, whereas the North Sea is much colder. (NOTHING)
The North Sea is ____________________________________________ the Mediterranean.
7. Christ would only eat a pizza if he could have a mushroom topping. ON
Christ ______________________________________________ when he ate a pizza.
8. My father persuaded me to learn another foreign language. TALKED
My father ______________________________________________ another foreign language.
9. Sam tried extremely hard to convince her, but it was no use. BRING
Hard ________________________________________________________________________
10. When he arrived at the airport, his family welcomed him warmly. GIVEN
On___________________________________________________________________________
II. Write an essay to give your opinion on the following topic.
Some people believe that history has little to tell us. Other people believe that people must have
knowledge of history in order to understand the present.

Page 60
Give reasons for your answer and include any relevant examples from your own knowledge or
experience.
Write at least 250 words.
ENGLISH PRACTICE 15
PART I – PRONUNCIATION
Choose the word whose stress pattern is different from the rest
1. A. vulnerable B. conservation C. generation D. disappearance
2. A. encounter B. influence C. agency D. memory
3. A. crossbar B. advance C. goalie D. Polo
4. A. succeed B. anxious C. well-done D. reduce
5. A. studious B. semester C. similar D. century
6. A. teacher B. brochure C. achieve D. purchase
7. A. departure B. inherit C. elegant D. authentic
8. A. satisfy B. quality C. mistake D. Sequential
9. A. repetition B. television C. residential D. mountaineer
10. A. communication B. international C. university D. necessarily
PART II: LEXICO-GRAMMAR
I. Complete each of the following sentences with the correct answer (A, B, C or D). Identify your answer
by writing the corresponding letter A, B, C or D on your answer sheet.
1. Millie’s father accused her of _________.
A. trying not hard enough B. trying not hard enough
C. not trying hard enough D. trying not enough hard
2. The public library _________ to all readers who are interested in reading and doing research.
A. is opening B. is open C. is opened D. is being opened
3. I bought some new shoes. They felt a bit strange _________ because I wasn’t used to them.
A. first B. at first C. firstly D. first of all
4. I quickly packed my new belongings and spent _________ money I had on a one-way ticket
home.
A. little B. a little C. the little D. a little of
5. She believes that all countries should _________ the death penalty as it is inhumane.
A. put down to B. catch up on C. get down to D. do away with
6. Keep your ticket _________ you have to show it to an inspector.
A. if B. in case C. unless D. supposing
7. He always _________ aside some time every day to read to his children.
A. sets B. leaves C. spares D. lets
8. For the past few months she’s been as a street _________ selling fruit and vegetable.
A. dealer B. trader C. pusher D. vendor
9. I use weed-killer to _________ the weeds in the garden.
A. get rid of B. get out of C. get away with D. get in the way with
10. If there is new evidence that proves his innocence, it is likely that the authorities will _________
him from jail.
A. release B. relieve C. remove D. rehabilitate
11. The defendant’s lawyer wasn’t very good and he was found _________ by the jury.
A. faulty B. mistaken C. guilty D. sinful
12. The dish was so tasty that I asked for second _________.
A. helping B. portion C. ration D. share
13. I’m amazed that this game ever _________- it is so silly!
A. took in B. caught on C. took up D. caught by
14. “I’m sure the Whitleys were involved.” - “They _________ have since they know nothing about
the business.”
A. can’t B. wouldn’t C. shouldn’t D. mustn’t

Page 61
15. “Candy’s an excellent pianist, isn’t she?” – “She _________ to win the prize if she plays this
well during the competition.”
A. is due B. is bound C. is about D. is set
16. His friends offered to _____the next time he was in town so that he wouldn’t have to pay for a
hotel.
A. place him in B. put him up C. back him up D. turn him out
17. Susan became so tired of city life that she decided to buy a piece of land _________.
A. out of the ordinary B. as the crow flies
C. far and away D. in the middle of nowhere
18. My mind went ____ when the official asked me my phone number- I couldn’t remember it at all.
A. empty B. clear C. blank D. vacant
19. During their first date, Jane had nervously peppered the conversation with _____talk.
A. unimportant B. tiny C. small D. trivial
20. We do go _________ in the office, but the odd rule gets broken from time to time.
A. by the book B. for a song C. astray D. in phrases
II. Supply the correct FORM of the word in capital letter. Write your answers on your answer
sheet.
Before going to an interview, it is advisable to go through a mock interview. This will give you
the opportunity to try out your technique and answers live. It is also a chance to receive feedback that is
(1. BENEFIT) __________ in guiding you towards improving your interview style and general (2.
PRESENT) _________ Just one mock interview will result in a (3. NOTICE)_________ improvement
in your interview skill. Why? For the same reason that a (4. SPEAK) _________ doesn’t exist while it is
still on paper or floating in your head. It only exists when you give it (5. ORAL)_________ The first
time you give it in front of an audience, it will come out nothing like the one you prepared.
It is the same with being interviewed. It is not enough to look at a question and say, ‘Yeah, I
know the answer to that one.’ You need to practise your answer live; this is not the time to talk to
yourself in front of a mirror. Seek out a (6. PROFESSION) _________ and have the session
videotaped. Then you will have two opinions – the interview’s and your own. You will find you get a
completely different (7. IMPRESS)________ when listening to yourself than when you are watching
yourself saying something. Just as your voice always sounds different on tape, so do your (8.
RESPOND) _______. You will be glad the image is captured on tape and not in a potential employer’s
mind. For maximum effect, you should (9. VISIT)________ your answers and go through a second
mock interview. This should help with any (10. EASE)________ and give you more confidence for the
real interview.
III. There are 10 errors in the following passage. Identify and correct them. The first has
been done for you.

Page 62
Most children with (0) health appetites are ready to eat almost anything that is (0) healthy
offering them and a child rare dislikes food unless it is badly cooked. The way (1)_________
the meal is cooked and served is most important and an attractive served meal (2)_________
will often improve a child’s appetite. Never ask a child whether he likes and
(3)_________
dislikes a food and never discuss them in front of him or allow nobody else to do
so. If the father says he hates fat meat or the mother refuses vegetables under the (4)_________
child’s hearing he is likely to copy this procedure. Take it for grant that he likes (5)_________
everything and he probably will. Nothing healthful should be omitted out the (6)_________
meal because of a supposing dislike. At meal time, it is a good idea to give a (7)_________
child a small portion and let him come back for a second helping other than give (8)_________
him as much as he is likely to at all once. Do not talk too much to the child (9)_________
during the meal times. But let him get on with his food, and do not allow him to
(10)_________
leave the table immediately after a meal or he will soon learn to swallow his food
so that he can hurry back to his toys. Under no circumstance must a child be
coaxed and forced to eat.
PART III: READING
I. Read the following passage and choose the best answer for each question. Identify your answer by
writing the corresponding letter A, B, C or D on your answer sheet.
The ability to weep is a uniquely human form of emotional response. Some scientists have
suggested that human tears are (1) _____ of an aquatic past – but this does not seem very likely. We cry
from the moment we enter this world, for a number of reasons. Helpless babies cry to persuade their
parents that they are ill, hungry or uncomfortable. As they (2) _____, they will also cry just to attract
parental attention and will often stop when they get it.
The idea that having a good cry do you (3) _____ is a very old one and now it has scientific
validity since recent research into tears has shown that they (4) _____ a natural painkiller called
enkaphalin. By fighting sorrow and pain this chemical helps you feel better. Weeping can increase the
quantities of enkaphalin you (5) _____.
Unfortunately, in our society we impose restrictions upon this naturally (6) _____ activity. Because
some people still regard it as a (7) _____ of weakness in men, boys in particular are admonished when
they cry. This kind of repression can only increase stress, both emotionally and physically.
Tears of emotion also help the body (8) _____ itself of toxic chemical waste, for there is more
protein in them than in tears resulting from cold winds or other irritants. Crying comforts, calms and can
be very enjoyable – (9) _____ the popularity of highly emotional films which are commonly (10) _____
“weepies”. It seems that people enjoy crying together almost as much as laughing together.
1. A. witness B. evidence C. result D. display
2. A. evolve B. change C. develop D. alter
3. A. better B. fine C. good D. well
4. A. contain B. retain C. hold D. keep
5. A. construct B. achieve C. provide D. produce
6. A. curing B. treating C. healing D. improving
7. A. hint B. symbol C. feature D. sign
8. A. release B. rid C. loosen D. expel
9. A. consider B. remark C. distinguish D. regard
10. A. named B. entitled C. subtitled D. called
II. Read the text below and think of the word which best fits each space. Use only ONE word in
each space. Write your answers on your answer sheet.
ITALY’S MOST POETIC CITY
Venice has been an inspiration for writers, artists and musicians throughout history. In the 15th
century it was the world’s (1) _____ port. Since then it has built up an astonishing collection of art and
architecture (2) _____ to its trade with the East.
The city (3) ____ its visitors incredible sights. Do not believe those who say Venice is a
museum. This is still a living city full of joys. Venice looks good in any light. The sun makes the domes
sparkle, but even on a grey, (4) _____ day the city can be extremely romantic.

Page 63
And (5) _____ it gets overcrowded, (6) _____ is an easy escape to the other islands in the Venice
Gulf, (7) _____ brightly-colored houses are a photographer’s dream.
In a curious way, Venice is a model city for the future; it is free from cars and the (8) _____ way
to get around is by public transport or on foot. This one fact alone (9) _____ it a unique city, one (10)
_____ traffic noise, the creation of genius indeed.
III. Read the passage then choose the best answer to each question that follows. Identify your answer by
writing the corresponding letter A, B, C or D on your answer sheet.
Legend has it that sometime toward the end of the Civil War (1861-1865) a government train
carrying oxen traveling through the northern plains of eastern Wyoming was caught in a snowstorm and
had to be abandoned. The driver returned the next spring to see what had become of his cargo. Instead of
the skeletons he had expected to find, he saw his oxen, living, fat, and healthy. How had they survived?
The answer lay in a resource that unknowing Americans lands trampled underfoot in their haste to
cross the “Great American Desert” to reach lands that sometimes proved barren. In the eastern parts of
the United States, the preferred grass for forage was a cultivated plant. It grew well with enough rain,
then when cut and stored it would cure and become nourishing hay for winter feed. But in the dry
grazing lands of the West that familiar bluejoint grass was often killed by drought. To raise cattle out
there seemed risky or even hopeless.
Who could imagine a fairy-tale grass that required no rain and somehow made it possible for cattle
to feed themselves all winter? But the surprising western wild grasses did just that. They had
wonderfully convenient features that made them superior to the cultivated eastern grasses. Variously
known as buffalo grass, grama grass, or mesquite grass, not only were they immune to drought; but they
were actually preserved by the lack of summer and autumn rains. They were not juicy like the cultivated
eastern grasses, but had short, hard stems. And they did not need to be cured in a barn, but dried right
where they grew on the ground. When they dried in this way, they remained naturally sweet and
nourishing through the winter. Cattle left outdoors to fend for themselves thrived on this hay. And the
cattle themselves helped plant the fresh grass year after year for they trampled the natural seeds firmly
into the soil to be watered by the melting snows of winter and the occasional rains of spring. The dry
summer air cured them much as storing in a barn cured the cultivated grasses.
1. What does the passage mainly discuss?
A. A type of wild vegetation B. Western migration after Civil War
C. The raising of cattle D. The climate of the Western United States
2. What can be inferred by the phrase “Legend has it” in line 1?
A. Most history book include the story of the train.
B. The story of the train is similar to other ones from that time period.
C. The driver of the train invented the story. D. The story of the train may not be completed
factual.
3. The word “they” in line 4 refers to _____.
A. plains B. skeletons C. oxen D. Americans
4. What can be inferred about the “Great American Desert” mentioned in line 7?
A. Many had settled there by the 1860’s. B. It was not originally assumed to be a fertile area.
C. It was a popular place to raise cattle before the Civil War.
D. It was not discovered until the late 1800’s.
5. The word “barren” in paragraph 2 is closed in meaning to _____.
A. lonely B. uncomfortable C. infertile D. dangerous
6. The word “preferred” paragraph 2 is closed in meaning to _____.
A. favored B. available C. ordinary D. required
7. Which of the following can be inferred about the cultivated grass mentioned in the second
paragraph?
A. Cattle raised in the Western United States refused to eat it.
B. It had to be imported into the United States.
C. It would probably not grow in the western United States. D. It was difficult for cattle to digest.
8. Which of the following was NOT one of the names given to the western grasses?
A. Mesquite grass B. Bluejoint grass C. Buffalo grass D. Grama grass
9. Which of the following was NOT mentioned as a characteristic of western grasses?
Page 64
A. They contain little moisture B. They have tough stems
C. They can be grown indoors D. They are not affected by dry weather
10. According to the passage, the cattle help promote the growth of the wild grass by_____.
A. eating only small quantities of grass B. continually moving from one grazing area to
another
C. naturally fertilizing the soil D. stepping on and pressing the seeds into the ground
PART IV: WRITING
I. Rewrite the following sentences without changing their meaning, using the words given. These
words must not be changed in any way.
1. Return the product to the shop if you have any complaints about it.
Should ______________________________________________________________________
2. It’s almost nine months since I stopped subscribing to that magazine.
I cancelled ___________________________________________________________________
3. Her success went beyond her
expectation.→Never_______________________________________
4. His fondness for the game increased with his proficiency.
The more ____________________________________________________________________
5. Simon hadn’t expected that he would feel so weak after the operation.
The operation left ______________________________________________________________
6. I am sure he recognized us at the airport. →He must
______________________________________
7. The journalists only heard about the changes to the wedding plans when they arrived at the
avenue.
Not until _____________________________________________________________________
8. “Why can’t you do your work more carefully?” Helen’s boss said to her.
Helen’s boss criticized __________________________________________________________
9. Someone has suggested raising the parking fees in the city.
It ___________________________________________________________________________
10. Don’t misquote. I never said I hated ballet. → Don’t put
___________________________________
II. Write an essay (about 250 words) about the following topic:
Sometimes tourists may damage tourist sites. Explain what some of the negative effects may be. Suggest
some solutions to this problem.

ENGLISH PRACTICE 16
PART I – PRONUNCIATION
Choose the word whose stress pattern is different from the other three in the following questions
1. A. argument B. reject C. attract D. install
2. A. librarian B. religious C. commercial D. Japanese
3. A. technology B. activity C. experience D. presentation
4. A. informality B. entertainment C. situation D. appropriate
5. A. experience B. cosmetics C. economics D. fertility
6. A. preparation B. apology C. geography D. experience
7. A. surplus B. surface C. surgeon D. surprise
8. A. award B. control C. regard D. conquer
9. A. colleague B. neighbour C. career D. classmate
10. A. disappear B. precision C. mechanic D. discover
PART 2: LEXICO - GRAMMAR
Section 1: Choose the best option (A, B, C or D) to complete each of the following sentences.
1. Much of what he said had little _______ to the issue we were discussing.

Page 65
A. accordance B. involvement C. concern D. relevance
2. The restless lion kept pacing _________ along the front of its cage.
A. back and fort B. up and down C. on and off D. back and out
3. You can have _______ for the meals during your package vacation.
A. vouchers B. tags C. checks D. records
4. A new motorcycle model has just been _______ on the Internet.
A. delivered B. designed C. launched D. exploited
5. Why don’t you _______ a go? - It’s not difficult!
A. do B. have C. set D. make
6. The Prime Minister has _______ from the dogmatic position he adopted a few days ago.
A. backed out B. backed down C. backed up D. backed on
7. At the South Pole _______, the coldest and most desolate region on Earth.
A. Antarctica lies where B. Antarctica lies and
C. where Antarctica lies D. lies Antarctica
8. _______ in the diet is especially important for vegetarians.
A. Enough protein is obtained B. Obtaining enough protein
C. They obtain enough protein D. By obtaining enough protein
9. _______ over long distances is a fact.
A. That electricity can be transmitted B. That electricity transmitting
C. That electricity D. That can be transmitted
10. Don’t leave paraffin heaters in a draught or where they’re _______
A. dangerously knocked over B. in danger knocked over
C. in danger of being knocked over D. in danger of knocking over
11. My father took _______ of the fine weather to a day’s work in his garden.
A. chance B. advantage C. interest D. charge
12. By 2050, medical technology _______ many diseases.
A. has conquered B. will conquer C. will have conquered D. is conquering
13. I realized _______ that he was a thief.
A. sooner or later B. all along C. at the beginning D. eventually
14. _______ his assistance in those days, I would not be so successful now.
A. If it had not for B. Had it not been for
C. If there were not D. Unless I had
15. They thought they could deceive me but they were wrong. I could ________.
A. see them off B. see off them C. see through them D. see them through
16. Janet will see you if you use the computer without permission. She has eyes like a _______.
A. hawk B. bird C. goose D. fox
17. My sister remained very calm. She _______.
A. kept her temper B. kept her soul C. kept her head D. lost her head
18. A network of railroads to unite the continent and encourage Western settlement _______ before the
Civil War by Asa Whitney.
A. when proposed B. to propose C. was proposed D. a proposal
19. Farmers are at the _______ of the bad weather that can destroy their crop.
A. change B. influence C. force D. mercy
20. He showed his _______ for the TV programme by switching it off.
A. distaste B. discontent C. annoyance D. boredom

Section 2: Use the word given in capitals at the end of each line to form a word that fits in the space at the
same line.
OBSESSED WITH YOUR INBOX?

It was not so long ago that we dealt with colleagues through face-
to-face interaction and with counterparts and customers by phone or
letter. But the world of communication has (1) ________ a dramatic 1. GO
transformation, not all for the good. Email, while (2) _______ a swift 2. DOUBT
means of communication providing your server is fully (3) ________
Page 66
and that the address you have contains no (4) _________ has had a 3. FUNCTION
(5) _______ effect on certain people's, both at home and in business. 4. ACCURATE
For these people, the use of email has become (6) _______ addictive 5. SIGNIFY
to the extent that it is (7) ________ their mental and physical health. 6. RESIST
Addicts spend their day (8) ________ checking for email and have a 7. THREAD
(9) ________ to panic if their server goes down. It is estimated that 8. COMPULSION
one in six people spend four hours a day sending and receiving 9. TEND
messages the equivalent to more than two working days a week. The
negative effect on (10) ________ is something employers are well
aware of. 10. PRODUCE

Section 3: There are 10 errors in the following passage. Identify and correct them
Children who tell pop music does not interfere with their homework receive support today, with
the discovery that pay attention to visual stimuli and sounds requires completely different brain
pathways which can operate at the same time with your appreciation of either being damaged.
Researchers have founded that listening to car stereos does not create much interference when you are
driving. Similarly, pop music should not interfere to children’s homework. The affect of pop music on
their performance at it is far outweigh by other factors, such as how happy they are to be doing it. These
findings could be applied for the design of places which people have to take in large amounts of
information very quickly. They could, for example, be relevantly to the layout of pilot cockpits on
aircraft.

LINE MISTAKE CORRECTION LINE MISTAKE CORRECTION

Section 4: Complete each of the following sentences with a suitable preposition.

1. I have already told Tim that I won’t go to Spain with him, but he still trying to talk me _______.
2. _______ receipt of your instructions, I immediately sent a telex message to Algeria.
3. Grandfather must be tired. He’s nodding _______ in his chair.
4. The applause died _______ after 5 minutes.
5. He accepted my advice _______ reservation.
6. Our students are orientated _______ the science subjects.
7. Cheques should be made _______ to Wellington.
8. I’m in no hurry. I’ll hang _______ here until you’re ready.
9. She is Polish _______ birth, but she has French nationality.
10. Your plan doesn’t allow _______ changes in the weather.

PART 3. READING
Section 1: Fill each of the following numbered blanks with ONE suitable word and write your
answers in the corresponding boxes provided below the passage.
How to win friends and influence colleagues
The worst sins that office workers can commit in the eyes of colleagues are interrupting people on
the phone, talking loudly in front of someone’s desk and arriving late (1) _______ a meeting. It is almost
(2) _______ bad to eat at your desk, make personal phone calls or leave coffee cups (3) _______ around,
according to Bodytalk, a guide published by the Industrial Society, (4) _______ lists the 30 traits most
likely to annoy colleagues.
High on the list are signing on at someone else’s computer terminal (5) _______ signing off
afterwards, messing up the photocopier by jamming the paper or leaving it set to produce multiple copies.
Reading newspaper, doodling, chewing gum, keeping cuddly toys or ornaments on your desk, and putting up
postcards or supposedly witty slogan (6) _______ invite disapproval. Also unpopular are (7) _______ who
never fetch a colleague a drink from the machine, or who (8) _______ to hold the lift door open when others

Page 67
are approaching. (9) _______ conduct, according to the guide’s author, Judi James, not only gives you an
unprofessional image but is downright anti-social and could result (10) _______ dismissal.

Section 2: Fill the gaps in the passage by choosing the best phrases from A-M.
A few years ago one enlightened city decided to ease traffic congestion by (1) _______. People
would just take a bike, ride it to where they were going and leave it (2) _______. The trouble was the citizens
naturally found it (3) _______ to have their own free bicycle and (4) _______.
That slight detail of human nature apart, it was a good idea and (5) _______. The electric cars
buzzing round the streets of La Rochelle in France are seen (6) _______ and, it is clamed, will make it easier
(7) _______ within the city centre.
The idea is based on the belief that people like their own space, the freedom to
drive a vehicle themselves (8) _______. It’s also based on the fact that cars in towns
usually carry just one person. In effect, it’s (9) _______. At the moment, it’s a slightly
utonpian view but the concept has grown (10) _______.
A. to end the use of private cars H. supplying sufficient communal bicycles
B. rather than crowd on to a bus or train I. taking the place of bicycles
C. to encourage their use J. a sort of do-it-yourself taxi
D. stock swiftly dwindled K. as a supplement to conventional cars
E. out of a practical study L. instead of car
F. for someone else to use M. it’s now the basic of a new scheme
G. even more convenient

Section 3: Read the following passage and mark the letter A, B, C, or D on your answer sheet to
indicate the correct answer to each of the questions.
One of the seven wonders of the ancient world, the Great Pyramid of Giza was a monument of
wisdom and prophecy built as a tomb for Pharaoh Cheops in 2720 B.C. Despite its antiquity, certain
aspects of its construction makes it one of the truly wonders of the world. The thirteen- acre structure
near the Nile river is a solid mass of stone blocks covered with limestone. Inside are the number of
hidden passageways and the burial chamber of the Pharaoh. It is the largest single structure in the world.
The four sides of the pyramid are aligned almost exactly on true north, south, east and west-an incredible
engineering feat. The ancient Egyptians were sun worshippers and great astronomers, so computations
for the Great Pyramid were based on astronomical observations.
Explorations and detailed examinations of the base of the structure reveal many intersecting lines.
Further scientific study indicates that these represent a type of timeline of events – past, present and
future. Many of the events have been interpreted and found to coincide with known facts of the past.
Others are prophesied for future generations and are currently under investigation. Many believe that
pyramids have supernatural powers and this one is no exception. Some researchers even associate it with
extraterrestrial beings of ancient past.
Was this superstructure made by ordinary beings, or one built by a race far superior to any known
today?
1. What has research of the base revealed?
A. there are cracks in the foundation B. Tomb robbers have stolen the Pharaoh’s body
C. The lines represent important events D. A superior race of people built in
2. Extraterrestrial beings are _______.
A. very strong workers B. astronomers in the ancient times
C. researchers in Egyptology D. living beings from other planets
3. What was the most probable reason for providing so many hidden passages?
A. To allow the weight of the pyramid to settle evenly
B. To permit the high priests to pray at night
C. To enable the Pharaoh’s family to bring food for his journey to the afterlife
D. To keep grave robbers from finding the tomb and the treasure buried with the pharaoh
4. The word “feat”in the first paragraph is closet in meaning to _______.
A. accomplishment B. Appendage C. festivity D. structure
5. What do the intersecting lines in the base symbolize?
Page 68
A. Architects’plan for the hidden passages
B. Pathways of the great solar bodies
C. Astrological computations
D. Dates of important events taking place throughout time
6. In the second passage, the word ‘prophesied’ is closest in meaning to _______.
A. affiliated B. precipitated C. terminated D. foretold
7. What is the best title for the passage?
A. Symbolism of the Great Pyramid B. Problems with the Construction of the Great
Pyramid
C. Wonders of the Great Pyramid of Giza D. Exploration of the Burial Chamber of Cheops
8. On what did the ancient Egyptians base their calculations?
A. Observation of the celestial bodies B. Advanced technology
C. Advanced tools of measurement D. knowledge of the earth’s surface
9. Why was the Great Pyramid constructed?
A. As a solar observatory B. As a religious temple
C. As a tomb for the Pharaoh D. as an engineering feat
10. Why is the Great Pyramid of Giza considered one of the seven wonders of the world?
A. it is perfectly aligned with the four cardinal points of the compass and contains many prophecies
B. It was selected of the tomb of Pharaoh Cheops
C. It was built by a super race
D. It is very old

Section 4: Choose the best answer (A, B, C or D) to complete each numbered gap in the passage
below. Identify your answer by writing the corresponding letter A, B, C or D on your answer sheet.

CARNIVOROUS PLANTS

All plants rely on nutrients taken from the soil in order to survive. However, in areas where the soil does
not contain enough (1) _______ nutrients, some plants have adapted to (2) _______ their diets from another
source: living organisms. Though they are few in number, carnivorous plants are (3) _______ fascinating
beings that “eat” anything from one-celled organisms to insects in order to survive. They are commonly
found in marshlands. Carnivorous plants feature one of several types of “traps” to ensnare prey, which they
consume to make up for nutrients that may be missing from the soil. While there are over 400 species of
carnivorous plants in the world today, some are more (4) _______ than others.
The most well-known of these plants are the snap traps, which include the Venus flytrap. Snap traps are
easily identified by their leaves, which are separated into two lobes that have the ability to fold together.
Inside the lobes, the surface is covered with tiny hairs that are (5) _______ to movement. When the plant’s
prey brushes against the hairs, it triggers a closing mechanism that rapidly brings the two lobes together,
trapping the prey (6) _______ inside. The response of the traps is phenomenal (7) _______ speed: the time
between triggering the hairs and snapping shut is less than a second. As the prey struggles inside the trap, it
only triggers more hairs, causing the leaves to tighten their (8) _______. The plant then secrets liquid
chemicals from special glands into the trap to dissolve the prey and absorb all of its nutrients. Besides the
Venus flytrap, only one other type of snap trap exists today, (9) _______ to as the waterwheel plant. The two
share a common ancestor and differ only in a few ways. For instance, the waterwheel is an aquatic plant,
while the flytrap is exclusively terrestrial. In addition, the flytrap feeds primarily on arthropods like spiders,
while the waterwheel lives (10) _______ simple invertebrates, like certain types of plankton.
1. A. critical B. vital C. crucial D. indispensable
2. A. modify B. enlarge C. augment D. supplement
3. A. nonetheless B. though C. contradictorily D. yet
4. A. prevalent B. current C. domineering D. prevailing
5. A. vulnerable B. liable C. prone D. sensitive
6. A. closely B. securely C. irreplaceably D. steadily
7. A. in accordance with B. in preference to C. in regard to D. on merits of
8. A. fist B. hold C. seizure D. grip
9. A. denoted B. referred C. indicated D. implicated
10. A. off B. onto C. though D. with
Page 69
PART E. WRITING
Section 1: Write a new sentence as similar as possible in meaning to the original sentence, but using
the word given. The word must not be altered in any way.
1. The new lecturer was unpopular with his students. TAKE
 ___________________________________________________.
2. John decided that an actor's life was not of him. CUT
 ___________________________________________________.
3. The number of people out of work has been going down little by little. GRADUAL
 ___________________________________________________.
4. I really want to see her again. DYING
 ___________________________________________________.
5. What did his silence mean? SIGNIFICANCE
 ___________________________________________________.

Section 2: Write an essay of around 250 words on the following topic.


The idea of going overseas for university study is an exciting prospect for many people. But while it
may offer some advantages, it is probably better to stay home because of the difficulties a student
inevitably encounters living and studying in a different culture.
To what extent do you agree or disagree this statement?
Give reasons for your answer.

ENGLISH PRACTICE 17
I – PRONUNCIATION
Choose the word whose stress pattern is different from the other three in the following questions
1. A. complain B. gather C. product D. transcript
2. A. mosquito B. elephant C. dinosaur D. buffalo
3. A. protect B. settle C. relate D. compose
4. A. government B. understand C. interview D. reference
5. A. scholarship B. develop C. equipment D. discourage
6. A. integral B. terrific C. cathedral D. museum
7. A. appreciative B. archeologist C. architectural D. cosmopolitan
8. A. photograph B. separate C. recommend D. hurricane
9. A. tonight B. control C. purpose D. involve
10. A. enough B. aware C. support D. swallow
II. LEXICO-GRAMMAR
Exercise 1: Choose the best answer for each question.
1. My sister never allows ____ a party at home.
A. to have B. me having C. having had D. having
2. She forgot _____ her toilet bag for her trip and she is now a little annoyed about it.
A. to be packing B. having packed C. packing D. to pack
3. When I __________ how to ski for the first time, I felt so happy.
A. succeeded B. began C. studied D. learned
4. Thailand defeated Laos ___2______3.
A. with/by B. by/of C. by/to D. with/of
5. A number of political prisoners….. since Independent Day
A. have been released B. have released C. was released D. has been released
6. The street is very noisy,….makes sleeping difficult
A. which B. what C. that D. it
7. There’s someone at the door. ………….. it
A. I am going to answer B. I’m answering C. I’ll answer D. I answered
Page 70
8. There are a lot of…. buildings in the centre of Sydney.
A. many-floored B. multi-storey C. multi-storied D. many-story
9. It’s surprising how few people can… between margine and butter.
A. distinguish B. seprate C. disgnose D. estimate
10. Rapid _______ would cause frequent floods and leave little water during dry seasons.
A. run-off B. run-in C. run-out D. run-over
11. You are _______ to clean the bathroom.
A. declared B. imagined C. supposed D. spoken
12. They are passing new laws to ________ wildlife in the area.
A. conserve B. protest C. shield D. shelter
13. They ________ a serious threat to security.
A. put B. place C. set D. pose
14. _______ did he pay for his transitor radio?
A. How high B. How many C. How much D. How often
15. More than half the staff ___________ temporary.
A. is B. are C. was D. were
16. He reduced _________ and turned sharp left.
A. speed B. rapidity C. fast D. rate
17. What’s taking an entrance exam ___________?
A. like B. alike C. likely D. look like
18. The kids found it hard to stay ________.
A. unmoved B. unmoving C. still D. stuck
19. _________ he became famous did he realize the price of fame.
A. Only until B. Not until C. Since D. Until
20. He is _________ now so he can’t afford to buy a car.
A.out of job B. out of order C. out of work D. out of
employment
1. 2. 3. 4. 5.
6. 7. 8. 9. 10.
11. 12. 13. 14. 15.
16. 17. 18. 19. 20.
Exercise 2: Provide the correct article A/AN/THE/X (zero article) in the following gaps.
(21)________first few days after getting home from (22)______ college were okay but
then I began to feel rather bored and lonely. My parents were busy working and
(23)______most of my friends were either away on holiday or else had (24)________
holiday jobs. I tried to get a job myself but without success. I was beginning to wish I had
not come home when I saw (25)_______advertisement for holiday courses in
(26)________local newspaper. For (27)_______small fee you could try lots of different
activities organized by (28)________ town’s youth club. Each activity lasted half
(29)________ day and they were all exciting. I also met new people as many of
(30)________ tourists staying in the town joined in.
21. 22. 23. 24. 25.
26. 27. 28. 29. 30.
Exercise 3: Provide the correct form of the word in bold capitals.
The World Health Organisation (WHO) is (31) WORRY that 1.1 billion teenagers and people in
their twenties and early thirties are damaging their (32) HEAR by listening to loud music. It says
(33) NEAR half the young people in middle and high-income countries risk hearing loss because of
the "(34) SAFETY use" of personal music (35) PLAY, including smartphones. Loud music in
Page 71
nightclubs, bars and at sporting events also increases the risk. The WHO recommends a safe (36)
LIMITED of listening to music for just one hour a day. The WHO director for (37) INJURE
prevention, Dr Etienne Krug, told the BBC that: "What we're trying to do is raise (38) AWARE of
an issue that is not (39) TALK about enough." He said hearing loss is easily (40) PREVENT.
21. 22. 23. 24. 25.
26. 27. 28. 29. 30.
Exercise 4: Choose the incorrect parts and correct them.
31. Despite (A) fats and oil are nutritionally (B) important as (C) energy sources, medical research
indicates that saturated fats may contribute to hardening (D) of the arteries.
32. The radio telescope, invented (A) in 1932, has capabilities (B) beyond far (C) those of optical
telescopes in tracking (D) signals from galaxies.
33. The boat rose and fell slow (A) as (B) the huge sea swells (C) moved (D) towards shore.
34. She was determined (A) to pursue (B) a tertiary education however (C) happened to her (D).
35. Today there is (A) an unusually (B) large amount (C) of students in the class (D).
III. READING
Exercise 1: Questions 1 – 10
Clinic for internet addicts opens in US
If you have to take a quick break from reading this article to send a text message or check your
Blackberry, then the good news is that help is at hand.
A new centre for cyberspace addicts has opened in the US city of Seattle. It claims to be America’s first
residential centre for people addicted to the internet and video games. For $14,000, six people at a time
can spend 45 days fighting against their constant need to connect to the internet. Residents are given
counselling and psychotherapy, as well as group activities such as household chores, walks in the
grounds and exercising.
The centre, in the countryside about 50km outside Seattle, is the idea of therapist Hilarie Cash. She
recorded her first case of cyberspace addiction in 1994, with a patient so glued to video games that he
lost his marriage and two jobs. But until this summer she had only met patients on a day-by-day basis.
Cash points out that although countries such as China, South Korea and Taiwan have places where
internet addicts can find help, America has been slow to recognise the problem.
Ben Alexander, 19, was one of the centre’s first residents. He became addicted to the video game World
of Warcraft, which took up most of his time awake. He started playing the game when he was a new
student at Iowa University. “At first it was a couple of hours a day. By midway through the first
semester, I was playing 16 or 17 hours a day,” he said.
By comparison to the whizz-bang on the screen in front of him, the social life at university seemed very
boring. He came to see the game as an “easy way to socialise and meet people.” Alexander eventually
left the university and found help with his gaming problem. “I don’t think I’ll go back to World of
Warcraft any time soon,” he said.
Questions 1 – 6: Read the text and choose the correct answers to these questions.
1. The centre is for people who spend too much time ____________
A. at the office B. on the phone C. online
2. The residents at the centre do activities ___________
A. in pairs B. in small groups C. in big groups
3. Cash’s first patient was addicted to _____________
A. video games B. exercising C. household chores
4. People have been slow to recognise the addiction in ___________
A. China B. South Korea C. the US
5. Ben began to have problems when he started ______________
A. at college B. at the centre C. at work
6. Ben found the game a good way to _____________
A. study B. make friends C. help his problem
Questions 7 – 12: Read the text and write NO MORE THAN THREE WORDS for each answer to
these questions.
Page 72
7. Where is the new centre?
8. How much does the course cost?
9. How long is the course?
10. When did Cash have her fist addicted patient?
11. What internet game did Ben play?
12.How many hours a day did Ben play the game?
1. 2. 3. 4.
5. 6. 7. 8.
9. 10. 11. 12.

Exercise 2: Questions 13 – 20
Read the text and fill in each blank with a most suitable word.
School students do not shower after sports class
A report has found that more than half of high (13)__________ boys and two-thirds of
girls never shower after a physical (14)__________ class. Researchers suggest that
students don't want to sweat and take a (15)__________ , so they are less active in sports
classes. The researchers questioned almost 4,000 children in schools in Essex, England.
Lead researcher Dr Gavin Sandercock said he was surprised (16)__________ how rarely
students showered. He said: "We know (17)__________ children aren't getting
(18)__________ physical activity because we have seen their fitness declining;
(19)__________ the unwillingness to shower is a barrier to working up a sweat
(20)__________ playing sport, it's something we need to tackle to promote activity at
schools."
13. 14. 15. 16.
17. 18. 19. 20.

Exercise 3: Questions 21 – 30
Read the text and choose the most suitable option to complete the text.
The Internet bus
In the desert areas that surround Tucson city, USA, students spend hundreds of hours on yellow buses
each year getting to and from their schools. But when mobile internet equipment was (21)____________
on a yellow school bus, the bored, often noisy, teens were transformed into quiet, studious individuals.
District officials got the idea during (22)____________ drives on school business to Phoenix, two hours
each way, when they (23)____________ that, when they went in pairs, one person could drive and the
other could work using a laptop and a wireless card. They (24)____________ if internet access on a
school bus could (25)____________ students’ academic productivity, too.
But the idea for what students call ‘the internet bus’ really took shape when the district’s chief
information officer (26)____________ across an article about having internet across in cars. He
thought, “What if you could put that in a bus?”
The officials have been delighted to see the (27)____________ of homework getting done, morning and
evening, as the internet bus (28)____________ up and drops off students along the 70-minute drive.
(29)____________ some students spend their time playing games or visiting social networking sites,
most students do make (30)____________ their travel time to study.
21. A. installed B. set C. included D. structured
22. A. extraordinary B. occasional C. exceptional D. few
23. A. believed B. acknowledged C. estimated D. realised
24. A. thought B. imagined C. suspected D. wondered
25. A. increase B. enlarge C. rise D. heighten
26. A. got B. looked C. came D. put
27. A. total B. amount C. number D. measure
28. A. brings B. picks C. rides D. catches
Page 73
29. A. Since B. Despite C. Although D. However
30. A. progress B. work C. use D. part

31. 32. 33. 34. 35.


36. 37. 38. 39. 40.

IV. WRITING
Exercise 1: Complete the following sentences so that their meanings are close to the originals.
1. I can’t say I enjoy it when people tear my writing to pieces in front of me.
 I can’t say I enjoy having …………………………………………..
2. We’ll have to make up our minds by the end of the week, won’t we?
 A decision will have to …………………………………………..
3. The job received over a hundred applications.
 Over a hundred people …………………………………………..
4. This story of yours isn’t true!
 You have made …………………………………………..
5. The best solution was thought of by Sally.
 Sally came …………………………………………..
Exercise 2: Write a PARAGRAPH of at least 120 words on the following topic:
If you could study a subject that you have never had the opportunity to study, what would you choose?
Explain your choice, using specific reasons and details.

ENGLISH PRACTICE 20
PART I. PHONETICS
I. Pick out the word whose underlined part is pronounced differently from those of the other
words.
1. A. latter B. label C. ladder D. latitude
2. A. pour B. roar C. flour D. soar
3. A. chase B. purchase C. bookcase D. suitcase
4. A. thread B. feather C. bread D. bead
5. A. prepare B. preface C. preparation D. prejudice
II. Choose the word whose stress pattern is differently from those of the other words.
1. A. amphibian B. champagne C. cathedral D. creature
2. A. accommodation B. antibiotic C. counterclockwise D. deforestation
3. A. consciousness B. ecotourism C. biosphere D. confirm
4. A. architectural B. cosmopolitan C. appreciative D.
archeologist
5. A. consolidate B. context C. conference D. confidence

PART II. LEXICO-GRAMMAR


I. Read the sentences carefully and choose the best answers to make complete ones.
1. From 1949 onward, the artist Georgia O’keeffe made New Mexico _______.
A. her permanent residence was B. where her permanent residence
C. permanent residence for her D. her permanent residence
2. Just as remote-controlled satellites can be employed to explore outer space, _______ employed to
investigate the deep sea.
A. can be robots B. robots can be C. can robots D. can robots that are
3. _______ is not clear to researchers.
A. Why dinosaurs having become extinct. B. Why dinosaurs became extinct
C. Did dinosaurs become extinct D. Dinosaurs became extinct
4. The first transatlantic telephone cable system was not established _______ 1956.
A. while B. until C. on D. when
Page 74
5. Drinking water _______ excessive amounts of fluorides may leave a stained or mottled effect on the
enamel of teeth.
A. containing B. in which containing
C. contains D. that contain
6. In the 1820’s physical education became _______ of the curriculum of Harvard and Yale Universities.
A. to be part B. which was part C. was part D part
7. If you can win his attention ___________ for you.
A. the so much better B. the better so much
C. so much the better D. so the much better
8. The President resigned; the whiff of scandal remained ____________.
A. otherwise B. therefore C. immediately D. nevertheless
9. Which __________ agency do you work for?
A. ads B. advertised C. advertising D. advertisement
10. Van Gogh suffered from depression ____________ by overwork and ill-health.
A. brought on B. coming about C. taken up D. put through

II. Each sentence below contains 1 mistake. IDENTIFY the mistakes and WRITE THEIR
CORRECT FORMS.
1. Had the committee members considered the alternatives more carefully, they would have realized that
the second was superior than the first:
2. Malnutrition is a major cause of death in those countries where the cultivation of rice have been
impeded by recurrent drought.
3. Because the residents had worked so diligent to renovate the old building, the manager had a party.
4. John’s wisdom teeth were troubling him, so he went to a dental surgeon to see about having them pull.
5. Time spends very slowly when you are waiting for a bus to arrive
6. Judy decided to wait until after she had taken her exams before having her wisdom teeth pull.
7. Hardly the plane had landed when Adam realized that he had left the file that he needed at his office.
8. When she was asked for her opinion on the course, she said it had been a waist of time.
9. I try to remember your name but I am afraid I cannot remember it.
10. I’d prefer to do it on myself because other people make me nervous.
III. Fill in each blank with a suitable PREPOSITION or PARTICLE.
1. I know ________ experience that I do my best work early in the morning.
2. I am astonished ________ the way my students can spend all night at the disco and still remember
their prepositions next morning.
3. Is it possible to insure yourself ________ nuclear attack?
4. At school today, we had a long discussion _______ the best way to learn a foreign language.
5. “How can I discourage my boyfriend _______ trying to kiss me all the time?” “Eat plenty of garlic.”
6. Raise the gun to your shoulder, aim _______ the target, and try not to kill anyone.
7. Small children should be watched ____________ carefully.
8. Would you give up your country cottage __________ a town flat?
9. He’ll inherit the money when he comes ______________ age.
10.The performance on the first night came ______________ pretty.
IV. Write the correct FORM of each bracketed word.
1. (ADAPT)__________ to a new environment is a difficult thing for old people.
2. Police asked (PASS) ______________ if they had seen the accident happen.
3. What a (WORRY) ______________pair of children. They have to be watched every minute of the
day.
4. Children are taught from young to be (RESPECT) _____________ to their elders.
5. They frequently (MOBILE) _________________ the traffic as they march through the streets.
6. He shook his head in _____________ (APPROVE)
7. He fought the illness with courage and ________________ (DETERMINE)
8. She seems (REASON) __________happy in her new job.
9. Can we (ARRANGE) _____________the meeting for next Monday at 7?
10. If you weren’t so (ACT)_______________, you wouldn’t be so fat!

PART III. READING


Page 75
I. Read the text & decide which word best fits each space by choosing A, B, C or D.
Action scenes in films
Modern cinema audiences expect to see plenty of thrilling scenes in action films. These scenes, which
are (1) __________ as stunts, are usually performed by stuntmen who are specially trained to do
dangerous things safely. (2) ________ can crash a car, but if you are shooting a film, you have to be
extremely (3) __________ sometimes stopping right in front of the camera and film crew. At an early
(4) ___________ in the production, an expert stuntman is (5) __________ in to work out the action
scenes and form a team. He is the only person who can go (6) __________ the wishes of the director,
although he will usually only do this in the (7) __________ of safety.
Many famous actors like to do the dangerous parts themselves, which produces better shots, since
stuntmen don’t have to (8) _________ in for the actors. Actors like to become (9) _________ in all the
important aspects of the character they are playing, but without the recent progress in safety equipment,
insurance companies would never let them take the risk. To do their own stunts, actors need to be good
athletes, but they must also be sensible and know their (10) ___________.If they were to be hurt, the
film would come to a sudden halt.
1. A. remarked B. known C. referred D. named
2. A. Everyone B. Someone C. Anyone D. No one
3. A. detailed B. plain C. straight D. precise
4. A. period B. minute C. part D. stage
5. A. led B. taken C. drawn D. called
6. A. over B. against C. through D. across
7. A. interests B. needs C. purposes D. regards
8. A. work B. get C. put D. stand
9. A. connected B. arranged C. involved D. affected
10. A. limits B. ends C. frontiers D. borders

II. Fill in each blank space with an appropriate word.


THE BIRTH OF THE T-SHIRT
The T-shirt, or at least the T-shirt as we know it, was born in the theatre. When Tennessee
William's play A Streetcar Named Oesire opened in New York in December 1947, a young actor (1)
____________ Marlon Brando went (2) __________ stage wearing a (3) _________ of blue jeans and a
bright, white, capped-sleeve T-shirt. It was the first time the T-shirt had been seen publicly as anything
(4) ____________ an item of underwear and it set a fashion trend that was to last through (5)
_____________ the end of the century. The idea for the T-shirt came (6) __________ Brando himself.
He had worn one at rehearsals for the play. The director was so impressed by the look that was created
that he asked Brando to wear the shirt in the play itself. Brando may have seen the shirt being advertised
by the American company Sears Roebuck. They had decided to market the shirt (7) ___________ a
fashionable garment in its (8) ____________ right, rather than just something to be worn (9)
__________ warmth beneath a denim workshirt (10) ___________ an army uniform. It was Brando,
however, who popularized it, especially with the release of the film version of Streetcar in 1951. A short
leather jacket completed the look that was to be adopted by teenage rebels in many countries for decades
afterwards.

III. Read the following passage and choose the best answer.
Since the world became industrialized, the number of animal species that have either become
extinct or have neared extinction has increased. Bengal tigers, for instance, which once roamed the
jungles in vast numbers, now number only about 2,300. By the year 2025, it is estimated that they will
become extinct. What is alarming about the case of the Bengal tiger is that this extinction will have
been caused almost entirely by poachers who, according to some sources, are not always interested in
material gain but in personal gratification. This is an example of the callousness that is contributing to
the problem of extinction. Animals, such as Bengal tiger, as well as other endangered species, are
valuable parts of the world’s ecosystem. International laws protecting these animals must be enacted to
ensure their survival and the survival of our planet.
Countries around the world have begun to deal with the problem in various ways. Some
countries, in an effort to circumvent the problem, have allocated large amounts of land to animal
reserves. They then charge admission prices to help defray the costs of maintaining the parks, and they
Page 76
often must also depend on world organizations for support. This money enables them to invest in
equipment and patrols to protect the animals. Another response to the increase in animal extinction is an
international boycott of products made from endangered species. This has had some effect, but by itself
it will not prevent animals from being hunted and killed.
1. What is the main topic of the passage?
A. the Bengal tiger B. international boycott
C. endangered species D. problems with industrialization
2. Which of the following is closest in meaning to the world “alarming” in the first paragraph?
A. dangerous B. serious C. gripping D. distressing
3. The word “poachers” as used in the first paragraph could best be replaced by which of the following?
A. illegal hunters B. enterprising researchers
C. concerned scientists D. trained hunters
4. The word “callousness” in the first paragraph could best be replaced by which of the following?
A. indirectness B. independence C. incompetence D. insensitivity
5. The above passage is divided into two paragraphs in order to contrast.
A. a problem and a solution B. a statement and an illustration
C. a comparison and contrast D. specific and general information
6. What does the word “this” in the first paragraph refer to in the passage?
A. Bengal tigers B. Interest in material gain
C. Killing animals for personal satisfaction D. The decrease in the Bengal tiger population
7. Which of the following could best replace the word “allocated” in the second paragraph?
A. set aside B. combined C. organized D. taken off
8. The word “defray” in the second paragraph is closest in meaning to which of the following?
A. lower B. raise C. make a payment on D. make an investment
toward
9. What does the term “international boycott” in the second paragraph refer to?
A. buying and selling of animal products overseas
B. a refusal to buy animal products worldwide
C. a global increase in animal survival
D. defraying the cost of maintaining national parks
10. Which of the following best describes the author’s attitude?
A. forgiving B. concerned C. vindictive D. surprised

PART IV. WRITING


I. Finish each of the sentences in such a way that it means exactly the same as the sentence printed
before it.
1. She never seems to succeed even though she works hard.
Hard _____________________________________________________.
2. Do all the washing, please!  
Let ______________________________________________________.
3. I regret not going to the airport to say good bye to him.
I wish ___________________________________________________.
4. Mick thought that we were married.
Mick was under _________________________________________.
5. The only thing that kept us out of prison was the way he spoke the local dialect.
But for his command ____________________________________.
II. Write an essay ( about 250 words) about the following topic:
Some people say that traffic accidents are caused by the increasing number of motorbikes.
Others blame for man’s fault. Which point of view do you agree? State at least three relevant
evidences.
ENGLISH PRACTICE 19
I – PRONUNCIATION
Choose the word whose stress pattern is different from the other three in the following questions
1. A. curriculum B. grammatical C. repetitious D. historical
2. A. emergency B. geothermal C. alternative D. expensively

Page 77
3. A. technology B. variety C. centenarian D. Compulsory
4. a. reindeer b. engineer c. pioneer d. referee
5. a. dangerous b. marvelous c. conspicuous d. numerous
6. a. establish b. illustrate c. intimidate d. inheritance
7. a. expansion b. conversion c. precision d. explosion
8. a. eradicate b. character c. malaria d. spectacular
9. A.encourage B.financial C.departure D.average
10. A.career B.request C.ancient D.machine
PART II. GRAMMAR AND VOCABULARY
I. Complete the following sentences by choosing the correct answer among four options (A, B, C or
D).
1. She looked very______ when I told her the good news.
A. happily B. happiness C. happy D. was happy
2. I don't want much sugar in my coffee. Just_______, please.
A. few B. a few C. little D. a little
3. He's left his book at home; he's always so______.
A. forgetful B. forgettable C. forgotten D. forgetting
4. 'I'm very ______ to you for putting in so much hard work,' the boss said.
A. thanking B. grateful C. considerate D. careful
5. Steel_______ with chromium to produce a noncorrosive substance known as stainless steel.
A. is combined B. that is combined C. combining D. when combined
6. Joe seemed to be in a good mood, ________ he snapped at me angrily when I asked him to join
us.
A. yet B. so C. for D. and
7. A fire must have a readily available supply of oxygen. ________, it will stop burning.
A. Consequently B. Furthermore C. Otherwise D. However
8. “Who has brothers and sisters?”
“Everyone ________Virginia who’s an only child.”
A. except to B. with C. from D. but
9. He is the manager of the factory. He’s ________it.
A. charged with B. charged C. in charge D. in charge of
10. He was new on the job, but he quickly fit himself into the ________ routine of the office.
A. establishing B. established C. establishes D. establish
11. Florida, ________ the Sunshine State, attracts many tourists every year.
A. is B. is known as C. known as D. that is known as
12. ________becoming extinct is of great concern to zoologists.
A. That giant pandas are B. Giant pandas are
C. Are giant pandas D. Giant panda is
13. Her parents gave her everything she asked for. She's a completed _______ child.
A. wounded B. spoilt C. damaged D. destroyed
14. _______ my experience, very few people really understand the problem.
A. To B. In C. With D. From
15. When she died, she gave _________ all her money to a charity for cats.
A. away B. out C. on D. off
II. Complete the following passage by supplying the correct form of the word to fill in each blank.
The increase in city crime is a global  phenomenon. Some people say that a lot of crime in this
country is because of (1 - migrate) ______ and the new people arriving from other countries bring
different (2 - culture) ______ values and attitudes to the law. I don’t agree with this idea because the
most common crimes are (3 - local) ______ produced and not imported from other countries. (4 -
Vandal) ______ is one of the biggest crimes in my city with bus shelters and shop windows being
popular targets. Another popular crime is (5 - pay) ______ parking fines, which is unlikely to be
because of immigrants because most of them do not own cars. The (6 - oppose) ______ point of view is
that young local people feel angry when they can’t get a job and in order to (7 - hand) ______ the
change in their environment, they strike out at easy targets. This would explain why bicycle (8 -
Page 78
thief) ______ is more common than car crime these days, especially in rich (9 - neighbors) ______
where most cars are protected with electronic alarms. Another reason, though, for so many bicycles
getting stolen might be that the (10 - punish) ______ is not very severe compared to car stealing which
can land you in prison for a number of years.
III. Fill in each blank with one suitable preposition.
1. Most American men earn _______ average about $110 a week.
2. He was walking through the park when a strange dog suddenly went _______ him.
3. My car is not worth much _______ most $ 50.
4. I’d better not drink that milk, Joe. It’s gone _______.
5. Clive was really cut _______ when he failed his proficiency exam.
IV. Identify the error in each sentence. Write the corresponding letter A, B, C or D on the answer
sheet.
1. The children forgot picking up the note from the office and now they are worried.
A B C D
2. Helen has never met such good person who is ready to help others.
A B C D
3. If only I have done the test better than other students did.
A B C D
4. If you want to borrow my car, promise to drive careful.
A B C D
5. Rumors began circulating that the Prime Minister was seriously illness.
A B C D
6. Everybody in class has to choose a topic of your own to write an essay of 500 words.
A B C D
7. Neither of the boys had ever been out of town before, so they were really exciting.
A B C D
8. I hope that I can help you with the historic questions.
A B C D
9. My son learned to talk the time before he was 5 years old.
A B C D
10. Jim offered us presents as if it had been Xmas.
A B C D

PART III: READING


I. Read the passage carefully, then choose the correct option (marked A, B, C or D) to answer the
questions.
Million of people are using cell phones today. In many places, it is actually considered unusual
not to use one. In many countries, cell phones are very popular with young people. They find that the
phones are means of communication – having a mobile phone shows that they are cool and connected.
The explosion in mobile phone use around the world has made some health professionals
worried. Some doctors are concerned that in the future many people may suffer health problems from
the use of mobile phones. In England, there has been a serious debate about this issue. Mobile phone
companies are worried about the negative publicity of such ideas. They say that there is no proof that
mobile phones are bad for your health.
On the other hand, medical studies have shown changes in the brain cells of some people who
use mobile phones. Signs of change in the issues of the brain and head can be detected with modern
scanning equipment. In one case, a traveling salesman had to retire at young age because of serious
memory loss. He couldn't remember even simple tasks. He would often forget the name of his own son.
This man used to talk on his mobile phone for about six hours a day, every day of his working week, for
a couple of years. His family doctor blamed his mobile phone use, but his employer's doctor didn't agree.
What is it that makes mobile phones potentially harmful? The answer is radiation. High – tech
machines can detect very small amounts of radiation from mobile phones. Mobile phone companies
agree that there is some radiation, but they say the amount is too small to worry about.
Page 79
As the discussion about their safety continues, it appears that it's best to use mobile phones less
often. Use your regular phone of you want to talk for a long time. Use your mobile phone only when you
really need it. Mobile phones can be very useful and convenient, especially in emergencies. In the future,
mobile phones may have a warning label that says they are bad for your health. So for now, it's wise not
to use your mobile phone too often.
1. According to the passage, cell phones are especially popular with young people because ______.
A. they are indispensable in everyday communication
B. they make them look more stylish
C. they keep the users alert all the time
D. they cannot be replaced by regular phones
2. The changes possible caused by the cell phones are mainly concerned with______.
A. the mobility of the mind and the body B. the smallest units of brain
C. the arteries of the brain D. the resident memory
3. The word "means" in the passage most closely means _______.
A. meanings B. expression C. method D. transmission
4. The word "potentially" in the passage most closely means _______.
A. obviously B. possibly C. certainly D. privately
5. "Negative publicity" in the passage most likely means _______.
A. information on the lethal effects of cell phones
B. widespread opinion about bad effects of cell phones
C. the negative public use of cell phones
D. poor ideas about the effects of cell phones
6. Doctors have tentatively concluded that cell phones may _______.
A. damage their users' emotions B. cause some mental malfunction
C. change their users' temperament D. change their users' social behavior
7. The man mentioned in the passage, who used his cell phone too often _______.
A. suffered serious loss of mental ability B. could no longer think lucidly
C. abandoned his family D. had a problem with memory
8. According to the passage, what makes mobile phones potentially harmful is ______.
A. their radiant light B. their power of attraction
C. their raiding power D. their invisible rays
9. According to the writer, people should _______.
A. only use mobile phones in urgent cases
B. only use mobile phones in medical emergencies
C. keep off mobile phones regularly
D. never used mobile phones in all cases
10.The most suitable title for the passage could be ______.
A. "The reasons why mobile phones are regular"
B. "Technological Innovation and their price"
C. "They way mobile phones work"
D. "Mobile phones – a must of our time"
II. Complete the following passage by choosing A, B, C or D to fill in each blank.
Stamp collecting! What a wonderful hobby! I began when I was only five. I used to (1) ______
for the postman's arrival, always (2) ______ to seize unwanted envelopes and tear off the corner with the
stamp stuck on it.
Once – I remember it was too clearly – my mother and father were sunning themselves in the
garden when the post (3) _____ on the doormat. I heard the clatter of the letter flap and hurriedly went to
(4) ______. There were four or five envelops, all with very exciting stamps.
Even at the (5) ______ age of five I knew one doesn't open mail addressed to other people.
However, tearing just the corners off the envelops (6) ______ me as perfectly fair and allowable, and
just what I did. I carefully tore as (7) ______ to the stamps as (8) _______, feeling that even the
envelopes, which were addressed to my parents and not to be, should be treated with (9) _______.
There were nothing furtive in what I did. I knew my parents would see what I'd done, and I didn't
think there was any (10) ______ in it. They always let me (11) ______ the corners after they'd opened

Page 80
them. Why should I think there was any harm in doing it first, (12) ______ in mind that they weren't on
hand to be (13) ______. Wouldn't they rather be left to doze in their summer deckchairs?
(14) ______, though, my father solemnly showed me his letters. They looked distinctly moth-
eaten, with bites taken out of the corners and sites. I began to (15) ______ what I've done.
1. A. stare B. watch C. look D. peer
2. A. glad B. pleased C. eager D. excited
3. A. came B. was C. lay D. arrived
4. A. investigate B. observe C. see D. notice
5. A. junior B. tender C. small D. little
6. A. struck B. seemed C. appeared D. felt
7. A. nearby B. close C. next D. round
8. A. able B. possibly C. possible D. could
9. A. gentleness B. caution C. honor D. respect
10. A. trouble B. wrong C. bad D. harm
11. A. take B. tear C. cut D. remove
12. A. having B. holding C. bearing D. keeping
13. A. consulted B. advised C. queried D. requested
14. A. After B. Then C. Later D. Soon
15. A. accept B. realize C. admit D. confess
III. Read the following passage and answer the questions that follow. Write A, B, C or D to
indicate your answers on the answer sheet.
Every drop of water in the ocean, even in the deepest parts, responds to the forces that create the
tides. No other force that affects the sea is so strong. Compared with the tides, the waves created by the
wind are surface movements felt no more than a hundred fathoms below the surface. The currents also
seldom involve more than the upper several hundred fathoms despite their impressive sweep.
The tides are a response of the waters of the ocean to the pull of the Moon and the more distant
Sun. In theory, there is a gravitational attraction between the water and even the outermost star of the
universe. In reality, however, the pull of remote stars is so slight as to be obliterated by the control of
the Moon and, to a lesser extent, the Sun. Just as the Moon rises later each day by fifty minutes, on the
average, so, in most places, the time of high tide is correspondingly later each day. And as the Moon
waxes and wanes in its monthly cycle, so the height of the tide varies. The tidal movements are strongest
when the Moon is a sliver in the sky, and when it is full. These are the highest flood tides and the lowest
ebb tides of the lunar month and are called the spring tides. At these times the Sun, Moon, and Earth are
nearly in line and the pull of the two heavenly bodies is added together to bring the water high on the
beaches, to send its surf upward against the sea cliffs, and to draw a high tide into the harbors. Twice
each month, at the quarters of the Moon, when the Sun, Moon and Earth lie at the apexes of a triangular
configuration and the pull of the Sun and Moon are opposed, the moderate tidal movements called neap
tides occur. Then the difference between high and low water is less than at any other time during the
month.
1. What is the main point of the first paragraph?
A. The waves created by ocean currents are very large.
B. Despite the strength of the wind, it only moves surface water.
C. Deep ocean water is seldom affected by forces that move water.
D. The tides are the most powerful force to affect the movement of ocean water.
2. The words "In reality" in the passage is closest in meaning to
A. surprisingly B. actually C. characteristically D. similarly
3. It can be inferred from the passage that the most important factor in determining how much
gravitational effect one object in space has on the tides is
A. size B. distance C. temperature D. density
4. The word "configuration" in the passage is closest in meaning to
A. unit B. center C. surface D. arrangement
5. Neap tides occur when
A. the Sun counteracts the Moon's gravitational attraction
B. the Moon is full
C. the Moon is farthest from the Sun
D. waves created by the wind combine with the Moon's gravitational attraction
Page 81
PART IV. WRITING
I. Write the new sentences using the word give. Do not change the word given in any way.
1. We are afraid that the next train to Cardiff has been cancelled. REGRET
We  ________________________ the next train to Cardiff has been cancelled.
2. 'I was going to phone Sarah but I had too much homework to do', said John MEANT
John said that ___________________  but he'd had too much homework to do.
3. Many people think that Gone with the Wind is one of the greatest films ever made. BE
Gone with the Wind _________________  one of the greatest films ever made.
4. If you hold your breath for a few seconds it might get rid of your hiccups. HOLDING
You might be able to get rid of your hiccups if you _____________ for a few seconds.
5. Anna didn't like it at all when one of her colleagues got the promotion instead of her. RESENTED
Anna  _____________________ the promotion instead of her.
6. I have a good relationship with my neighbors. ON
My neighbors __________________well together.
7. It wasn't a good idea for me to spend all my money on beer and cigarettes, but I did. UP
I wish ________________________ my money on beer and cigarettes.
8. The police left and then all the journalists arrived. ALREADY
The police ____________________all the journalists arrived.
9. Although Sarah wasn't wearing a helmet, she wasn't injured in the accident. OF
Sarah wasn't injured in the accident, ______________ a helmet.
10. She felt she had achieved a lot in life, despite her disadvantaged background. SENSE
Despite her disadvantaged background, she felt _______________ about her life.
II. Write a complete letter using the given word/phrases as cues. You should change the form of
words if necessary.
1. Thank/ much/French/ cookery book/ you/ give/ me.
……………………………………………………………………………………………………………

2. recipes/ look/ wonderful/, and/ I/ certainly/ enjoy/ use/ it.
……………………………………………………………………………………………………………
………………………..
3. I/ hope/ you/ and/ Auntie Susan/ have/ good/ Christmas.
……………………………………………………………………………………………………………
……
4. we/ stay/ at/ home/, but/ have/ couple/ day/ trips/ out/ nearby.
……………………………………………………………………………………………………………
………………………..
5. our/ local/ theatre/ put/ on/ production/ "A Christmas Carol"/, which/ fun.
……………………………………………………………………………………………………………
………………………..
6. I/ go/ back/ University/ next/ week/, and/ this/ year/ I/have/ to/ study/ hard/ final/ exams.
……………………………………………………………………………………………………………
………………………..
7. I/ really/ enjoy/ the/ course/ so/ far/, but/ I/ not/ sure/ what/ do/ once/ I/ get/degree.
……………………………………………………………………………………………………………
………………………..
8. university/ arrange/ career/ interviews/, but/ I/ not/really/ have/clear/ idea/what/ I/ want/ do.
……………………………………………………………………………………………………………
………………………..
9. hopefully/ it/ all/ become/ clearer/ during/ course/ the/ year.
……………………………………………………………………………………………………………
………………………..
10. Thank/ again/ lovely/ present/, and/ Happy/ New/ Year/
Page 82
……………………………………………………………………………………………………………

ENGLISH PRACTICE 21
I. LEXICO-GRAMMAR (10.8 pts)
a. Choose the best option A, B, C or D to complete the sentences.
1. She swam strongly and_______ cross the river easily, even though it was swollen by the heavy rain.
A. used to B. was able to C. could D. was supposed to
2. The government has spent one million pounds on an advertising ______ to encourage energy
conservation.
A. campaign B. promotion C. operation D. competition
3. Toxic chemicals are one of the factors that lead wildlife to the ______ of extinction.
A. wall B. fence C. verge D. bridge
4. ______ the best of my knowledge, this system cannot work automatically.
A. For B. To C. Within D. In
5. The aim of ASEAN is to promote closer economic_______.
A. delegation B. integration C. reputation D. migration
6. The International Committee of the Red Cross is a private _______ institution founded
in 1863 in Geneva, Switzerland.
A. human B. humanity C. humanization D. humanitarian
7. Affected by the Western cultures, Vietnamese young people's attitudes _______ love and marriage
have dramatically changed.
A. for B. with C. through D. towards
8. How do you speak the fraction 2/5?
A. two-five B. two-fifths C. second-fifths D. two-fifth
9. Don’t be late for the interview, _______ people will think you are a disorganized person.
A. unless B. otherwise C. if not D. or so
10. You may borrow as many books as you like, provided you show them to ______ at the desk.
A. whoever B. who C. whom D. which
11. It is essential that your luggage _______ thoroughly before delivery.
A. to be checked B. being checked C. be checked D. should be checking
12. The reason why this game attracts so many youngsters is that ___________ other video games, this
one is far more interesting.
A. comparing to B. in compared with
C. on comparison to D. in comparison with
13. Take the shortcut round the church _______ late for school.
A. in order not be B. in order that we won't be
C. so that not to be D. so that we couldn't
14. A cooperative program between two companies building a famous ancient city, into
a(n) _______ city has proceeded smoothly since it started in September last year.
A. friendly ecology B. ecology-friendly
C. friendly-ecological D. ecological-friendly
15. Doctor Pike ______ the hospital after he ______an uneventful evening on duty. He ______ of his
day of rest.
A. was leaving / has had / thought B. left / had had / was thinking
C. will leave / had / will think D. is leaving / will have / thinks
16. Something _____ immediately to prevent teenagers from _____ in factories and mines.
A. should be done /being exploited B. we should do /exploiting
C. should do / be exploited D. should have done /exploited
17. John: “Have you decided on a present yet?”
Ben: “Almost, I need to choose one of _______.”
A. new two exciting spy novels B. two spy exciting new novels
C. two exciting new spy novels D. exciting new two spy novels
b. Choose the best option A, B, C or D to indicate the word or phrase that is CLOSEST in
meaning to the underlined part in each of the following questions.
1. I knew he was only flattering me because he wanted to borrow some money.
Page 83
A. making me impatient         B. praising me too much        
C. making me feel worse D. elevating me        
2. We spent the entire day looking for a new apartment.
A. all day long B. all long day C. the long day D. day after day
3. He testified that he had seen the man leaving the building around the time of the murder.
A. said B. provided evidence C. disapproved D. denied
4. She has a deep aversion to getting up in the morning so she usually goes to work late.
A. intense dislike B. feeling C. provocation D. trouble
5. Those who transgress the laws of society will be punished.
A. disagree with B. disperse C. violate D. interfere with

c. Fill a suitable preposition/ particle into each gap to complete the sentences.
1. She felt a bit dizzy and had to lean _________ the wall before walking on.
2. Next year, we intend to bring __________ several new products. But at the moment,
we are still testing them.
3. I’m not sure if this is a suitable topic for a research paper. I need to talk it ______ with my teacher
before I start writing.
4. The store had to lay _______ a number of clerks because sales were down.
5. “Have you ever heard _________ William Carlos Williams?”
“I believe so. He was a poet, wasn’t he?”
6. “Mitchell looked pale and tired.”
“ He told me he was feeling a little _______ the weather.
7. “Are you going to take that job?”
“No, I decided to pass it _________ because I don’t want to relocate.”

d. Choose and give correct form of the verbs in column A with particles in column B to make
phrasal verbs to complete the sentences. There are some extra.
A B
give fall step go get hold off down in up through on
come

1. Let’s hope the rain will ___________ long enough for them to finish the cricket match.
2. He twisted my arm to make me tell what I knew, but I refused to ___________.
3. Some people think that the Queen should ___________ and allow the Prince to become King.
4. Whenever the subject of holidays _______________, Ruth and her husband have an argument.
5. They were planning to hold a Pop Concert in one of the parks but it ___________ owing to
opposition from the local residents.

e. Give the correct form of the words given to complete the sentences.
1. One is left with the ______________ problem of larger populations requiring more and more food.
(solve)
2. The ______________ of the trains and buses causes frustration and annoyance. (frequent)
3. Women who are slimming can never enjoy a meal without being afraid of ______________ their
diet. (organize)
4. They frequently ______________ the traffic as they march through the streets. (mobile)
5. There will sometimes be silences and pauses in a TV commentary, although these are becoming
______________ rare. (increase)
6. More women than men emphasized ______________ and trust; more men than women
emphasized pleasure in a friend’s company, going out with a friend and having a friend in one’s
home. (confidence)
7. Among the middle-aged men and women, the lack of contact with relatives, even those who lived
nearby, was notable, suggesting that friends and relatives are indeed ____________. (change)
8. There is __________ dissatisfaction with the government’s policies. (wide)
9. Welcome to another edition of the club newsletter. A list of ___________ events for the autumn is
being prepared. (come)

Page 84
The two common ways of infringing upon the copyright are plagiarism and
piracy. Plagiarizing the work of another person means passing it off as one's
own. The word plagiarism is derived from the Latin plagiarus, which means
"abductor." Piracy may be an act of one person but, in many cases, it is a
joint effort of several people who
10. One of the ____________ of this car is its high petrol consumption. (draw)
f. Fill a suitable word in each space to complete the passage.
The Countryside Agency began the process of designating the South Downs as a National Park in
April last year. We believe that being a National Park is the best way to protect the Downs, build on
the achievements of the past to (1) ______________ and enhance the area in its widest sense for future
(2) ______________ . The Downs are under huge and increasing pressure. The South East is one of the
busiest and most pressurised regions (3) ______________ Europe. This means there is (4)
______________ demand for the development of new homes, roads and industry. There are also more
people living and working in the region, which means more need for people to be able to access
beautiful and peaceful countryside to get (5) ______________ from it all. The Downs already
receives around 35 million visits a year: this number is likely to increase as more and more people
live and work in the area.
Designation will bring a body with new ideas and resources specifically focused on (6)
______________ and visitor management, working with others across the whole of the Park to
encourage co-ordination and joint action, and taking action itself where needed. It will be able to
manage the increasing number of visitors so that the Downs themselves are not (7) ______________,
but are still a resource everyone can enjoy. It will also be best placed to protect and enhance the
Downs, so that the qualities so many people love in this special area remain for future generations. We
have two (8) ______________: to identify a boundary for the proposed National Park and to prepare
advice to the Government on the arrangements needed for a South Downs National Park Authority.
The Agency is launching a widespread public consultation in November 2015 which will last for three
months and give all interested organisations and individuals the (9) _________ to comment in detail
on our initial proposals for the boundary and the administrative options for the National Park
Authority. If you would like to receive a copy of the consultation document once it is published then
please contact us (10) _____________ the feedback form.
II. READING (5.2 pts)
a. Read the passage and choose the best option A, B, C or D to answer the questions.
Most forms of property are concrete and tangible, such as houses, cars, furniture, or anything else that is
included in one's possessions. Other forms of property can be intangible, and copyright deals with
intangible forms of property. Copyright is a legal protection extended to authors of creative works, for
example, books, magazine articles, maps, films, plays, television shows, software, paintings,
photographs, music, choreography in dance, and all other forms of intellectual or artistic property.
Although the purpose of artistic property is usually public use and enjoyment, copyright establishes
the ownership of the creator. When a person buys a copyrighted magazine, it belongs to this individual
as a tangible object. However, the authors of the magazine articles own the research and the writing that
went into creating the articles. The right to make and sell or give away copies of books or articles
belongs to the authors, publishers, or other individuals or organizations that hold the copyright. To copy
an entire book or a part of it, permission must be received from the copyright owner, who will most
likely expect to be paid.
Copyright law distinguishes between different types of intellectual property. Music may be played by
anyone after it is published. However, if it is performed for profit, the performers need to pay a fee,
called a royalty. A similar principle applies to performances of songs and plays. On the other hand,
names, ideas, and book titles are excepted. Ideas do not become copyrighted property until they are
published in a book, a painting, or a musical work. Almost all artistic work created before the 20 th
century is not copyrighted because it was created before the copyright law was passed.
reproduce copyrighted material and sell it for profit without paying royalties to the creator.
Technological innovations have made piracy easy, and anyone can duplicate a motion picture on
videotape, a computer program, or a book. Video cassette recorders can be used by practically anyone to
copy movies and television programs, and copying software has become almost as easy as copying a
book. Large companies zealously monitor their copyrights for slogans, advertisements, and brand names,
protected by a trademark.
1. What does the passage mainly discuss?
Page 85
A. Legal rights of property owners
B. Legal ownership of creative work
C. Examples of copyright piracy work
D. Copying creating work for profit
2. Which of the following properties is NOT mentioned as protected by copyright?
A. music and plays B. paintings and maps
C. printed medium D. scientific discoveries
3. It can be inferred from the passage that it is legal if ___________.
A. two songs, written by two different composers, have the same melody
B. two books, written by two different authors, have the same titles
C. two drawings, created by two different artists, have the same images
D. two plays, created by two different playwrights, have the same plot and characters
4. With which of the following statements is the author most likely to agree?
A. Teachers are not allowed to make copies of published materials for use by their students.
B. Plays written in the 16th century cannot be performed in theaters without permission.
C. Singers can publicly sing only the songs for which they wrote the music and the lyrics.
D. It is illegal to make photographs when sightseeing or traveling.
5. The phrase "infringing upon" is closest in meaning to _________.
A. impinging upon B. inducting for
C. violating D. abhorring
6. The purpose of copyright law is most comparable with the purpose of which of the following?
A. A law against theft B. A law against smoking
C. A school policy D. A household rule
7. According to the passage, copyright law is _____________.
A. meticulously observed B. routinely ignored
C. frequently debated D. zealously enforced
b. Read the passage and do the tasks followed
At any given time, more than a million international students around the world are engaged in the study
of the English language in a predominantly English-speaking country. The five most popular
destinations, in order of popularity, are the U. S., Britain, Australia, New Zealand, and Canada. The
reasons for choosing to study English abroad differ with each individual, as do the reasons for the choice
of destination.
Numerous studies conducted in Britain and the United States show that the country of choice depends to
a large extent on economic factors. While this should not provoke much surprise, careful analysis of the
data suggests that students and their parents are most influenced by the preconceptions they have of the
countries considered for study abroad, which, in turn, influence the amount they or their parents are
prepared to outlay for the experience. The strength of international business connections between
countries also gives a good indication of where students will seek tuition. In the main, students tend to
follow the traditional pattern of study for their national group. The United States attracts the most
diverse array of nationalities to its English language classrooms, this heterogeneity being largely due to
its immense pulling power as the world's foremost economy and the resulting extensive focus on U.S.
culture. Furthermore, throughout the non-European world, in Asia and North and South America
especially, the course books used to teach English in most elementary and high schools introduce
students to American English and the American accent from a very early age. Canada also benefits from
worldwide North American exposure, but has the most homogenous group of students - most with
French as their first language. Before furthering their English skills, students in Europe study from
predominantly British English material; most Europeans, naturally, opt for neighboring Britain, but
many Asian, Middle-Eastern, and African students decide upon the same route too.
Australia and New Zealand are often overlooked, but hundreds of thousands of international
students have discovered the delights of studying in the Southern Hemisphere. The majority are Asian
for reasons that are not difficult to comprehend: the proximity of the two countries to Asia, (Jakarta,
the capital of Australia's closest Asian neighbor, Indonesia, is only 5506 kilometers from Sydney), the
comparatively inexpensive cost of living and tuition, and, perhaps of most importance to many Asian
students whose English study is a prelude to tertiary study, the growing awareness that courses at
antipodean universities and colleges are of an exceptionally high standard. In addition, revised entry

Page 86
procedures for overseas students have made it possible for an increasing number to attend classes to
improve their English for alternative reasons.
Australia and New Zealand have roughly the same mix of students in their language classrooms,
but not all students of English who choose these countries are from Asia. The emerging global
consciousness of the late twentieth century has meant that students from as far as Sweden and Brazil
are choosing to combine a taste for exotic travel with the study of English 'down under' and in 'the land
of the long white cloud'. But even the Asian economic downturn in the 1990s has not significantly
altered the demographic composition of the majority of English language classrooms within the region.
Nor have the economic problems in Asia caused appreciable drops in full-time college and university
attendances by Asian students in these two countries. This is partly because there has always been a
greater demand for enrolment at Australian and New Zealand tertiary institutions than places available
to overseas students. In addition, the economic squeeze seems to have had a compensatory effect. It has
clearly caused a reduction in the number of students from affected countries who are financially able to
study overseas. However, there has been a slight but noticeable shift towards Australia and New
Zealand by less wealthy Asian students who might otherwise have chosen the United States for English
study.
The U.S. and Britain will always be the first choice of most students wishing to study the English
language abroad, and it is too early to tell whether this trend will continue. However, economic
considerations undoubtedly wield great influence upon Asian and non-Asian students alike. If student
expectations can be met in less traditional study destinations, and as the world continues to shrink,
future international students of English will be advantaged because the choice of viable study
destinations will be wider.

1-3. Complete the missing information in the table below


U.S Britain Australia New Canada
Zealand
order of 1st 2nd 3rd 4th 5th
popularity
type of English in American (1)……………… (2)……………. not given not given
course books
used in this
country
student 1 2 (3)……………… equal 3 5
heterogeneity
(1 = most
heterogenous
5 = least
heterogenous)

4-9. Tick (√ ) in the right column if the statement is True, False or Not Given in the text

True False Not Given


4. Study destination choices are mostly influenced by proximity to
home.

5. Students who wish to study business will probably study English


overseas.
6. Students of the same nationality usually make similar study
choices.

7. English language classrooms in the U.S. have the widest range of


student nationalities.
8. Standards at Australian and New Zealand tertiary institutions are
improving.
9. Despite the 1990s Asian economic crisis, Asian students still
Page 87
dominate the English language classrooms of Australia and New
Zealand.

c. Read the following passage and choose the best option A, B, C, or D to indicate the correct
word(s) for each of the numbered blanks.
The expression on your face can actually dramatically alter your feelings and perceptions, and it
has been proved that (1) ____ smiling or frowning can create corresponding emotional responses. The
idea was first (2) _______ by a French physiologist, Israel Waynbaum, in 1906. He believed that
different facial (3) _______ affected the flow of blood to the brain, and that this could create positive
or negative feelings. A happy smile or irrepressible laughter increased the blood flow and contributed
to joyful feelings. But sad, angry expressions decreased the flow of oxygen- carrying blood, and
created a vicious (4) _______ of gloom and depression by effectively (5) _______ the brain of
essential fuel.
Psychologist Robert Zajonc rediscovered this early research, and (6) ____ that the temperature of
the brain could affect the production and synthesis of neurotransmitters which definitely influence our
moods and energy levels. He argues that an impaired blood flow could not only deprive the brain of
oxygen, but create further chemical imbalance by inhibiting these vital hormonal messages. Zajonc
goes on to propose that our brains remember that smiling is associated with being happy, and that by
deliberately smiling through your tears you can (7) ____ your brain to release uplifting
neurotransmitters – replacing a depressed condition with a happier one. People suffering from
psychosomatic illness depression and anxiety states could (8) _____ from simply exercising their
zygomatic (9) ____ which pull the corners of the mouth (10) ____ to form a smile, several times an
hour.

1. A. desperately B. determinedly C. deliberately D. decidedly


2. A. put off B. put down C. put by D. put forward
3. A. aspects B. looks C. expressions D. appearances
4. A. cycle B. spiral C. circle D. vortex
5. A. cutting B. starving C. removing D. eliminating
6. A. advises B. wants C. demands D. suggests
7. A. make B. persuade C. give D. decide
8. A. recover B. improve C. benefit D. progress
9. A. muscles B. nerves C. veins D. bones
10. A. to and from B. up and back C. now and then D. up and down

III. WRITING (4.0 pts)


a. Finish each of the sentences in such a way that it means exactly the same as the sentence printed
before it.
1. I am having a lot of trouble now because I lost my passport last week.
If I
_________________________________________________________________________________.
2. The hurricane blew the roof off the house.
The house _________________________________________________________________.
3. He was sentenced to six months in prison for his part in the robbery.
He received a _______________________________________________________________.
4. That rumor about the politician and the construction contract is absolutely false.
There is ____________________________________________________________________.
5. I thought about what had happened all those years before.
I cast _____________________________________________________________________.
6. There aren’t many other books which explain this problem so well.
In few other books __________________________________________________________.
7. Don’t you think we should ask the price?
Hadn’t _____________________________________________________________________________?
Page 88
8. Doris tiptoed up the stairs because she didn’t want to wake anyone up.
To ________________________________________________________________________________
9. I’m sure it wasn’t Mr. Bill you saw because he’s in New York.
It can’t ____________________________________________________________________________.
10. The collision didn’t damage my car much.
Not a great _________________________________________________________________________.
b. Complete the second sentence so that it has a similar meaning to the first sentence, using the word
given. Do not change the word given in any way.
1. Most people regard him as the best man for the job. (widely)
____________________________________________________________________________________
_
2. The suspect could not explain why he had sand on his boots. (account)
____________________________________________________________________________________
_
3. The best solution was thought of by Sally. (came)
____________________________________________________________________________________
_
4. You should consider the fact that he hasn’t spoken French for years. (allowances)
____________________________________________________________________________________
5. He stood no chance of passing his driving test. (inevitable)
____________________________________________________________________________________
_
______The end ______

ENGLISH PRACTICE 22
Part 1: Circle the word whose underlined part is pronounced differently from that of the others in
each group. Circle A, B, C or D to indicate your answer. (5 pts)
1. A. arrow B. loud C. moldy D. poultry
2. A. basilisk B. bison C. basic D. basin
3. A. subtlety B. indebtedness C. bombard D. combing
4. A. benevolent B. content C. molecules D. technique
5. A. conscience B. bronchitis C. shuttle D. chauffeur
Part 2: Choose the word whose syllable is stressed differently from that of the others in each
line. Circle A, B, C or D to indicate your answer. (5 pts)
6. A. comic B. clementine C. climax D. thermonuclear
7. A. diligent B. dimension C. action D. innate
8. A. characterize B. absence C. datum D. charcoal
9. A. solicitor B. separately C. spacious D. sequence
10. A. parachute B. armchair C. accent D. accidentally
II. LEXICO – GRAMMAR
Part 1: Choose the word or phrase that best completes each sentence. Circle A, B, C or D to indicate
your answer. (20 pts)
11. Assembly lines are useful for producing a large _____ of identical products.
A. quality B. quantity C. quandary D. qualification
12. Only the _____ of the building is going to be remodeled.
A. insides B. interior C. indoors D. inner
Page 89
13. Whether the sports club survives is a matter of complete _____ to me.
A. indifference B. disinterest C. importance D. interest
14. After years of neglect there was a huge _____ program to return the city to its former glory.
A. restoration B. preservation C. conservation D.
refurbishment
15. The assistant suggested _____ the next day when the manager would be there.
A. we are coming back B. to come back C. we will come back D. we came
back
16. I never get a _____ of sleep after watching a horror film.
A. wink B. blink C. night D. ounce
17. As it was Christmas, the _____ at church was much larger than usual.
A. audience B. convention C. congregation D. grouping
18. The sheep were huddled into a _____ to protect them from overnight frosts.
A. cage B. kennel C. hutch D. pen
19. The jury _____ the defendant “not guilty”.
A. gave B. returned C. subscribed D. found
20. Many _____ crafts such as weaving are now being revived.
A. customary B. habitual C. traditional D. ordinary
21. He managed to finish his thesis under the _____ of his tutor.
A. guidance B. help C. aid D. assistance
22. Mr. Henry was given a medal in _____ of his service to his country.
A. gratitude B. knowledge C. recognition D. response
23. Everyone knows about pollution problems, but not many people have _____ any solutions.
A. thought over B. come up with C. looked into D. got round to
24. You _____ as well seek for a fish in the tree as try to do that.
A. must B. would C. should D. might
25. _____ calculations have shown that the earth’s resources may run out before the end of the next
century.
A. Raw B. Rude C. Crude D. Blunt
26. By the time you receive this letter, I _____ for China.
A. will have left B. have left C. would have left D. will leave
27. Prizes are awarded _____ the number of points scored.
A. resulting from B. adding up C. presented to D. according to
28. The needs of gifted children in schools have long been _____ neglected.
A. dolefully B. woefully C. idly D. pathetically
29. I must take this watch to be repaired; it _____ over 20 minutes a day.
A. increases B. gains C. accelerates D. progresses
30. It had been a trying afternoon, _____ at about six o’clock in the television breaking down.
A. culminating B. leading C. arriving D. finalizing
Part 2: Write the correct FORM of each bracketed word in the numbered space provided in the
column on the right. (0) has been done as an example. (10 pts)
According to some (0) _____ (SCIENCE), high-risk sports can be particularly (31) scientists
_____ (VALUE) for certain types of people. Such activities help them to learn that 31. ________________
being (32) _____ (FRIGHT) doesn’t mean that they have to lose control. The 32. ________________
recent fashion for jumping from bridges attached to a (33) _____ (LONG) of elastic 33. ________________
rope, known as “bungee jumping”, has now been tried by over one million people 34. ________________
(34) _____ (WORLD) and interest in it is continuing to grow. 35. ________________
Before the special elastic rope (35) _____ (TIGHT) around them, jumpers reach 36. ________________
speeds of nearly 160kph. First-timers are usually too (36) _____ (TERROR) to 37. ________________
open their mouths, and when they are finally (37) _____ (LOW) safely to the 38. ________________
ground, they walk around with broad smiles on their faces, saying (38) _____ 39. ________________
(REPEAT) how amazing it was. However, for some people, it is only the (39)
_____ (EMBARRASS) of refusing to jump at the last minute that finally persuades 40. ________________
them to conquer their fear of (40) _____ (HIGH) and push themselves off into
space.

Page 90
Part 3: The passage below contains 10 mistakes. Underline the mistakes and write their
correct forms in the space provided in the column on the right. (0) has been done as an example. (10
pts)
Traditional, mental tests have been divided into two types. 0. traditional → traditionally
Achievement tests are designed to measure acquiring skills and 41. _____________________
knowledge, particularly those that have been explicitness taught. The 42. _____________________
proficiency exams required by few states for high school graduation 43. _____________________
are achievement tests. Aptitude tests are designed and measure a 44. _____________________
person’s ability to acquire new skills but knowledge. For example, 45. _____________________
vocation aptitude tests can help you decide whether you would do 46. _____________________
better like a mechanic or musician. However, all mental tests are in 47. _____________________
some sense achievement tests because they assumption some sort of 48. _____________________
past learning or experience with certainly objects, words, or 49. _____________________
situations. The difference between achievement and aptitude tests is 50. _____________________
the degree and intention use.
Part 4: Fill in each of the gaps with the correct preposition or particle. Write your answers in
the numbered spaces provided below the passage. (10 pts)
The show was fully booked (51) _____ for weeks, and when it opened last night, the public
poured (52) _____ and very soon the London Arts Center was packed (53) _____. But why? What did
they come to see? They came to see human beings take (54) _____ circus animals, men in cat suits who
stood (55) _____ for real lions and tigers. The show was put (56) _____ by its creators to protest (57)
_____ traditional circuses and to send a message about cruelty to animals. The show was timed to tie
(58) _____ with the National Protection of Animals Week. It was a good idea, but the standard of the
performances was third-rate and an embarrassing number of people simply walked (59) _____ before it
ended. There were some amusing moments when the performers sent (60) _____ typical circus folks, but
overall it was a dismal show. Despite the large turnout for the show’s first night, I doubt it will attract
many people during the rest of its seven-day run.
Part 5: Complete each sentence with the correct form of ONE of the phrasal verbs below.
Write your answers in the numbered spaces provided below the passage. Each verb is used only once.
(10 pts)
try out slip up carry on get by put out
take after get down look up go through turn down
61. If you’re finding it difficult to _____ on your salary, why don’t you ask for a raise?
62. I know what you’re _____ and I feel really sorry for you.
63. In many ways you _____ your father.
64. If you _____ you’ll get into trouble.
65. I proposed to her but she _____ me _____.
66. You’d better _____ your cigarette because smoking isn’t allowed in here.
67. If you _____ working so hard, you’ll make yourself ill.
68. Stop worrying about it. Don’t let this failure ____ you _____.
69. The car’s in quite good condition but you can _____ it _____ before you make any decision to buy,
70. When I was in New York, I was able to _____ several old friends I hadn’t seen for years.
Part 6: Insert A, AN, THE or Ø (zero article) where necessary. Write your answers in the
numbered spaces provided below the passage. (10 pts)
Suddenly (71) _____ blackbird flew to (72) _____ top of (73) _____ beach. She perched way up on (74)
_____ topmost twig that stuck up thin against (75) _____. Then she commenced to sing.
Her little black body seemed only (76) _____ tiny dark speck at that distance. She looked like (77)
_____ old dead leaf. But she poured out her song in (78) _____ great flood of rejoicing through (79)
_____ whole forest. And (80) _____ things began to stir.
III. READING COMPREHENSION
Part 1: Read the following passage and decide which answer (A, B, C or D) best fits each gap.
Circle A, B, C or D to indicate your answer. (10 pts)
The ability to weep is a uniquely human form of emotional response. Some scientists have
suggested that human tears are (81) _____ of an aquatic past – but this does not seem very likely. We
cry from the moment we enter this world, for a number of reasons. Helpless babies cry to persuade their

Page 91
parents that they are ill, hungry or uncomfortable. As they (82) _____, they will also cry just to attract
parental attention and will often stop when they get it.
The idea that having a good cry do you (83) _____ is a very old one and now it has scientific validity
since recent research into tears has shown that they (84) _____ a natural painkiller called enkaphalin. By
fighting sorrow and pain this chemical helps you feel better. Weeping can increase the quantities of
enkaphalin you (85) _____.
Unfortunately, in our society we impose restrictions upon this naturally (86) _____ activity. Because
some people still regard it as a (87) _____ of weakness in men, boys in particular are admonished when
they cry. This kind of repression can only increase stress, both emotionally and physically.
Tears of emotion also help the body (88) _____ itself of toxic chemical waste, for there is more protein
in them than in tears resulting from cold winds or other irritants. Crying comforts, calms and can be very
enjoyable – (89) _____ the popularity of highly emotional films which are commonly (90) _____
“weepies”. It seems that people enjoy crying together almost as much as laughing together.
81. A. witness B. evidence C. result D. display
82. A. evolve B. change C. develop D. alter
83. A. better B. fine C. good D. well
84. A. contain B. retain C. hold D. keep
85. A. construct B. achieve C. provide D. produce
86. A. curing B. treating C. healing D. improving
87. A. hint B. symbol C. feature D. sign
88. A. release B. rid C. loosen D. expel
89. A. consider B. remark C. distinguish D. regard
90. A. named B. entitled C. subtitled D. called
Part 2: Read the following passage and fill the blank with ONE suitable word. Write your
answer in the space provided below the passage. (10 pts)
New technologies, like all technologies, are morally neutral. (91) _____ their advent makes the
world a better place or not depends on the uses to which they are (92) _____. And that, (93) _____ turn,
depends upon the decisions of many people, especially of politicians, managers, trade (94) _____
leaders, engineers and scientists. The new technologies, cheap, flexible, dependent on knowledge and
information as their main input, can (95) _____ human being from many of their current constraints for
example constraints of resources and geography. (96) _____ the new technologies could also (97) _____
those with power to control their fellow citizens even more effectively than in the (98) _____ efficient
dictatorships of the past. The new technological society will (99) _____ colossal demands on our
imagination and ingenuity and on the capacity (100) _____ our institutions to respond to new challenges.
Part 3: Read the following passage and answer the questions from 101 to 110. (10 pts)
Legend has it that sometime toward the end of the Civil War (1861-1865) a government train
carrying oxen traveling through the northern plains of eastern Wyoming was caught in a snowstorm and
had to be abandoned. The driver returned the next spring to see what had become of his cargo. Instead of
the skeletons he had expected to find, he saw his oxen, living, fat, and healthy. How had they survived?
The answer lay in a resource that unknowing Americans lands trampled underfoot in their haste to cross
the “Great American Desert” to reach lands that sometimes proved barren. In the eastern parts of the
United States, the preferred grass for forage was a cultivated plant. It grew well with enough rain, then
when cut and stored it would cure and become nourishing hay for winter feed. But in the dry grazing
lands of the West that familiar bluejoint grass was often killed by drought. To raise cattle out there
seemed risky or even hopeless. Who could imagine a fairy-tale grass that required no rain and somehow
made it possible for cattle to feed themselves all winter? But the surprising western wild grasses did just
that. They had wonderfully convenient features that made them superior to the cultivated eastern
grasses. Variously known as buffalo grass, grama grass, or mesquite grass, not only were they immune
to drought; but they were actually preserved by the lack of summer and autumn rains. They were not
juicy like the cultivated eastern grasses, but had short, hard stems. And they did not need to be cured in a
barn, but dried right where they grew on the ground. When they dried in this way, they remained
naturally sweet and nourishing through the winter. Cattle left outdoors to fend for themselves thrived on
this hay. And the cattle themselves helped plant the fresh grass year after year for they trampled the
natural seeds firmly into the soil to be watered by the melting snows of winter and the occasional rains
of spring. The dry summer air cured them much as storing in a barn cured the cultivated grasses.
101. What does the passage mainly discuss?
Page 92
A. A type of wild vegetation B. Western migration after Civil War
C. The raising of cattle D. The climate of the Western United States
102. What can be inferred by the phrase “Legend has it” in line 1?
A. Most history book include the story of the train.
B. The story of the train is similar to other ones from that time period.
C. The driver of the train invented the story.
D. The story of the train may not be completed factual.
103. The word “they” in line 4 refers to _____.
A. plains B. skeletons C. oxen D. Americans
104. What can be inferred about the “Great American Desert” mentioned in line 7?
A. Many had settled there by the 1860’s.
B. It was not originally assumed to be a fertile area.
C. It was a popular place to raise cattle before the Civil War.
D. It was not discovered until the late 1800’s.
105. The word “barren” in line 7 is closed in meaning to _____.
A. lonely B. uncomfortable C. infertile D. dangerous
106. The word “preferred” in line 8 is closed in meaning to _____.
A. favored B. available C. ordinary D. required
107. Which of the following can be inferred about the cultivated grass mentioned in the second
paragraph?
A. Cattle raised in the Western United States refused to eat it.
B. It had to be imported into the United States.
C. It would probably not grow in the western United States.
D. It was difficult for cattle to digest.
108. Which of the following was NOT one of the names given to the western grasses?
A. Mesquite grass B. Bluejoint grass
C. Buffalo grass D. Grama grass
109. Which of the following was NOT mentioned as a characteristic of western grasses?
A. They contain little moisture B. They have tough stems
C. They can be grown indoors D. They are not affected by dry weather
110. According to the passage, the cattle help promote the growth of the wild grass by_____.
A. eating only small quantities of grass.
B. continually moving from one grazing area to another.
C. naturally fertilizing the soil.
D. stepping on and pressing the seeds into the ground.
IV. WRITING (6/20 points)
Part 1: Finish the second sentence in such a way that it means exactly the same as the
sentence printed before it
111. Customs officials are stopping more travelers than usual this week.
→ An increased ________________
112. I left without saying goodbye as I didn’t want to disturb the meeting.
→ Rather ________________
113. My decision to get up and dance coincided with the band’s decision to stop playing.
→ The moment ________
114. He never suspected that the bicycle had been stolen.
→ At no time ________________
115. How could I help, except to offer to lend her some money?
→ Other ________________
Part 2: Use the word(s) given in the brackets and make any necessary additions to complete a
new sentence in such a way that it is as similar as possible in meaning to the original sentence. Do
NOT change the form of the given word(s).
116. To this day no one has equaled his achievements in the field of technology. (unsurpassed)
→ To this day ………………………………………………………… in the field of technology.
117. Peter grimaced as he swallowed the foul-tasting medicine. (pulled)
→ Peter ……………………………………………………………….the foul-tasting medicine.

Page 93
118. It’s unfortunately that the construction of the building will not be finished as originally planned.
(longer)
→ The construction of the building ………………………………………………………… unfortunate.
119. What he told me made me very curious to hear the rest of the history. (appetite)
→ What he told me ………………………………………………………………………….. the story.
120. They chose not to drive because they thought there would be too much snow. (fear)
→ They chose …………………………………………………………………….…. too much snow.
_____THE END_____

ENGLISH PRACTICE 24
I. PRONUNCIATION
Choose the word whose stress pattern is different from the others in the following questions
1. A.creative B.sensitive C.ambitious D.attentive
2. A.professor B.manager C.reporter D.director
3. A.exercise B.refusal C.collective D.abundant
4. A. material B. maritime C. marathon D. masculine
5. A. conceal B. consul C. contour D. contraband
6. A. participant B. accidental C. parentheses D. industrial
7. A. competent B. compliance C. commuter D. computer
8. A. malevolent B. pasteurise C. satellite D. manicure
9. A. emotional B. sympathetic C. responsible D. indifferent
10. A. continue B. inhabit C. disappear D. imagine

II. LEXICO-GRAMMAR
Part 1 : Choose the word or phrase that best completes each sentence. Write your answer A,
B, C or D in the numbered boxes.
1. ___________ the weather forecast, it will rain heavily later this morning.
A. On account of B. According to C. Because of D. Due to
2. I read the contract again and again __________ avoiding making spelling mistakes.
A. in view of B. in terms of C. with a view to D. by means of
3. It's a shame they didn't pick you up, but it doesn't _______ out the possibility that you might get a job
in a different department.
A. strike B. cancel C. draw D. rule
4. I reckon Mark is ________ of a nervous breakdown.
A. in charge B. under suspicion C. on the verge D. indicative
5. Many local authorities realize there is a need to make _______ for disabled people in their housing
programmes.
A. assitance B. conditions C. admittance D. provision
6. It turned out that we _________ rushed to the airport as the plane was delayed by several hours.
A. hadn't B. should have C. mustn't have D. needn't have
7. All three TV channels provide extensive _________ of sporting events.
A. coverage B. vision C. broadcast D. network
8. No matter how angry he was, he would never ________ to violence.
A. resolve B. recourse C. exert D. resort
9. ________ as a masterpiece, a work of art must transcend the ideals of the period in which it was
created.
A. Ranking B. To be ranked C. Being ranked D. In order to be ranking
10. _________, the people who come to this club are in their twenties and thirties.
A. By and large B. All together C. To a degree D. Virtually
11. I am afraid that you have _________ the deadline, so we can't take your application into account.
A. missed B. met C. delayed D. put off
12. The main aim of the campaign is to raise _______ of the issues involved.
A. knowledge B. awareness C. attention D. acquaintance
13. After so many years, it is great to see him _________ his ambitions.

Page 94
A. get B. realise C. possess D. deserve
14. It was confirmed that the accident was caused by human ________.
A. error B. slip C. fault D. blunder
15. The roadworks made ________ to the hotel from the main road difficult.
A. entrance B. approach C. access D. ways in

Your answers:
1. 2. 3. 4. 5.
6. 7. 8. 9. 10.
11. 12. 13. 14. 15.

Part 2: Write the correct FORM of each bracketed word in the numbered space provided in the
column on the right. (0) has been done as an example.

The origins of Halloween 0.____ western _____


Halloween is celebrated in many parts of the (0)________ (WEST) world, and 1. _________________
is a time when people dress up as witches or ghosts, and go "trick-or treating". 2. _________________
It is (1) _________ (DOUBT) one of the most popular traditions in the United 3. _________________
States and Britain. 4. _________________
The celebration (2) _________(ORIGIN) about two thousand years ago with 5. _________________
the Celts. These people were the (3) ________ (INHABIT) of an area that 6. _________________
includes Britain, Ireland and Brittany. They relied on the land for their (4) 7. _________________
_______ (LIVE), and this meant that they were at the mercy of (5) ________ 8. _________________
(PREDICT) weather conditions, especially during the winter. 9. _________________
The Celtic new year began on 1st November, which also marked the beginning 10. ________________
of winter, a period (6) _________ (TRADITION) associated with death. On
the eve of the new year, it was believed that the barriers between the worlds of
the living and the dead were (7) ________ (TEMPORARY) withdrawn, and it
was possible to communicate with spirits. The Celts believed that the spirits
offered them (8) ________ (GUIDE) and protection, and the Druids (Celtic
priests) were (9) _________ (REPUTE) able to predict the future on this point.
When the Roman completed their (10) ________ (CONQUER) of Celtic
lands, they added their own flavour to this festival. The advent of Christianity
brought about yet other changes.

Part 3: The passage below contains 10 mistakes. UNDERLINE the mistake and WRITE
THEIR CORECT FORMS in the space provided in the column on the right. (0) has been
done as an example.
The horse and carriage is a thing of the past, but love and marriage are still 0. __ interrelated ____
with us and still closely interrelating. Most American marriages, particular 1. ________________
first marriages uniting young people, are the result of mutual attraction and 2. _________________
affection rather with practical considerations. 3. _________________
In the United States, parents do not arrange marriages for their children. 4.__________________
Teenagers begin date in high school and usually find mates through their own 5. _________________
academic and social contacts. Though young people feel free to choose their 6. _________________
friends from different groups, almost choose a mate of similar background. 7. _________________
This is due partly to parental guidance. Parents cannot select spouses for their 8. _________________
children, but they can usually influence choices by voicing disapproval for 9. _________________
someone they consider suitable. 10._________________
However, marriages of members of different groups (interclass, interfaith, and
interracial marriages) are increasing, probably because of the greater mobile of
today's youth and the fact that they are restricted by fewer prejudices as their
parents. Many young people leave their hometowns to attend college, serve in
Page 95
armed forces, or pursue a career in a bigger city. One away from home and
family, they are more likely to date and marry outside their own social group.

Part 4: Fill in the gaps in the following sentences with suitable particles. Write your answers in the
corresponding numbered boxes. (0) has been done as an example.
0. Could you deal ________ this problem. I'm rather busy.
1. The Minister is also implicated __________ the scandal.
2. Irrespective _________ the poor weather conditions the search for the missing child was continued.
3. I have been using her computer ever since she placed it __________ my disposal.
4. I met him at the party and he asked __________ you.
5. You can't miss him. That haircut makes him stand ___________ in a crowd.
Your answers:

0. with 1. 2.
3. 4. 5.

Part 5: Complete each sentence with a correct phrasal verb from the box. Write your
answers in the numbered boxes. Each phrasal verb is used only once.
put down come out see off set about work out
stand for step up track down turn down make up for
1. The managing director __________ the company's poor performance to high interest rates.
2. The police were able to __________ the car thieves using satellite technology.
3. This is a sensitive matter, and we have to __________ dealing with it very carefully.
4. David's new album is expected to ___________ at the end of the year.
5. I hope this award will ____________ your disappointment at not winning the first prize.
6. The company has decided to ___________ production of cars at its factory in Hull.
7. You might need a calculator to __________ this problem.
8. Claire decided to __________ the job, because it would have meant more travelling.
9. Our maths teacher simply won't _________ any talking in class.
10. Helen is going to the airport to _________ some friends.
Your answers:

1. 2. 3. 4. 5.
6. 7. 8. 9. 10.

III. READING

Part 1: Read the following passage and decide which answer (A, B, C, or D) best fits each gap. Write
your answer in the numbered boxes.
The Mysterious Isle
In the early morning of 23 January, 2009, the most powerful storm for a decade hit western France. With
wind speeds in (1) ________ of 120 miles per hour, it flattened forests, (2) ________ down power lines
and caused massive destruction to buildings and roads. But it also left behind an extraordinary creation.
Seven miles out to sea at the (3) ________ where the Atlantic Ocean meets the estuary of the River
Gironde, a small island had (4) _________ out of water. Locals soon gave it the name The Mysterious
Isle. What was so remarkable, (5) ________ its sudden apparition, was the fact that the island (6)
_________ intact in what is often quite a hostile sea environment. It could well become a permanent
feature.
Scientists (7) ________ realised that the island's appearance (8) ________ a unique opportunity to study
the creation and development of a new ecosystem. Within months, it had been colonised by seabirds,
insects and vegetation. Unfortunately, however, they were not alone in (9) ________ the island
attractive. It became increasingly difficult to (10) ________ the site from human visitors. In its first year,

Page 96
day trippers came in powered dinghies, a parachute club used it as a landing strip, a rave party was even
held there one night.

1. A. surplus B. advance C. excess D. put


2. A. fetched B. brought C. carried D. sent
3. A. scene B. mark C. stage D. point
4. A. risen B. growth C. lifted D. surfaced
5. A. in spite of B. instead of C. apart from D. on account of
6. A. prolonged B. remained C. resided D. preserved
7 A. quickly B. briskly C. hastily D. speedily
8. A. delivered B. awarded C. proposed D. offered
9. A. regarding B. finding C. seeking D. deciding
10. A. prevent B. preserve C. protect D. prohibit

Your answers:

1. 2. 3. 4. 5.
6. 7. 8. 9. 10

Part 2: Fill each of the following numbered blanks with ONE suitable word and write your answers in
the
corresponding boxes provided below the passage.

Enjoy the benefits of stress!

Are you looking forward to another busy week? You should be according to some experts. They argue that
the stress encountered in our daily lives is not only good for us, but essential to survival. They say that the
response to (1) _________, which creates a chemical called adrenal in, helps the mind and body to act
quickly (2) ___________ emergencies. Animals and human beings use it to meet the hostile conditions
which exist on the planet.
Whilst nobody denies the pressures of everyday life, what is surprising is that we are yet to develop
successful ways of dealing with them. (3) ________ the experts consider the current strategies to be
inadequate and often dangerous. They believe that (4) ________ of trying to manage our response to stress
with drugs or relaxation techniques, we must exploit it. Apparently, research shows that people (5)
________ create conditions of stress for (6) _______ by doing exciting and risky sports or looking for
challenges, cope much better with life's problems. Activities of this type have been shown to create a lot
of emotion; people may actually cry or feel extremely uncomfortable. But there is a point (7) _________
which they realise they have succeeded and know that it was a positive experience. This is because we
learn through challenge and difficulty. That's (8) _________ we get our wisdom. Few of us,
unfortunately, understand this fact. For example, many people believe they (9) _________ from stress at
work, and take time off as a result. Yet it has been found in some companies that by far (10)
__________ healthiest people are those with the most responsibility. So next time you're in a stressful
situation, just remember that it will be a positive learning experience and could also benefit your health!

Your answers:
1. 2. 3. 4. 5.
6. 7. 8. 9. 10.

Part 3: Read the following passage. For question 1-5, choose the best answer (A, B, C, or D). Write
your answers in the corresponding numbered boxes.

Day after day we hear about how anthropogenic development is causing global warming. According to
an increasingly vocal minority, however, we should be asking ourselves how much of this is media hype
and how much is based on real evidence. It seems, as so often is the case, that it depends on which
expert you listen to, or which statistics you study.

Page 97
Yes, it is true that there is a mass of evidence to indicate that the world is getting warmer, with one of
the world's leading weather predictors stating that air temperatures have shown an increase of just under
half a degree Celsius since the beginning of the twentieth century. And while this may not sound like
anything worth losing sleep over the international press would have us believe that the consequences
could be devastating. Other experts, however, are of the opinion that what we are seeing is just part of a
natural upward and downward swing that has always been part of the cycle of global weather. An
analysis of the views of major meteorologists in the United States showed that less than 20% of them
believed that any change in temperature over the last hundred years was our own fault-the rest attributed
it to natural cyclical changes.

There is, of course, no denying that we are still at a very early stage in understanding weather. The
effects of such variables as rainfall, cloud formation, the seas and oceans, gases such as methane and
ozone, or even solar energy are still not really understood, and therefore the predictions that we make
using them cannot always be relied on. Dr. James Hansen, in 1988, was predicting that the likely effects
of global warming would be a raising of world temperature which would have disastrous consequences
for mankind: "a strong cause and effect relationship between the current climate and human alteration of
the atmosphere". He has now gone on record as stating that using artificial models of climate as a way of
predicting change is all but impossible. In fact, he now believes that, rather than getting hotter, our
planet is getting greener as a result of the carbon dioxide increase, with the prospect of increasing
vegetation in areas which in recent history have been frozen wastelands.

In fact. there is some evidence to suggest that as our computer-based weather models have become more
sophisticated, the predicted rises in temperature have been cut back. In addition, if we look at the much
reported rise in global temperature over the last century, a close analysis reveals that the lion's share of
that increase, almost three quarters in total, occurred before man began to 'poison' his world with
industrial processes and the accompanying greenhouse gas emissions in the second half of the twentieth
century.

So should we pay any attention to those stories that scream out at us from billboards and television news
headlines, claiming that man, with his inexhaustible dependence on oil-based machinery and ever more
sophisticated forms of transport is creating a nightmare level of 'greenhouse gas emissions, poisoning his
environment and ripping open the ozone layer? Doubters point to scientific evidence. which can prove
that, of all the greenhouse gases, only two percent come from man-made sources, the rest resulting from
natural emissions.

Who, then, to believe: the environmentalist exhorting us to leave the car at home, to buy re-usable
products packaged in recycled paper and to plant trees in our back yard? Or the sceptics, including, of
course, a lot of big businesses who have most to lose, when they tell us that we are making a mountain
out of a molehill? And my own opinion? The jury's still out as far as I am concerned!

1. The author __________


A. believe that man is causing global warming.
B. believes that global warming is a natural process.
C. is sure what the causes of global warming are.
D. does not say what he believes the causes of global warming are.

2. As to the cause of global warming, the author believes that _________


A. occasionally the facts depend on who you are talking to.
B. the facts always depend on who you are talking to.
C. often the facts depend on which expert you listen to.
D. you should not speak to experts.

3. More than 80% of the top meteorologists in the United States are of the opinion that _________
A. global warming should make us lose sleep.
B. global warming is not the result of natural cyclical changes but man-made.
Page 98
C. the consequences of global warming will be devastating.
D. global warming is not man-made, but the result of natural cyclical changes.

4. Our understanding of weather __________


A. leads to reliable predictions.
B. is variable.
C. cannot be denied.
D. is not very developed yet.

5. Currently, Dr James Hansen's beliefs include the fact that _________


A. it is nearly impossible to predict weather change using artificial models.
B. the consequences of global warming would be disastrous for mankind.
C. there is a significant link between the climate now, and man's changing of the atmosphere.
D. Earth is getting colder.
Your answers:
1. 2. 3. 4. 5.
For questions 6-10, write in the corresponding numbered boxes with YES, NO, or NOT GIVEN:
Yes if the statement agrees with the information in the passage
No if the statement contradicts the information in the passage
Not given if there is no information about the statement in the passage.

6. At the same time that computer-based weather models have become more sophisticated, weather
forecasters have become more expert.
7. Most of the increase in global temperature happened in the second half of the twentieth century.
8. The media wants us to blame ourselves for global warming.
9. The media encourages the public to use environmentally friendly vehicles, such as electric cars to
combat global warming.
10. Many big businesses are on the side of the sceptics as regards the cause of global warming.
Your answers:

6. 7. 8. 9. 10.

IV. WRITING
Part 1 : Finish each of the following sentences in such a way that it means exactly the same
as the sentence before it .
1. Although Mary was exhausted, she agreed to join in the activity.
→ Exhausted ......................................................................................................
2. We'll let you know as soon as we have received the information.
→ The .................................................................................................................
3. You can only really master a language if you use it regularly.
→ Only by ...........................................................................................................
4. It's nobody's fault that the match was cancelled.
→ Nobody is to .................................................................................................
5. I don't intend to apologise to either of them..
→ I have no ..........................................................................................................

Part 2: Use the word given in brackets and make any necessary additions to write a new sentence in
such a way that it is as similar as possible in meaning to the original sentence. Do NOT change the
form of the given word. You must use between three and six words, including the word given. (0) has
been done as an example.
0.        Fiona refused to wear her old dress. (not)
           Fiona said that ________ she would not wear ________ her old dress.
1. As long as he could see, Kevin really didn't mind where he sat in the stadium. (difference)
→ As long as he could see , ____________________________________ where he sat in the
stadium.
Page 99
2. Somebody should have told us that the date had been changed. (informed)
→ We should ___________________________________ the change of the date.
3. Jane's family persuaded her to enter the competition. (talked)
→ Jane was ____________________________________ the competition by her family.
4. We never imagined that Julian might be planning to resign from his job. (occurred)
→ It never __________________________________ Julian might be planning to resign from his
job.
5. Sally was all ready to leave the office when her boss asked her to type up a report (point)
→ Sally was ________________________________ the office when her boss asked her to type up
a report.

Part 3: Write an essay of about 250 words on the following topic:


"The widespread use of the Internet has brought many problems. What do you think are the main
problems associated with the use of the web? What solutions can you suggest"?

ENGLISH PRACTICE 25
PART 1: PRONUNCIATION
Choose the word whose stress pattern is different from the others in the following question
1. A. emit B. indeed C. belong D. private
2. A. economic B. territorial C. multiracial D. memorial
3. A. compromise B. correspond C. interview D. innocent
4. A. origin B. subsequent C. admirable D. conductor
5. A. favorite B. relatively C. ingredient D. notice
6. A. museum B. commemorate C. commercial D. commentator
7. A. zoology B. conquest C. cement D. duet
8. A. emergency B. vulnerable C. activity D. initiative
9. A. photocopy B. particular C. enthusiasm D. economy
10. A. innocent B. nevertheless C. contents D. supermarket

PART II. GRAMMAR AND VOCABULARY (3.5 PT)


I. Complete the following sentences by choosing the correct answer among four options (A, B, C or
D)
1. Jack _____________ a fortune when his great uncle Jack passed on.
A. made into B. went into C. came into D. bought
2. Telephone service to that remote village can't be ______ this year.
A. provided B. supplied C. improved D. made
3. His company had to close because of_______.
A. redundancy B. economic difficulties
C. subtitles D. a high rate of unemployment
4. I couldn't ______over how well the team play!
A. make B. get C. turn D. put
5. Even if you are good at a game, you shouldn't be______.
A. overconfident B. unconfidentC. confidential D. confidentable
6. “Is it true that you fell asleep in class yesterday?”
“Unfortunately, yes. ________ is unbelievable. I’m very embarrassed.”
A. That I could do such a thing it B. That I could do such a thing
C. I could do such a thing it D. I could do such a thing

Page 100
7. ________ is the biggest city in Michigan, it is not the capital.
A. Detroit B. If Detroit C. Although Detroit D. Detroit, which
8. An almost _______ line of traffic was moving at a snail’s pace through the town.
A. continuous B. constant C. continual D. stopping
9. They were walking on tiptoe ______ the Director's room.
A. pass B. passed C. past D. passing
10. I haven’t been feeling very well ________.
A. of late B. not long ago C. currently D. by now
II. Each line of the following passage has one mistake related to either grammar or vocabulary
usage. Find and correct them.

Air pollution is a cause for ill – health in human beings. It a lot of


countries, there are laws limited the amount of smoke which factories can 0. for --> of
produce. Because there isn't enough information on the amount of smoke in 1. _________
the atmosphere, doctors have proved that air pollution makes lung cancer. 2. _________
The gases from the exhausts of cars have also risen air pollution in most 3. _________
cities. The lead in petrol produces a poisoned gas which often collects in 4. _________
busy streets surrounding by high buildings. Children who live in areas 5. _________
where there is a lot of lead in the atmosphere cannot think as quick as other 6. _________
children and they are clumsy where they use their hands. There are long-term 7. _________
effects of pollution. If the gases in the atmosphere continues to increase, the 8. _________
earth's climate will become warmer. A lot of ice near the Poles may water and 9. _________
may cause serious floods. 10. ________

III. Use the given phrasal verbs to replace the underlined words/phrases in the sentences. Then
put the verbs in the correct form in the sentences.

come into fall through turn in draw up let on


go round do without make out take after turn down

1. Before we do anything else, we ought to prepare a plan of action.


2. It was getting late so I decided to go to bed.
3. I believe that Diana has recently inherited a lot of money.
4. Do you think there is enough food to feed everybody?
5. If we can’t get any bread, we’ll just have to manage.
6. He speaks very badly. I can’t understand what he’s saying.
7. Don’t say anything to the children about the party. I want it to be a promise.
8. I was rejected for the army on the health ground.
9. We’ve made all the arrangements. Let’s hope our plans don’t fail.
10. I resemble my mother. She was small with blond hair and had a terrible memory too.

IV. Use the word given in capitals to form a word that fits in the gap in the sentences. Write your
answers on your answer sheet.
1. I’ll never forget the _________ I felt in the situation. (HUMILIATE)
2. She’s so ________ that she won’t let anything stand in the way of her ambition. (MIND)
3. That was a very ________ thing to say. (HURT)
4. It’s ______ of him to lose his temper like that – he’s usually very calm. (CHARACTER)
5. He stood at the door to make sure that no one _______ the party. (GATE)

PART III. READING

I. Read the passage carefully , then fill in the blank a suitable word.

Page 101
The majority of lottery winners change their lives (1) __________ little, and continue on their
settled way happy ever after. A couple of years ago, a Mr. David Horabin won a million. He had been
struggling to (2) _________ a success of his dry cleaning shop for the past 12 months. He accepted his
cheque in a small ceremony (3) ________ the premises at 2.30, and by three o'clock he had reopened for
business. The reaction of Mr. Pasquale Consalvo who won $30 million in the New York state lottery
was very (4) _________. He was unhappy not to be able to fulfill his desire to go to work as (5)
___________ on the day he won. He also said that if the money made him (6) ____________ he would
give it back. In fact, the chances of his life being made a misery by his new-found wealth are almost (7)
_________ slim though not quite as the sixty million-to-one odds he beat to take a jackpot (8)________
had remained unclaimed through six previous draws. Gambling small amounts (9) __________ the
lottery is a harmless if futile hobby. (10) __________, gambling can become an addiction, increasingly
so as the activity becomes socially acceptable.
II. Read the passage carefully, then choose the correct option (marked A, B, C or D) to answer the
questions.
A recent survey of crime statistics shows that we are all more likely to be burgled now than 20
years ago and the police advise everyone to take a few simple precautions to protect their homes.
The first fact is that burglars and other intruders prefer easy opportunities, like a house which is
very obviously empty. This is much less of a challenge than an occupied house, and one which is well-
protected. A burglar will wonder if it is worth the bother.
There are some general tips on how to avoid your home becoming another crime statistic. Avoid
leaving signs that your house is empty. When you have to go out, leave at least one light on as well as a
radio or television, and do not leave any curtains wide open. The sight of your latest music centre or
computer is enough to tempt any burglar.
Never leave a spare key in a convenient hiding place. The first place a burglar will look is under
the doormat or in a flower pot and even somewhere more 'imaginative' could soon be uncovered by the
intruder. It is much safer to leave a key with a neighbor you can trust. But if your house is in a quiet,
desolate area be aware that this will be a burglar's dream, so deter any potential criminal from
approaching your house by fitting security lights to the outside of your house.
But what could happen if, in spite of the aforementioned precautions, a burglar or intruder has
decided to target your home? Windows are usually the first point of entry for many intruders.
Downstairs windows provide easy access while upstairs windows can be reached with a ladder or by
climbing up the drainpipe. Before going to bed you should double-check that all windows and shutters
are locked. No matter how small your windows may be, it is surprising what a narrow gap a determined
burglar can manage to get through. For extra security, fit window locks to the inside of the window.
What about entry via doors? Your back door and patio doors, which are easily forced open, should
have top quality security locks fitted. Even though this is expensive it will be money well spent. Install a
burglar alarm if you can afford it as another line of defence against intruders.
A sobering fact is that not all intruders have to break and enter into a property. Why go to the
trouble of breaking in if you can just knock and be invited in? Beware of bogus officials or workmen
and, particularly if you are elderly, fit a chain and an eye hole so you can scrutinize callers at your
leisure. When you do have callers never let anybody into your home unless you are absolutely sure they
are genuine. Ask to see an identity card, for example.
If you are in the frightening position of waking in the middle of the night and think you can hear an
intruder, then on no account should you approach the intruder. Far better to telephone the police and
wait for help.
1. According to the writer, we should _______.
A. avoid leaving our house empty
B. only go out when we have to
C. always keep the curtains closed
D. give the impression that our house is occupied when we go out
2. The “aforementioned precautions” refer to steps that _______.
A. will tell a burglar if your house is empty or not
B. are the most important precautions to take to make your home safe
C. will stop a potential burglar
D. will not stop an intruder if he has decided to try and enter your home
3. Gaining entry to a house through a small window _______.
Page 102
A. is surprisingly difficult
B. is not as difficult as people think
C. is less likely to happen than gaining entry through a door
D. is tried only by very determined burglars
4 . According to the writer, window locks, security locks and burglar alarms _______.
A. cost a lot of money but are worth it B. are good value for money
C. are luxury items D. are absolutely essential items
5. The writer argues that fitting a chain and an eye hole _______.
A. will prevent your home being burgled
B. avoids you having to invite people into your home
C. is only necessary for elderly people
D. gives you time to check if the visitor is genuine
III. Choose the word that best fits each of the blanks in the following passage. Circle A, B, C or D
to indicate your answers.
Viewed from the outside (1) ________, the Houses of Parliament look impressive. The
architecture gives the place a traditional look, and the buildings are sandwiched between a busy square
and the river, making them a (2) ________between the country house of an eccentric duke and a
Victorian railway station. You have only to learn that the members (3) ______ to each other as ‘The
Honorable Member to (4) ______ the picture of a dignified gentlemen’s club, with of course a few ladies
to (5) _______ the numbers. Sadly, over the past few years first radio, and now television, have shown
the general public, who are (6) ______ the electorate, what in fact goes on when bills are discussed and
questions are asked. The first obvious fact is that the chamber is very rarely full, and there may be only a
handful of members present, some of whom are quite clearly asleep, telling jokes to their neighbor, or
shouting like badly-behaved schoolchildren. There is not enough room for them all in the chamber in
any (7) _______, which is a second worrying point. Of course, television does not follow the work of
committees, which are the small discussions groups that do most of the real work of the House. But the
(8) ______ impression that voters receive of the workings of government is not a good one. To put it (9)
_______, parliament looks disorganized, is clearly behind the time and seems to be filled with bores and
comedians. This is presumably why members (10) _______ for so long the efforts of the BBC to
broadcast parliamentary matters on television.
1. A. likewise B. at least C. nevertheless D. as well
2. A. mixture B. combination C. cross D. match
3. A. call B. refer C. speak D. submit
4. A. finalize B. end C. conclude D. complete
5. A. take away B. bring about C. make up D. set in
6. A. after all B. anyway C. even D. furthermore
7. A. point B. way C. matter D. case
8. A. total B. broad C. overall D. comprehensive
9. A. bluntly B. shortly C. directly D. basically
10. A. prevented B. checked C. defied D. resisted

PART IV: WRITING


I. Finish each of the following sentences in such a way that it means exactly the same as the
sentence printed before it.
1. The phone stopped ringing the moment I got down stairs.
No sooner _______________________________________________
2. He is determined to carry on working when he is 65.
He has no ________________________________________________
3. He was very sorry that he didn’t see Audrey on her trip to London.
He greatly ______________________________________________
4. She agreed to go out to dinner with him because she assumed he was not married.
Had she __________________________________________________
5. Everyone was surprised that the singer had very little money when he died.
The singer had _____________________________________________

Page 103
II. Write about the following topic:
‘Some people feel that certain workers like nurses, doctors and teachers are undervalued
and should be paid more’
How far do you agree?
Give reasons for your answer and include any relevant examples from your own knowledge or
experience.
You should write at least 150 words.

ENGLISH PRACTICE 26
PART A: PRONUNCIATION
Choose the word whose stress pattern is different from the others in the following question
1. A. argument B. enquiry C. medicine D. justify
2. A. convenience B. mechanic C. preference D. official
3. A. recently B. attitude C. octopus D. proposal
4. A. economy B. advisory C. peninsula D. economics
5. A. arithmetic B. agriculture C. contributor D. assassinate
6. A. magnificent B. appliances C. potentially D. preservation
7. A. computer B. property C. horizon D. imagine
8. A. architect B. electric C. mineral D. luxury
9. A. photochemical B. trigonometry C. dramatically D. alphabetical
10. A. popularity B. conscientious C. apprenticeship D. personality
PART B. LEXICO - GRAMMAR
I. Choose the best answer from the four options (marked A, B, C, or D) to complete the sentences.
1. Of course, I will play the piano at the party but I’m a little ________.
A. out of use B. out of practice C. out of reach D. out of turn
2. Where’s that _______ dress that your grandma gave you?
A. pink, long, lovely, silk B. lovely, long, pink, silk
C. lovely, pink, long, silk D. long, pink, silk, lovely
3. _________ air is essential to man, so is water to fish.
A. As B. Just C. Since D. Like
4. Having selected to represent the Association of American Engineers at the International Convention,
_________.
A. the members applauded him B. a speech had to be given by him
C. the members congratulated him D. he gave a short acceptance speech
5. We _______ wandering about without any food.
A. hungered B. made hungry C. had been hungry D. got hungry
6. James was ________ upset after being rejected in love.
A. strongly B. hardly C. deeply D. highly
7. The company have to _______ways of reducing costs.
A. take in B, think over C. look D. work out
8. Don’t take any _______ of Mike – he’s always rude to everyone.
A. notice B. view C. attention D. sight
9. Before you sign the contract, ________ in mind that you won’t be able to change anything later.
A. hold B. bear C. retain D. reserve
10. Does that name _________ to you?
A. ring a bell B. break the ice C. foot the bill D. fall into place

II. Complete the following passage with the correct forms of the given words.
ACCESS APPEAR COMMIT DENY EXCLUDE
IMMERSE INFANT INSTITUTE LONELY SEE

One of the most challenging aspects of the science anthropology comes from its fieldwork.
Certainly, in its (1)________ as a profession, anthropology was distinguished by its concentration on so-
Page 104
called primary societies in which social (2)________ appear to be fairly limited and social interaction to
be conducted almost (3)_________ face – to – face. Such societies, it was felt, provided anthropologists
with a valuable (4)________into the workings of society that contrasted with the many complexities of
more highly developed societies. There was also a sense that the way of life represented by these smaller
societies were rapidly (5)________and that preserving a record of them was a matter of some urgency.
The (6)________ of anthropologists to the first – hand collection of data led them to some of
the most (7)________ places on earth. Most often they worked alone. Such lack of contact with other
people created feelings of intense (8)________ in some anthropologists, especially in the early stages of
fieldwork. Nevertheless, this process of (9)________ in a totally alien culture continues to attract men
and women to anthropology, and is (10)_________ the most effective way of understanding in depth
how other people see the world.

III. Complete the following sentences with one preposition/particle for each blank.
1. Kate fell _______ with her boyfriend and they stopped seeing each other.
2. She would just sit in her chair, dreaming her life ________.
3. I’ve taken this watch _______ pieces, and now I can’t put it together again.
4. I went to the library, but the book I wanted was out _______ loan.
5. Sorry I’m late. Something cropped _________ at the office.
6. I’m sure my brother will never get married because he hates the feeling of being tied _____.
7. Deborah is going to take extra lessons to catch up______ what she missed why she was away.
8. I don’t think anyone understood what I was saying at the meeting. I failed to get my point _______.
9. Jane’s very modest, always playing ________ her success.
10. Check the bottles carefully to make sure they have not been tampered________.

PART C. READING
I. Read the following passage and choose the options among A, B, C or D that best complete the
blanks.
Stressful situations that (1)________ almost every day in life seem to be unavoidable. However,
we can do little sometimes to avoid a misfortune or an unpleasant occurrence which may (2)________
us unexpectedly as only it can. At such a moment, one may hit the (3)_______, give in to the
helplessness of the situation or, ideally, put a brave face on it trying to (4)_________ the burden.
Can you (5)_________ in your mind an hour spent in a traffic jam, say, this morning? Do you
light one cigarette after another? Do you sound the horn every few seconds like the other neurotics? Or
do you take a different (6)________ and make good use of the time drawing up a schedule for the days
to come? To withstand the stressful moment you can also do a crossword puzzle, listen to your favorite
music or even compose a menu for your Sunday dinner.
In fact, whatever way you (7)________ to the annoying situation, you can exert no impact on it
as the traffic jam will only reduce in due (8)_________. Nevertheless, your reaction might considerably
influence your mood for the rest of the day. The inability to confront a stressful occurrence like that with
a deal of composure and sensibility adds much more strain to your life and in this way puts your well –
being in (9)_________. Surprisingly, it is seemingly negligible hardships we stumble on daily that run
double the risk of developing serious health disorders rather than our isolated tragedies however painful
they may be. (10)_______that so many of those wretched stresses and inducing troubles affect us in a
day, we should, at best, try to avoid them or possibly make radical alterations in the way we lead our
daily lifestyles.
1. A. devise B. create C. originate D. emerge
2. A. arise B. happen C. befall D. occur
3. A. post B. roof C. bottom D. wall
4. A. subsist B. remain C. cow D. bear
5. A. envision B. observe C. picture D. image
6. A. manner B. stance C. practice D. mode
7. A. strike B. deal C. respond D. challege
8. A. term B. course C. timing D. period
9. A. risk B. weakness C. insecurity D. jeopardy
10. A. Providing B. Given C. Hence D. As much

Page 105
II. Read the text and then answer the questions 1 – 5 by choosing A, B, C or D.
Federal Express is a company that specializes in rapid overnight delivery of high-priority
packages. The first company of its type, Federal Express was founded by the youthful Fred Smith in
1971, when he was only 28 years old. Smith had actually developed the idea for the rapid delivery
service in a term paper for an economics class when he was a student at Yale University. The term paper
reputedly received a less – than – stellar grade because of the infeasibility of the project that Smith had
outlined. The model that Smith proposed had never been tried; it was a model that was efficient to
operate but at the same time was very difficult to institute.
Smith achieved efficiency in his model by designing a system that was separate from the
passenger system and could, therefore, focus on how to deliver packages most efficiently. His strategy
was to own his own planes so that he could create his own schedules and to ship all packages through
the hug city of Memphis, a set – up which resembles the spokes on the wheel of a bicycle. With this
combination of his own planes and hub set – up, he could get packages anywhere in the United States
overnight.
What made Smith’s idea difficult to institute was the fact that the entire system had to be created
before the company could begin operations. He needed a fleet of aircraft to collect packages from
airports every night and deliver them to Memphis, where they were immediately sorted and flown out to
their new destinations; he needed a fleet of trucks to deliver packages to and from the various airport; he
needed facilities and trained staff all in place to handle the operation. Smith had a $4 million inheritance
from his father, and he managed to raise an additional $91 million dollars from venture capitalists to get
the company operating.
When Federal Express began service in 1973 in 25 cities, the company was not an immediate
success, but success did come within a relatively short period of time. The company lost $29 million in
the first 26 months of operations. However, the tide was to turn relatively quickly. By late 1976, Federal
Express was carrying an average of 19,000 packages per night and had made a profit of $36 million.

1. The word developed in paragraph 1 could be replaced by


A. come up with B. come about C. come across D. come into
2. What was a key idea of Smith’s?
A. That he should focus on passenger service.
B. That package delivery should be separate from passenger service.
C. That packages could be delivered on other companies’ planes.
D. That passenger service had to be efficient.
3. A hug city in paragraph 2 is
A. a large city with small cities as destinations.
B. a city that is the final destination for many routes.
C. a city where many bicycle routes begin.
D. a centralized city with destinations emanating from it.
4. It can be inferred from the passage that Smith selected Memphis as his hub city because it
A. was near the middle of the country.
B. had a large number of passenger aircraft
C. already had a large package delivery service.
D. was a favorite passenger airport.
5. Which paragraph explains what made Smith’s model effective?
A. The first paragraph B. The second paragraph
C. The third paragraph D. The last paragraph

III. Complete the following passage by filling one word for each blank.
If there is just one single thing more astonishing than the ability of the adult human being to talk,
it is the process by which someone learns to do this. Some parts of the process are still (1)________
much a closed book, but it is for the (2)_________ part possible to describe what the child is doing at
various stages in its development, even if we cannot account (3)________ how exactly it learns to do
these things.
In fact, research carried out by various linguists has (4)_________ rise to as many theories as
there are differences in the rate of development. A baby actually makes sounds from the moment it is
born, but for some time these are rather far removed (5)_________ articulate speech. In something like a
Page 106
year, a baby will probably be at a stage where one or two syllables represent the peak of its achievement
as a speaker; one more year and it will be (6)_______ out with short phrases, and after this it seems
(7)________ time at all before the child is capable of uttering complete sentences.
Despite being a truly remarkable feat of learning, this is one that is performed by the vast
(8)_________ of human beings. Complex operations are brought (9)________ play in these dealing with
speech and language; the key (10)_______ in brain work, though tongue – work and ear – work play a
part in the whole process.

PART D. WRITING
I. Rewrite each sentence so that it contains the word/phrase in capitals, and so that the meaning stays
the same.
1. Please excuse Jane’s poor typing; she’s only been learning for a month. ALLOWANCES
………………………………………………………………………………………………………………
2. This is the procedure. FOLLOWS
……………………………………………………………………………………………………………
3. I just didn’t know what to say. LOST
………………………………………………………………………………………………………………
4. Alison bought the big house because she wanted to open a hotel. VIEW
………………………………………………………………………………………………………………
5. We decided to spend the afternoon exploring the shop LOOKING
…………………………………………………………………………………………………

II. With 250-300 words, write about the following topic:

Increasing the price of petrol is the best way to solve growing traffic and pollution problems.

To what extent do you agree or disagree? What other measures do you think might be effective?
Give reasons for your answer with relevant examples from your knowledge and experience.

ENGLISH TEST NO. 27


I. VOCABULARY AND GRAMMAR
Part 1: Choose the word or phrase which best completes each sentence. (15 points)
1. These figures give you some idea of the cost of ….. your car for one year.
A. controlling B. handling C. managing D. maintaining
2. It can take up to three months to ….. a man to do this specialist work.
A. guide B. raise C. train D. learn
3. In today’s paper it ….. that we shall have an election this year.
A. says B. admits C. expresses D. proposes
4. I had to pay ….. on a carpet I bought in through the Customs today.
A. taxes B. rates C. fines D. duty
5. Tropical diseases are comparatively ….. in Europe.
A. scarce B. rare C. slight D. few
6. The ….. charged by the architect for the plans of the new building were unusually high.
A. hire B. price C. fees D. sum
7. It takes a great deal of ….. for the class to make a trip abroad.
A. arrangement B. organisation C. expense D. business
8. You shouldn’t eat so many sweets. They’re ….. for you.
A. bad B. unhealthy C. unsuitable D. disagreeable
9. I wondered whether you would like to ….. to the theatre tomorrow.
A. visit B. go away C. go out D. walk out
10. Do you think he is ….. of doing the job?
A. capable B. competent C. able D. suited
Page 107
11. I bought these shoes in the sale. They were a real ……….
A. cheap B. economy C. bargain D. purchase
12. If you put your money in the bank, it will earn ten per cent ……….
A. investment B. profit C. deposit D. interest
13. Edward was named after one of his father ‘s distant ……….
A. family B. brothers C. members D. relations
14. Jane and Brian got married a year after they got ……….
A. divorced B. proposed C. engaged D. separated
15. Julie had a terrible ………. with her parents last night.
A. row B. discussion C. argue D. dispute
Part 2: Read the text and use the word given in capitals at the end of each line to form a word that
fits in the space in the same line. (10 points)
Margaret started English Literature this term, and I’m afraid that her (1) INTRODUCE
…………… to the subject has not bee entirely (2) SUCCESS …………….. She has not show much
enthusiasm and does not always pay (3) ATTEND ……......… in class. Her assignments are often (4)
READ…......… , because she is so untidy, and because of her (5) FAIL.....… to check her work
thoroughly. She failed to do any (6) REVISE ...…...… before the end of term test, and had poor results.
She seems to have the (7) MISTAKE ……....… idea that she can succeed without studying. She has
also had many (8) ABSENT..…......… and has frequently arrived late for class. This has resulted in
several (9) PUNISH…...... . Although Margaret is a (10) GIFT ...…..… student in some respects, she
has not had a satisfactory term.
Part 3: Identify the one underlined word or phrase that must be changed in order for the
sentence to be correct. Circle your answers. (5 points)
1. We sent a present for the children living next door.
(A) (B) (C) (D)
2. Penny took three exams and managed to succeed them all.
(A) (B) (C) (D)
3. Items of luggage whose weight exceed 50 kilograms will not be allowed on the ferry.
(A) (B) (C) (D)
4. The unemployment rate has increased considerably since last year. Another two millions are jobless.
(A) (B) (C) (D)
5. The President often contradicts himself. He often says something which doesn’t agree with what he
says earlier.
(A) (B) (C)
(D)

Part 4: The following sentences are badly constructed. Rewrite them in better style and correct
any grammatical errors. (5 points)
1. All students must pay their fees, except foreigners, to the university bursar.

...........................................................................................................................................................
2. Swimming is a sport to any healthy person I would recommend.

...........................................................................................................................................................
3. The president said that the new education programme was essential addressing a large audience in
the provincial capital yesterday.

...........................................................................................................................................................
4. Talks have begun to plan a railway linking the provincial capitals between the representatives of
the provincial assemblies.
Page 108
...........................................................................................................................................................
5. The advertisement said that they wished to employ a secretary for an expanding company with
good shorthand and typing speeds at their head office.

...........................................................................................................................................................

II. READING 
Part 1: Read the passage and choose the correct answer for the following questions:
All at once Hazel was coming in through the French windows, pulling off gardening gloves, and Bill
was entering through the door, both at once. So I only had time to take one quick look at her before I
turned to face him. All very confusing. What that first glimpse showed me was that time had thickened
her figure but didn’t seem to have made much difference to her face. It still had good skin and youthful
outlines. She was holding a bunch of roses – must have been cutting them in the garden while waiting
for me. The gardening gloves lent a delightfully informal touch. It was quite an entrance, though Bill
spoilt it a bit by making his at the same time.
Bill seemed longer and thinner. His tightly massed hair had a tinge of grey. Apart from that, twenty
years had done nothing to him, except deepen the lines of thoughtfulness that had already, when I knew
him, begun to spread across his face. Or was that all? I looked at him again, more carefully, as he looked
away from me at Hazel. Weren’t his eyes different somehow? More inward looking than ever? Gazing
in not merely at his thoughts, but at something else, something he was keeping hidden or perhaps
protecting.
Then we were chattering and taking glasses in our hands, and I came back to earth. For the first ten
minutes we were all so defensive, so carefully probing, that nobody learnt anything. Bill had forgotten
me altogether, that much was clear. He was engaged in getting to know me from scratch, very cautiously
so as not to hit a wrong note, with the object of getting me to contribute a big subscription to his African
project. I kept trying to absorb details about Hazel, but Bill was talking earnestly about African
education, and the strain of appearing to concentrate while actually thinking about his wife proved so
great that I decided it would be easier just to concentrate. So I did. I let him hammer away for about ten
more minutes, and then the daughter, who seemed to be acting as parlourmaid, showed in another
visitor. Evidently we were to be four at lunch.
1. What effect had time had on Hazel and Bill?
A. They had both lost weight. B. They were more withdrawn.
C. They hadn’t changed at all. D. They had changed in subtle ways.
2. When they all started talking, the writer
A. relaxed at last. B. stopped dreaming.
C. spoke most to Hazel. D. began to remember things.
3. The writer found the first part of their conversation
A. sentimental. B. irritating. C. uninformative. D. trivial.
4. Why did Bill speak seriously?
A. He wanted some money from the writer. B. He did not remember the writer.
C. His wife was present. D. He was talking about the past.
5. In the end the writer found Bill’s conversation
A. monotonous B. convincing C. thought-provoking D. instructive
Part 2: Read the following passage and choose the most suitable word for each space.

According to a magazine article I read recently, we (1) ......... live in an age of increasing leisure.
Not only are more and more people reaching (2) ......... age with their taste for enjoyment and even
adventure relatively (3) ......... but the working week is becoming shorter and the opportunities for
(4) ......... are becoming greater and greater all the time. Not to mention the fact that people (5) .........to
Page 109
spend less time travelling to work or may even be working from home. What I can't understand,
however, is who these people are. As far as I can (6) ......... the whole thing is another one of (7) .........
journalistic fictions. I admit that there are a lot of retired people (8) ......... but I am not sure that all of
them are dashing about learning hang-gliding or sailing single-handed (9) ......... the world. My own
parents seem to (10) ......... most of their time gazing at the television. And as for the shorter working
week, I wish someone (11) .........remind my company about it. I seem to be working longer and longer
hours (12) ......... the time. The little leisure time I have is eaten into by sitting in the traffic jams or
waiting for trains to (13) ......... up at rain-swept platforms. I haven't noticed any dramatic improvements
in my (14) ......... either, but perhaps I just have to wait until I get my (15) .........

1. A. presently B. at the moment C. now D. at this time


2. A. retirement B. their C. later D. third
3. A. present B. survived C. free D. intact
4. A. this B. longer C. leisure D. people
5. A. use B. tend C. have D. demand
6. A. concern B. imagine C. expect D. tell
7. A. the B. those C. these D. some
8. A. in our days B. in these times C. nowadays D. now and again
9. A. round B. over C. through D. into
10. A. have B. use C. the D. spend
11. A. would B. to C. had D. might
12. A. at B. for C. take D. all
13. A. keep B. line C. show D. set
14. A. cost of living B. lifestyle C. lifeline D. livelihood
15. A. pension B. retirement C. insurance D. salary
Part 3: Read the text and fill in each blank with ONE suitable word.
Our classes take place for three hours every morning from Monday to Friday. The maximum
class size is twelve (1) .................... the average is ten. We use modern methods of (2) .................... and
learning, and the school has a language laboratory, a video camera and recorders. You will only be
successful in improving (3) .................... English, however, if you work hard and (4) ..................
speaking English as much as you can. You will take a short (5) ................... in English as soon as you
arrive. In this way, we can put you in a (6) .................... at the most suitable level.
There are two classes at the Elementary level; one is for complete (7) ..................... and the other
is for students who know only a little English, in both classes you will practise simple conversations. In
the class (8) ..................... the intermediate level you will have a lot of practice in communication in real-
life situation because we help you to use the English you have previously (9) ................... in your own
country, You will also have the chance to improve your (10) ................... of English grammar and to
build up your vocabulary...

III. PRONUNCIATION
Part 1: Choose the word whose underlined part is pronounced differently from that of the others.
(05 points)
1. a. technical b. change c. much d. exchange

2. a. weapon b. increase c. threat d. spread

3. a. requirements b. subject c. secondary d. levels

4. a. prohibit b. exhibit c. habit d. hunting

Page 110
5. a. elephants b. decades c. poachers d. actions

Part 2: Choose the word whose main stress pattern is not the same as that of the others.

1. a. continent b. however c. elephant d. tropical

2. a. represent b. intensive c. domestic d. employment

3. a. social b. proportion c. industry d. easily

4. a. habitat b. century c. difficult d. prohibit

5. a. generally b. secondary c. education d. specialize

III. WRITING
Part 1: Finish each of the following sentences in such a way that it is as similar as possible in
meaning to the sentence printed before it. 1. It's a long time since we spoke to your sister.
We ............................................................................................................................. .
2. "Why don't you leave now. That's what I'd do."
If ............................................................................................................................... .
3. Jill was the only person who came late.
Everyone ................................................................................................................... .
4. This matter is so complicated that we don't know how to deal with it.

So ............................................................................................................................................................
.................................. .
5. It's a pity you didn't ask us to spend the time with you.
If
only .........................................................................................................................................................
........................... .

Part 2: For each of the sentences below, write a new sentence as similar as possible in meaning to
the original sentence, but using the word given. This word must not be altered in any way.
1. I'm afraid that we haven't got any eggs left. (run)
................................................................................................................... .
2. Let me tell you what I think you should do. (advice)
................................................................................................................... .
3. The numbers of cars on the roads must be reduced. (down)
................................................................................................................... .
4. Come on Thursday or Friday. It' s all the same to me. (difference)
................................................................................................................... .
5. Amanda has improved a lot this term. (progress)
................................................................................................................... .

ENGLISH TEST 28

PART A: PHONETICS (5 pts)

Page 111
Question I: Choose the word whose underlined part is pronounced differently from the rest in the
same line. (2 pts)
1. A. genuine B. guy C. generate D. geneticist
2. A. breath B. spread C. break D. headline
Question II: Find the word with the stress pattern different from that of the other three words in
each question. (3 pts)
3. A. briefcase B. journalism C. calculate D. apply
4. A. redundant B. terrorism C. unique D. officer
5. A. bigoted B. perpetrate C. relativism D. picturesquely

Part B : LEXICAL AND GRAMMAR ( 45 pts )


Question I: Choose the most suitable word or phrase to complete each sentence. (15 pts)
6. They came to inspect the house _________ buying it.
A. in the event of B. with reference to C. with a view to D. on account of
7. The boy who failed the exam has to take another one, ?
A. did he B. hasn’t he C. didn’t he D. doesn’t he
8. It is _______ knowledge in the village that Mr. and Mrs. Thorne quarrel violently several time a
week.
A. common B. complete C. normal D. usual
9. _________ passenger pigeon, one of several species of extinct birds, was hunted to extinction over
_________ few decades.
A. The / Ø B. Ø / the C. The / a D. A / the
10. Don’t be angry with Sue. All that she did in good______
A. hope B. belief C. idea D. faith
11. Mrs. Brown always ______ in a crowd because she wore large hats.
A. found against B. looked up C. stood out D. showed up
12. Mary bought______ hat yesterday.
A. a red big plastic hat B. a big red plastic hat
C. a plastic big red hat D. a bit plastic red hat
13. ______ the two sisters, Mary is _______.
A. Of/ the prettier B. Between/ the prettiest
C. Of/ prettier D. Between/ the prettier
14. Helen: “I love your gardens. The plants are well taken care of!”
Ingrid: “Thanks. Yes, I suppose I’ve always __________.”
A. had green fingers B. let nature take its course
C. made it down on my luck D. drawn the short straw
15. My old riding boots served me well for eleven years before they finally ________.
A. wore off B. broke downC. wore out D. broke up
16. _______ of birds over a city usually predict cold weather.
A. Herds B. Packs C. Flocks D. Groups
17. We might have to change our plans for the weekend and, _______ we’ll let you know as soon as
possible.
A. in case B. if so C. providing D. supposing
18. It’s one of the worst books I’ve ever read. Its only redeeming ___________ is that it’s quite short.
A. aspect B. element C. feature D. factor
19. - Well, I’m sorry, that’s all I can offer you. - ____________.
A. Take it or forget it B. Get it or forget it
C. Take it or leave it D. Leave it or take it
20. He has been unable to find a job _______ with his ability as an accountant.
A. appropriate B. suitable C. requisite D. commensurate

Question II: The following passage contains 10 mistakes. Find and correct them (10 pts)
Too many parents have a tough time get clear and accessible 21
information about the public schools at their communities. That is why
22

Page 112
President Clinton had announced an initiative requiring all states to 23
produce annual reports cards that are easily understood by and widely 24
distributing to parents and the public, for each school, school district
and the state as a whole. The report cards will include information to 25
student achievement, teacher professionally qualifications, class size, 26
school safety and other factors that will help parents judging the 27
overall performance of the schools. President Clinton’s proposal will
help ensure which parents in every state have access to the information 28
they need to determine the quality of their schools and identity areas in 29
which improvement is needed. 30

Question III: Put each verb in brackets into an appropriate form. (10 pts)
31. It is essential that the plan (inform) ________to everyone in advance.
32-33. I realized that someone (steal) ___ my wallet when I (feel) ____ their hand in my jacket
pocket.
34-35. At school I (dislike) _______ the Chemistry teacher because she (always pick) _______ on
me.
36. Fortunately, the hospital's new air-conditioning system (install) _________ when the first heat
wave of the summer arrived.
37. The cake (make) _______________ by my mom tastes really delicious.
38. She was breathing fast and deep, as if she (run) _______.
39. I can't find my book anywhere. I (leave) ______ it on the train. I am not sure.
40. No wonder he was sacked! He seems (fiddle) ________ the accounts for years.

Question IV: Put each word in brackets into an appropriate form. (10 pts)
41. Many people make their living on theater. In essence the theater is not only for LIVE
entertainment, it is a _____ .
42. They never dare to leave their only child ________ for even a moment. ATTEND
43. What I don’t like about school uniform is that it completely destroys all_______. INDIVIDUAL
44. The main reason I believe children shouldn’t be exposed to violence on TV is that IMPRESS
they are so__________ at that age.
45. His ________ of his opponent led to his biggest failure. ESTIMATE
46. The curriculum is ________developed; therefore, students benefit a great deal. SYSTEM
47. The mother is usually the homemaker and the father is the _______. BREAD
48. There was a donation of $100,000 made by an anonymous _________. BENEFIT
49. Being the hardest natural substance, diamond is practically_________. DESTROY
50. The government’s ________ approach has brought criticism. COMPROMISE

PART C: READING COMPREHENSION (30 pts)

Question I: Read the following passage and then choose the most suitable word or phrase for each
space. (10 pts)
The money that some professional sportsmen earn shouldn’t impress anyone when you take into
(51)__ __the fact that only a few of them manage to attain immortality and everlasting fame. And once
they reach their (52) __and display their talent at their best, they are fully conscious that their brilliant
careers won’t last forever. They live under a constant pressure of being (53) and subsequently replaced
by someone who is younger, faster and more accomplished. For that reason, objectives like retirement
benefits and pensions are (54)_______great concern to all professional athletes.
Some of the retired competitors go as far as to organize strikes and rallies to voice their protest
against any policy unresponsive to their demand (55)_____the younger professionals seek more
upgrading solutions to the problem as more and more of them attach a proper significance to (56)___a
solid education, even at university level. Such an approach should help them find interesting and well-
paid jobs (57)____their sports career is over.
Page 113
A completely new strategy has been devised by the schools priding themselves
(58)_____supporting their own teams. Their authorities insist that the sports clubs members achieve high
academic standards or else they are debarred from partaking in certain sports events, which may lead to
further disruption in their professional careers.
By these practical and most effective (59)___, combining education with sports activity, the
(60)___of the professional athlete as being brainless and unintelligent may eventually be changing to the
sportsmen’s benefit.
51. A. reflection B. attention C. examination D. consideration
52. A. prime B. shape C. best D. capacity
53. A. outcast B. outshone C. outstayed D. outgrown
54. A. with B. in C. at D. of
55. A. whereby B. whereas C. whereupon D. wherein
56. A. mastering B. learning C. receiving D. attending
57. A. right away B. promptly C. barely D. once
58. A. with B. on C. for D. in
59. A. grounds B. results C. factors D. means
60. A. vision B. outlook C. image D. judgment

Question II: Supply the most suitable word for each blank. (10 pts)
In the 21st century food will (61)______ more than just you feed you. A new range of products
appearing on shelves in shops and supermarkets (62) ______ designed to give you specific health
benefits. The demands of modern life make these foods very attractive. Not only do they provide proven
ways to improve health, but they are also very attractive (63) ______ a quick and convenient way of
making sure we enjoy a healthy diet.
In some countries it is already possible to buy crisps that make you feel (64) ______ depressed,
chewing gum that increases your brain power and tea that helps you (65) ______ over the tiredness
associated (66) ______ long-distance air travel. In the future, experts promise biscuits that will keep you
healthy, and hot chocolate drink to give you strong bones.
Despite the fact that these “functional” foods cannot replace a balanced diet and regular exercise,
they can help the body perform at (67) ______ best a lot of the time. At (68) ______, these foods are
more expensive than other foods, but that is due to the ingredients they (69) ______ of and the way they
are made. All the foods contain probiotics (70) ______ increase the number of “good” bacteria in your
stomach, helping to keep your digestive system healthy.

Question III: Read the passage and choose the best answers to questions below. (10 pts)
The Digital Divide
Information technology is influencing the way many of us live and work today. We use the Internet
to look and apply for jobs, shop, conduct research, make airline reservations, and explore areas of
interest. We use e-mail and the Internet to communicate instantaneously with friends and business
associates around the world. Computers are commonplace in homes and the workplace.
Although the number of Internet users is growing exponentially each year, most of the world’s
population does not have access to computers or the Internet. Only 6 percent of the population in
developing countries are connected to telephones. Although more than 94 percent of U.S. households
have a telephone, only 42 percent have personal computers at home and 26 percent have Internet access.
The lack of what most of us would consider a basic communications necessity – the telephone – does not
occur just in developing nations. On some Native American reservations only 60 percent of the residents
have a telephone. The move to wireless connections may eliminate the need for telephone lines, but it
does not remove the barrier to equipment costs.
Who has Internet access? Fifty percent of the children in urban households with an income over
$75,000 have Internet access, compared with 2 percent of the children in low-income, rural households.
Nearly half of college-educated people have Internet access, compared to 6 percent of those with only
some high school education. Forty percent of households with two parents have access; 15 percent of
female, single-parent households do. Thirty percent of white households, 11 percent of black
households, and 13 percent of Hispanic households have access. Teens and children are the two fastest-
growing segments of Internet users. The digital divide between the populations who have access to the
Internet and information technology tools is based on income, race, education, household type, and
Page 114
geographic location. Only 16 percent of the rural poor, rural and central city minorities, young
householders, and single parent female households are connected.
Another problem that exacerbates these disparities is that African-Americans, Hispanics, and Native
Americans hold few of the jobs in information technology. Women hold about 20 percent of these jobs
and are receiving fewer than 30 percent of the computer science degrees. The result is that women and
members of the most oppressed ethnic groups are not eligible for the jobs with the highest salaries at
graduation. Baccalaureate candidates with degrees in computer science were offered the highest salaries
of all new college graduates in 1998 at $44,949.
Do similar disparities exist in schools? More than 90 percent of all schools in the country are wired
with at least one Internet connection. The number of classrooms with Internet connections differs by the
income level of students. Using the percentage of students who are eligible for free lunches at a school
to determine income level, we see that nearly twice as many of the schools with more affluent students
have wired classrooms as those with high concentrations of low-income students.
Access to computers and the Internet will be important in reducing disparities between groups. It will
require greater equality across diverse groups whose members develop knowledge and skills in
computer and information technologies. If computers and the Internet are to be used to promote equality,
they will have to become accessible to populations that cannot currently afford the equipment which
needs to be updated every three years or so. However, access alone is not enough. Students will have to
be interacting with the technology in authentic settings. As technology becomes a tool for learning in
almost all courses taken by students, it will be seen as a means to an end rather than an end in itself. If it
is used in culturally relevant ways, all students can benefit from its power.
71. Why does the author mention the telephone in paragraph 2?
A. To demonstrate that even technology like the telephone is not available to all
B. To argue that basic telephone service is a first step to using the Internet
C. To contrast the absence of telephone usage with that of Internet usage
D. To describe the development of communications from telephone to Internet
72. Which of the sentences below best expresses the information in the statement “Although the number
............. or the Internet.” in the paragraph 2?
A. Most of the people in the world use the Internet now because the number of computers has been
increasing every year.
B. The number of people who use computers and the Internet is increasing every year, but most
people in the world still do not have connections.
C. The number of computers that can make the Internet available to most of the people in the world
is not increasing fast enough.
D. The Internet is available to most of the people in the world, even though they don't have their own
computer terminals.
73. The word “eliminate” in the passage is closest in meaning to
A. accept B. dispute C. define D. remove
74. Based on information in paragraph 3, which of the following best explains the term "digital divide?"
A. The number of Internet users in developing nations
B. The disparity in the opportunity to use the Internet
C. Differences in socioeconomic levels among Internet users
D. Segments of the population with Internet access
75. Why does the author give details about the percentages of Internet users in paragraph 3?
A. To prove that there are differences in opportunities among social groups
B. To argue for more Internet connections at all levels of society
C. To suggest that improvements in Internet access are beginning to take place
D. To explain why many people have Internet connections now
76. According to paragraph 3, which of the following households would be least likely to have access to
the Internet?
A. A household with one parent B. A black household
C. A Hispanic household D. A household with both parents
77. The word “those” in the passage refers to
A. classrooms B. students C. schools D. concentrations
78. According to paragraph 4, why are fewer women and minorities employed in the field of computer
technology?
Page 115
A. They are not admitted to the degree programs.
B. They do not possess the educational qualifications.
C. They do not have an interest in technology.
D. They prefer training for jobs with higher salaries.
79. The word “concentrations” in the passage is closest in meaning to
A. protections B. numbers C. confidence D. support
80. What can be inferred from paragraph 6 about Internet access?
A. Better computers need to be designed.
B. Schools should provide newer computers for students.
C. The cost of replacing equipment is a problem.
D. Technology will be more helpful in three years.

PART D: WRITING (20 pts)


Question I: Finish each of the following sentences in such a way that it is as similar as possible in
meaning to the sentence printed before it. (5 pts)
81. There were not nearly as many people there as I had expected.
à There were far ______________________________________________________.
82. People say that the plane of Germanwings crashed into the mountains.
à The plane of Germanwings ____________________________________________.
83. It's nobody's fault that the meeting was cancelled.
à Nobody ___________________________________________________________.
84. The President is the statesman I admire most of all.
à There is ___________________________________________________________.
85. It was his lack of confidence that surprised me.
What ______________________________________________________________.

Question II: Finish each of the following sentences in such a way that it is as similar as possible in
meaning to the sentence printed before it. Do not change the form of the given word. (5 pts)
86. The manager should think about experience when hiring new staff.
CONSIDERATION
 __________________________________________________________________.
87. He is determined to become a doctor. HEART
 __________________________________________________________________.
88. I expect the book to be far better because it had been written by such a good novelist.
SHORT
 __________________________________________________________________.
89. He never felt so emotional when he looked at a picture. TIME
 __________________________________________________________________.
90. I’m not very keen on gold, I much prefer silver. RATHER
 __________________________________________________________________.

Question III. Essay writing (10 pts):


Do you agree or disagree with the following statement: “Children should begin learning a
foreign language as soon as they start school.”?
Write an essay ( about 250 to 300 words) to express your personal point of view.

ENGLISH PRACTICE 29
PART 1: PRONUNCIATION
Choose the word whose underlined part is pronounced differently from the other three in each of
the following questions
1. A. contributed B. eradicated C. developed D. needed
Page 116
2. A. approached B. unwrapped C. obliged D. sacrificed
3. A. brush B. crush C. rush D. push
4. A. breathe B. threaten C. healthy D. earth
5. A. said B. play C. lemonade D. plate
6. A. bother B. breakthrough C. thoughtful D. geothermal
7. A. leaves B. practices C. wishes D. introduces
8. A. dough B. doubt C. county D. blouse
9. A. fracture B. signature C. culture D. mature
10. A. pride B. life C. combine D. machine
PART II. GRAMMAR AND VOCABULARY
I. Complete the following sentences by choosing the correct answer among four options (A, B, C or
D).
1. I'm addicted ______ these sweets. I loved them!
A. on B. at C. to D. for
2. You are being thoroughly _________in refusing to allow this ceremony to take place.
A. unrequited B. unrepresentative C. unreliable D. unreasonable
3. Nylon, a synthetic _______ from a combination of water, air, and a by-product of coal, was first
introduced in 1938.
A. to make B. make C. made D. making
4. He ______ book the tickets, but he had no time to call at the Cinema.
A. would B. might C. could D. was going to
5. It is necessary that an employee _________ his work on time.
A. finish B. finished C. finishing D. finishes
6. This dress is eye-catching, and many people________ me on it.
A. complain B. complement C. complimentD. complementary
7. Bill is jealous ________ your promotion.
A. with B. of C. at D. about
8. _______ are sought by an elementary school.
A. Mentally retarded teachers B. Teachers retarded for mental cases
C. Teachers for the mentally retardedD. Mental retarded case teachers
9. Since I came in half an hour late this morning, I have to stay until 5:30 to _______ for it.
A. make up B. save up C. keep up D. hold up
10. Earthquake can damage a tree ______violently, and it can take several years for the tree to heal.
A. to cause to shake it B. when shaking it causes
C. to cause shaking it D. by causing it to shake
II. In the following text, some of the lines are correct, and some have a word which should not be
there. If a line is correct, put a tick ( √). If a line has a word which should not be there, write
the word. There are two examples at the beginning.
We can tend to think of poverty as the main cause of violent 0____ can ____
crime. However, research from both sides of the Atlantic shows that 00___√_______
fathers may be the most important factor in preventing to children from 1. ___________
turning over to crime. In the United States, children from better-off
2. ___________
family were compared with ones who from families with lower
incomes. Children from both groups that lived with their fathers also 3. ___________
committed the same number of crimes. In the United Kingdom, a study 4. ___________
was carried out of comparing a group of boys who had never been 5. ___________
accused of not any crimes such as assault and stealing vehicles. All the 6. ___________
boys had difficulties at the school and came from large families that 7. ___________
didn't earn a lot of money. The biggest difference between from the 8. ___________
groups was that fifty-five percent of the "good boys" lived with their
9. ___________
fathers, while only four per cent of the "bad boys" did it. Eighty per cent
of the well – behaved boys said how they felt close to their fathers even 10. __________
if they didn't live in the same house. It seems that by having a father 11. __________

Page 117
who takes an interest in his children encourages youngsters not to break 12. __________
the law. 13. __________
14. __________
15. __________
III. Complete the following passage by supplying the correct form of the word to fill in
each blank.
There is a saying that first impressions are generally correct and I would say that
(1) _______ throughout my entire life, I have found this to be true apart from one ALL
notable exception of a good friend and neighbor with whom my first (2) ______ was CHANGE
decidedly unfriendly.
At the time of our first meeting I was living in a (3) __________  area of London,
not far from the exit of a dual carriageway and this meant that although it was a (4) DEPRIVE
____________ street, even with the (5) _______ will in the world, drivers would often RESIDENCE
travel dangerously fast within inches of my front door. This used to really annoy me GOOD
and I have to admit I did often go a bit (6) ________ in expressing my anger even BOARD
though (or perhaps because) I knew I didn't stand a (7) ________ chance of being REASON
taken notice of or even noticed at all. So when one day, I shouted my usual string of
swear words at a rapidly passing car and it immediately screamed to a halt, my first
reaction was an enormous sense of (8) ______. But then the driver of the car opened
his window and shouted a stream of swear words back at me. ACHIEVE
So the next morning, I was more than a little surprised to find an apology note
from the same motorist in my mail box explaining that in an attempt to (9) _______ FAR
his career as an up and coming IT (10) ______, he had been driving too fast the CONSULT
previous evening and inviting me to have dinner with him and his wife.

PART III. READING

I. Read the passage carefully, then choose the correct option (marked A, B, C or D) to answer the
questions.
The food we eat seems to have profound effects on our health. Although science have made
enormous steps in making food more fit to eat, it has, at the same time, made any foods unfit to eat.
Some research has shown that perhaps 80 percent of all human illness are related to diet and 40 percent
of cancer is related to diet as well, especially cancer of colon. People of different cultures are more
prone to contact certain illnesses because of the characteristics food they consume.
That food is related to illness is not a new discovery. In 1945, government researchers showed
that nitrates and nitrites (commonly used to preserve color in meats) as well as other food additives
caused cancer. Yet, these carcinogenic additives remain in our food, and it becomes more difficult all
the time to know which ingredients on the packaging labels of processed food are helpful or harmful.
The additives we eat are not all so direct. Farmers often give penicillin to cattle and poultry, and
because of this, penicillin has been found in the milk of treated cows. Sometimes similar drugs are
administered to animals not for medical purposes, but for financial reasons. The farmers are simply
trying to fatten the animals in order to obtain a higher price on the market. Although the Food and Drug
Administration (FDA) has tried repeatedly to control these procedures, the practices continue.
A healthy diet is directly related to good health. Often we are unaware of detrimental substances
we ingest. Sometimes, well – meaning farmers or others who do not realize the consequences add these
substances to food without our knowledge.
1. How has science done a disservice to people?
A. Because of science, disease caused by contaminated food has been virtually eradicated.
B. It has caused a lack of information concerning the value of food.
C. As a result of scientific intervention, some potentially harmful substances have been added to
our food.
D. The scientists have preserved the color of meats, but not of vegetables.
2. The word "prone" is nearest in meaning to ______.
A. supine B. unlikely C. healthy D. predisposed
Page 118
3. What are nitrates used for?
A. They preserve flavor in packaged foods. B. They preserve the color of meats.
C. They are the objects of research. D. They caused to animals to become fatter.
4. The word "these" refers to_______.
A. meats B. colors C. researchers D. nitrates and nitrites
5. All of the following statements are true EXCEPT_______.
A. drugs are always given to animals for medical reasons.
B. some of the additives in our food are added to the food itself and some are given to the living
animals.
C. researchers have known about the potential hazards of food additives for more than 60 years.
D. food may cause 40 percent of cancer in the world

II. Read the passage and fill in the blank a suitable word.

PEER PRESSURE
One of the strongest influences on children today (1) ______ that of their peers. What their classmates
think, how they dress and how they act in class and out of it (2) ________ the behavior of nearly every
child at school. In their efforts not to be different, some children go so far as to hide their intelligence
and ability in case they are made ( 3)_________ of. Generally, children do not want to stand (4)
_______ from the crowd. They want to fit in, to be accepted. In psychological terms, the importance of
(5)_________ pressure cannot be over emphasized. There is a lot of evidence that it has great
bearing on all aspects of children's lives, (6) ________ the clothes they wear, the music they listen to
and their attitude to study to their ambitions in life, their relationships and their sense. (7)
_________, as children grow up into adolescents, individuality becomes more acceptable, desirable
even, and in their search for their (8) _______ personal style, the teenager and young adult will begin to
experiment and be more willing to run the (9) _______ of rejection by the group. Concern about
intellectual prowess and achieving good exam results can dominate as the atmosphere of competition
develops and worries (10) ______ the future override any fears of appearing too brainy.
III. Complete the following passage by choosing A, B, C or D to fill in each blank.

Last year Gladys Kalema became the Ugandan Wildlife Service's chief and only vet after (1)
_____ for the Royal Veterinary College in London. She was the first person to fill the post for 30 years
and at the age of 26, easily the youngest.
If Gladys did nothing else, caring for the world's (2) ______ population of 650 gorillas would
alone justify her wages. Since the 1970s, gorillas have (3) ______ severely from war and poaching. Now
for $150 each, tourists can be led through the forest and come within five meters of gorilla – no closer,
for (4) ______ of transmitting diseases such as measles and flu.
The gorillas here make a small but viable population. (5) ______ in the national parts the usual
animals, elephants, rhinos, giraffes, are either not there or present in insignificant numbers which are
dangerously out of (6) ______ with the creatures around them. If Uganda stays calm, wildlife may, in (7)
_______, return by itself. But Gladys believes the country cannot wait. Animals must be brought in to
swell tourism and provide (8) ______to expand her work.
Despite her difficulties, Gladys feels more useful and fulfilled than she would be anywhere else.
"At this moment, my friends from vet school are reading the best way to (9) ______ a cat or dog, and
here am I planning to translocate elephants. In my small (10) _______ I am part of the reconstruction
and rehabilitation of my country."
1. A. leaving B. qualifying C. graduating D. passing
2. A. living B. surviving C. continuing D. lasting
3. A. endured B. harmed C. died D. suffered
4. A. risk B. fear C. fright D. danger
5. A. Somewhere B. Anywhere C. Elsewhere D. Nowhere
6. A. balance B. relation C. comparison D. equality

Page 119
7. A. terms B. years C. ages D. time
8. A. figures B. funds C. accounts D. savings
9. A. treat B. prescribe C. heal D. operate
10. A. means B. manner C. method D. way

PART IV. WRITING


I. Write the new sentences using the word give. Do not change the word given in any way.
1. She felt she had achieved a lot in life, despite her disadvantaged background (SENSE)
………………………………………………………………………………………………………………
2. I think you should try to be optimistic as you can. (SIDE)
………………………………………………………………………………………………………………
3. In the end, it was quite a good sunny day after all. (OUT)
………………………………………………………………………………………………………………
4. I hate watching late night films on TV. (STAND)
………………………………………………………………………………………………………………
5. The principal was the first person who arrived at the meeting. (ARRIVE)
……………………………………………………………………………………………………………

II. Writing a topic.

Computers can translate all kinds of languages well, so our children needn’t learn more
languages in the future.
What is your opinion about this idea? Within 200-250 words, use evidence and examples to write
a passage to clarify your idea.

ENGLISH PRACTICE 30
PART A: PRONUNCIATION
Choose the word whose underlined part is pronounced differently from the other three in each of
the following questions
1. A. imagine B. discipline C. determine D. Valentine
2. A. purses B. blouses C. amuses D. Pleases
3. A. lie B. goalie C. society D. pie
4. A. finished B. jagged C. packed D. punched
5. A. accurate B. tale C. shape D. date
6. A. situations B. obstacles C. secrets D. Sounds
7. A. chasm B. phrase C. suitcase D. chase
8. A. vineyard B. Finland C. business D. ignite
9. A. touched B. crooked C. missed D. watched
10. A. dedicate B. private C. eliminate D. educate
PART B. LEXICO-GRAMMAR
I. Complete each of the following sentences with the correct answer (A, B, C or D). Identify your answer
by writing the corresponding letter A, B, C or D on your answer sheet.
1. I’m really sorry. We _____ stop at a service station and phone you, but we didn’t want to waste any
more time.
A. were due to B. were going to C. were to D. were about to
2. It is not until December 25 _________the exam results will be announced.
th

A. which B. what C. that D. when


3. I saw a ___________ scarf in a shop near my house.
A. sweet little green silk B. little green silk sweet
Page 120
C. sweet green little silk D. green little sweet silk
4. There has been a recommendation that Peter ______ the president of the country.
A. will be elected B. be elected C. is elected D. was elected
5. He kept his marriage for years, but eventually the truth ________.
A. came out B. went out C. came through D. fell out
6. It was in this house ______.
A. I was born in B. in which I was born C. where I was born D. that I was born
7. ______of half- starving wolves were roaming the snow- covered countryside.
A. Herds B. Flocks C. Packs D. Swarms
8. When ______ to explain his mistake, the new employee cleared his throat nervously.
A. asking B. to be asked C. to be asking D. being asked
9. ______, he felt so unhappy and lonely.
A. In spite of his being wealth B. Rich as was he
C. Rich as he was D. Despite his wealthy
10. According to the _______of the contract, tenants must give six months notice if they intend to leave.
A. laws B. rules C. terms D. details
11. It is difficult for museums to find funds to protect the nation’s_______.
A. inheritance B. heritage C. possessions D. legacy
12. A part-time job gives me the freedom to_______ my own interests.
A pursue B. chase C. seek D. catch
13. When his alarm went off, he shut it off and slept for_______15 minutes.
A. other B. others C. another D. the others
14. ____ I’ve told him not to go out with those people, but he wouldn’t listen. Just let him face the music
now.
A. Many a time B. Many the time C. Quite a time D. For a time
15. _______ in this national park declined from a few thousand to a few hundred in ten years.
A. For a number of tigers B. The number of tigers
C. A number of tigers D. That the number of tigers
16. _______ every industry in our modern world requires the work of engineers.
A. Wholly B. Hardly C. Most D. Virtually
17. Jane had a problem with her finances, so we talked _______ and now it's fine.
A. over B. it over C. over it D. over and over
18. When the electricity failed, he _______a match to find the candles.
A. rubbed B. scratched C. struck D. started
19. I usually buy my clothes _______. It’s cheaper than going to the dressmaker.
A. on the house B. off the peg C. in public D. on the shelf
20. She tried to _______.
A. talk out of me the plan B. talk me the plan out of
C. talk me out of the plan D. talk out me of the plan.
II. In most lines of this text there is one unnecessary word. It is either incorrect grammatically, or
does not fit the sense of the text. Write the unnecessary word in the space beside the text. Tick (√)
each correct line. There are two examples at the beginning. Transfer your answers to your answer
sheet.
Talent scouts are looking for the next generation of supermodels have realized Africa’s 0 ……are…
potential. Lyndsey Mclntyre, a former model herself, recently opened one agency’s first 00 …
African office. “African women are being graceful and serene” she says. “These qualities √………
could to make them do very well in this business.” However, spotting supermodels is rarely 1 ……….
easy, as well Mclntyre discovered when she visited the Orma tribe of remote north- eastern ……
Kenya, whose women are reported to be especially striking. “The tribal leaders were a bit 2 ………..
suspicious and I wasn’t allowed to be meet many of their girls,” she explains. Another …
problem is that the reports aren’t always reliable. Mclntyre discovered this when one of 3 ………….
village’s “most beautiful girls” turned out to be its heaviest ones. She had to explain that …
Western advertisers prefer far slimmer women. The Orma are not alone in believing fat it is 4 ……..
beautiful. In a recent Africa-wide beauty contest, all the Uganda contestants were ……
disqualified for being a little too large around the hips. “I don’t understand the fashion 5 ……….
Page 121
industry’s obsession with small hips,” said one judge for the contest. “But because we want ……
the girls to succeed and to see African models working internationally, we give the industry 6 ………….
what it wants” …
7 ………….

8
…………….
9 …….
…….…
10………..
……

III. Supply the correct form of the word in capital letter. Write your answers on your answer
sheet.
UK companies have received criticism from a business forum for what their report refers to as a
rather narrow-minded attitude towards the dress code for office workers. This follows a case in which a
male (1. EMPLOY) __________working in the post room of a large (2. ORGANISE) __________in
the United Kingdom received a (3. SUSPEND) __________ for wearing jeans to work. Whilst the
report accepts that there is a need for people dealing with (4. CUSTOM) __________to look well
dressed, it questions whether employees who work behind the scenes necessarily need to dress
formally. The authors of the report made a (5. COMPARE)  ___________ between the UK and other
European nations where employers seem (6. CONCERN)___________about the need for their workers
to wear smart clothes in the office. Their (7. ARGUE) ___________is based on research that claims
workers are far more (8. PRODUCT) _________ when they have the (9. FREE) _________to dress in
a way that they feel most (10. COMFORT) _______in.
IV. Complete the following sentences with one preposition/particle for each blank. Write your
answers on your answer sheet.
1. Kate fell _______ with her boyfriend and they stopped seeing each other.
2. I’ve taken this watch _______ pieces, and now I can’t put it together again.
3. I’m sure my brother will never get married because he hates the feeling of being tied _____.
4. Deborah is going to take extra lessons to catch up______ what she missed why she was away.
5. I don’t think anyone understood what I was saying at the meeting. I failed to get my point _______.

PART C. READING
I. Read the following passage and choose the options among A, B, C or D that best complete the
blanks. Write your answers on your answer sheet.
Stressful situations that (1)________ almost everyday in life seem to be unavoidable. However,
we can do little sometimes to avoid a misfortune or an unpleasant occurrence which may (2)________
us unexpectedly as only it can. At such a moment, one may hit the (3)_______, give in to the
helplessness of the situation or, ideally, put a brave face on it trying to (4)_________ the burden.
Can you (5)_________ in your mind an hour spent in a traffic jam, say, this morning? Do you
light one cigarette after another? Do you sound the horn every few seconds like the other neurotics? Or
do you take a different (6)________ and make good use of the time drawing up a schedule for the days
to come? To withstand the stressful moment you can also do a crossword puzzle, listen to your favorite
music or even compose a menu for your Sunday dinner.
In fact, whatever way you (7)________ to the annoying situation, you can exert no impact on it
as the traffic jam will only reduce in due (8)_________. Nevertheless, your reaction might considerably
influence your mood for the rest of the day. The inability to confront a stressful occurrence like that with
a deal of composure and sensibility adds much more strain to your life and in this way puts your well –
being in (9)_________. Surprisingly, it is seemingly negligible hardships we stumble on daily that run
double the risk of developing serious health disorders rather than our isolated tragedies however painful
they may be. (10)_______that so many of those wretched stresses and inducing troubles affect us in a

Page 122
day, we should, at best, try to avoid them or possibly make radical alterations in the way we lead our
daily lifestyles.
1. A. devise B. create C. originate D. emerge
2. A. arise B. happen C. befall D. occur
3. A. post B. roof C. bottom D. wall
4. A. subsist B. remain C. cow D. bear
5. A. envision B. observe C. picture D. image
6. A. manner B. stance C. practice D. mode
7. A. strike B. deal C. respond D. challenge
8. A. term B. course C. timing D. period
9. A. risk B. weakness C. insecurity D. jeopardy
10. A. Providing B. Given C. Hence D. As much

II. Read the text below and think of the word which best fits each space. Use only ONE word in
each space. Write your answers on your answer sheet.
Students frequently complain (1)___________ studying for hours on (2)________ and then not
doing well in their final exams. Many factors can (3) _________ in poor performances: (4)
_________illness or a personal problem. Getting high grades can also put a terrible strain on students (5)
_________ don’t want to let their parents down.
Can students’ knowledge be judged by a (6) _________exam? Certainly not! If we want to be
fair, students ought to be (7) _________on a regular basic. So does that mean more exams? Yes, but (8)
_________many written ones. In many countries, student do not have to take written exams (9)
_________ they don’t want to, they are (10) _________to take oral ones instead. This seems to be fairer
way of assessing understanding, not just testing it.
III. Read the passage then choose the best answer to each question that follows. Identify your answer by
writing the corresponding letter A, B, C or D on your answer sheet.
Quite different from storm surges are the giant sea waves called tsunamis, which derive their
name from the Japanese expression for “high water in a harbor.” These waves are also referred to by the
general public as tidal waves, although they have relatively little to do with tides. Scientists often
referred to them as seismic sea waves, far more appropriate in that they do result from undersea seismic
activity.
Tsunamis are caused when the sea bottom suddenly moves, during an underwater earthquake or
volcano for example, and the water above the moving earth is suddenly displaced. This sudden shift of
water sets off a series of waves. These waves can travel great distances at speeds close to 700 kilometers
per hour. In the open ocean, tsunamis have little noticeable amplitude, often no more than one or two
meters. It is when they hit the shallow waters near the coast that they increase in height, possibly up to
40 meters.
Tsunamis often occur in the Pacific because the Pacific is an area of heavy seismic activity.Two
areas of the Pacific well accustomed to the threat of tsunamis are Japan and Hawaii. Because the seismic
activity that causes tsunamis in Japan often occurs on the ocean bottom quite close to the islands, the
tsunamis that hit Japan often come with little warning and can, therefore, prove disastrous. Most of the
tsunamis that hit the Hawaiian Islands, however, originate thousands of miles away near the coast of
Alaska, so these tsunamis have a much greater distance to travel and the inhabitants of Hawaii generally
have time for warning of their imminent arrival.
Tsunamis are certainly not limited to Japan and Hawaii. In 1755, Europe experienced a
calamitous tsunami, when movement along the fault lines near the Azores caused a massive tsunami to
sweep onto the Portuguese coast and flood the heavily populated area around Lisbon. The greatest
tsunami on record occurred on the other side of the world in 1883 when the Krakatoa volcano
underwent a massive explosion, sending waves more than 30 meters high onto nearby Indonesian
islands; the tsunami from this volcano actually traveled around the world and was witnessed as far away
as the English Channel.

1. The paragraph preceding this passage most probably discusses _______.


A. underwater earthquakes B. storm surges C. tides D. tidal waves
Page 123
2. According to the passage, all of the following are true about tidal waves EXCEPT that _______.
A. they are caused by sudden changes in high and low tides
B. this terminology is not used by the scientific community
C. they are the same as tsunamis
D. they refer to the same phenomenon as seismic sea waves
3. The word “displaced” is closest in meaning to _______.
A. moved B. filtered C. located D. not pleased
4. It can be inferred from the passage that tsunamis _______.
A. cause severe damage in the middle of the ocean
B. generally reach heights greater than 40 meters
C. are far more dangerous on the coast than in the open ocean
Dare often identified by ships on the ocean
5. Water that is “shallow” is NOT _____________.
A. deep B. clear C. coastal D. tidal
6. A main difference between tsunamis in Japan and in Hawaii is that tsunamis in Japan are more likely
to ___________.
A. come from greater distances B. originate in Alaska
C. arrive without warning D. be less of a problem
7. The possessive “their” refers to ____________.
A. the Hawaiian Islands B. thousands of miles
C. these tsunamis D. the inhabitants of Hawaii
8. A “calamitous” tsunami is one that is _____________.
A. at fault B. disastrous C. extremely calm D. expected
9. From the expression “on record”, it can be inferred that the tsunami that accompanied the Krakatoa
volcano ____________.
A. was not as strong as the tsunami in Lisbon B. might not be the greatest tsunami ever
C. was filmed as it was happening D. occurred before efficient records were kept
10. The passage suggests that the tsunami resulting from the Krakatoa volcano ___________.
A. was far more destructive close to the source than far away
B. resulted in little damage
C. was unobserved outside of the Indonesian islands
D. caused volcanic explosions in the English Channel

PART D. WRITING
I. Rewrite the following sentences without changing their meaning, using the words given. These
words must not be changed in any way. Write your answers on your answer sheet.
1. The reduction in the price of the magazines led to a growth in sales. (RESULT)
Sales of the magazines __________________________________________.
2. Georgia hasn’t written to me recently. (HEARD)
I ___________________________________________________________.
3. Dinner will be served immediately upon our arrival at the hotel. (SOON)
Dinner will be served ___________________________________________.
4. Barbara couldn’t sing or dance. (UNABLE)
Besides ______________________________________________________.
5. It was careless of you to leave without locking the door. (OUGHT)
You _________________________________________________________.

II. Write an essay to express your opinion on the following statement. Use specific reasons and
details to support your idea.
“Face-to-face communication is better than other types of communication, such as letters, email,
or telephone calls”.

Page 124
Page 125
Page 126
Page 127
Page 128
Page 129
Page 130
Page 131

You might also like